ECONOMY UPSC

Previous Year Questions

www.laex.in Page No. 1 https://elearn.laex.in

www.laex.in

ECONOMY UPSC

Previous Year Questions

www.laex.in https://elearn.laex.in

www.laex.in

ECONOMY UPSC

Previous Year Questions

INDEX

No. of Page No. S.No. Topic Qns From To

1 National Income and Concepts of Economy 24 1 10

2 Planning 12 11 16

3 Poverty 06 16 20

4 Employment 13 20 26

5 Monetary policy 75 28 61

6 Fiscal Policy 34 61 80

7 Stock Market 15 80 86

8 Agriculture 26 86 101

9 Industry 20 101 111

10 Infrastructure 13 111 117

11 External Sector 48 117 135

12 International Economy 49 135 155

13 Indicies and Reports 04 156 158

14 Miscellaneous 10 158 163

www.laex.in https://elearn.laex.in

www.laex.in

ECONOMY UPSC

Previous Year Questions

1. National Income and 1.2 Economy Basics 5. A "closed economy" is an economy in Concepts of Economy which

1.1 Basic Concepts a) The money supply is fully controlled

1. A decrease in tax to GDP ratio of a b) Deficit financing takes place country indicates which of the c) Only exports take place following? d) Neither exports nor imports take place 1) Slowing economic growth rate 6. Match List-I with List II and select the 2) Less equitable distribution of national correct answer using the codes given income below the lists: Select the correct answer using the code List-I List-II given below. A. Boom 1. Business activity at high a) 1 only level with increasing b) 2 only income, output and c) Both 1 and 2 employment at macro d) Neither 1 nor 2 B. Recession 2. Gradual fall of income 2. Economic growth is usually coupled with output and employment with a) Deflation business activity in low gear b) Inflation C. Depression 3. Unprecedented level of c) Stagflation under employment and d) Hyperinflation

unemployment, drastic fall in 3. Economic growth in country X will income, output and necessarily have to occur if employment a) There is technical progress in the world D. Recovery 4. Steady rise in the general economy level of prices, income, output b) There is population growth in X and employment c) There is capital formation in X Codes: d) The volume of trade grows in the world A B C D economy a) 1 2 3 4 4. Despite being a high saving economy, capital formation may not result in b) 1 2 4 3 significant increase in output due to c) 2 1 4 3 a) Weak administrative machinery d) 2 1 3 4 b) Illiteracy 7. The supply-side economics lays greater c) High population density emphasis on the point of view of d) High capital-output ratio

www.laex.in Page No. 1 https://elearn.laex.in

www.laex.in

ECONOMY UPSC

Previous Year Questions

a) Producer c) 4 1 2 3 b) Global economy d) 4 1 3 2

c) Consumer 10. Consider the following statements: d) Middle-man Human capital formation as a concept is

8. A consumer is said to be in equilibrium, better explained in terms of a process, if which enables a) He is able to fulfil his need with a given level 1) Individuals of a country to accumulate more of income capital. 2) Increasing the knowledge, skill levels and b) He is able to live in full comforts with a capacities of the people of the country. given level of income 3) Accumulation of tangible wealth. c) He can fulfil his needs without consumption 4) Accumulation of intangible wealth. of certain items Which of the statements given above is/are d) He is able to locate new sources of income correct?

9. The product life cycle from inception to a) 1 and 2 demise is shown in the graph. Match List b) 2 only I with List II and select the correct c) 2 and 4 answer using the codes given below: d) 1, 3 and 4

List I (Stage) 1) Product Development 11. In the context of Indian economy, consider the following pairs: 2) Maturity Term Most appropriate 3) Growth Description 4) Introduction 1) Melt down Fall in stock prices List II (Zone) 2) Recession Fall in growth rate 3) Slow down Fall in GDP Which of the pairs given above is/are correctly matched? a) 1 only b) 2 and 3 only c) 1 and 3 only d) 1, 2 and 3

Codes: 1.3 Current Trends in GDP Growth Rate A B C D 12. With reference to Indian economy, a) 1 4 2 3 consider the following statements: b) 1 4 3 2 www.laex.in Page No. 2 https://elearn.laex.in

www.laex.in

ECONOMY UPSC

Previous Year Questions

1) The rate of growth of Real Gross Domestic 15. Which one among the following Product has steadily increased in the last countrieshas the lowest GDP per capita? decade. a) China 2) The Gross Domestic Product at market b) India prices (in rupees) has steadily increased in c) Indonesia the last decade. d) Sri Lanka Which of the statements given above is/are 1.4 Measuring National Income

correct? 16. The national income of a country for a a) 1 only given period is equal to the b) 2 only a) Total value of goods and services produced c) Both 1 and 2 by the nationals d) Neither 1 nor 2

b) Sum of total consumption and investment 13. In the context of Indian economy, expenditure consider the following statements: c) Sum of personal income of all individuals 1) The growth rate of GDP has steadily d) Money value of final goods and services increased in the last five years. produced 2) The growth rate in per capita income has 17. The most appropriate measure of a steadily increased in 5 years. country's economic growth is its Which of the statements given above is/are a) Gross Domestic Product correct? b) Net Domestic Product a) 1 only c) Net National Product b) 2 only d) Per Capita Real Income

c) Both 1 and 2 d) Neither 1 nor 2 18. The term National Income represents a) Gross national product at market prices 14. With reference to Indian economy, minus depreciation consider the following statements: b) Gross national product at market prices 1) The Gross Domestic Product (GDP) has minus depreciation plus net factor income increased by four times in the last 10 from abroad years c) Gross national product at market prices 2) The percentage share of Public Sector in minus depreciation and indirect taxes plus GDP has declined in the last 10 years subsidies Which of the statements given above is/ are d) Gross national product at market prices correct? minus net factor income from abroad

a) 1 only b) 2 only 19. The growth rate of per capita income at current prices is higher than that of per c) Both 1 and 2 capita income at constant prices, d) Neither 1 nor 2

www.laex.in Page No. 3 https://elearn.laex.in

www.laex.in

ECONOMY UPSC

Previous Year Questions

because the latter takes into account the 1) Agriculture, Forestry and Fishing rate of 2) Manufacturing a) Growth of population 3) Trade, Hotels, Transport and b) Increase in price level Communication c) Growth of money supply 4) Financing, Insurance, Real Estate and d) Increase in the wage rate Business Services 20. In an open economy, the national Choose The decreasing order of the income (Y) of the economy is: (C, I, G, X, contribution of these sectors to the Gross M stand for Consumption, Investment, Domestic Product (GDP) at factor cost at Government Expenditure, total exports constant price and total imports respectively). a) 3, 1, 2, 4 a) Y = C+I + G + X b) 1, 3, 4, 2 b) Y = C+ I + G - X+ M c) 3, 4, 1, 2 c) Y = C + I+ G+ (X- M) d) 1, 3, 2, 4

d) Y = C + IG+ X- M 24. Since 1980, the share of the tertiary

21. Increase in absolute and per capita real sector in the total GDP of India has GNP do not connote a higher level of a) Shown an increasing trend economic development, if (2018) b) Shown a decreasing trend a) Industrial output fails to keep pace with c) Remained constant agricultural output. d) Been fluctuating

b) Agricultural output fails to keep pace with industrial 1.Key and Explanations

c) Poverty and unemployment increase. 1.1 Basic Concepts d) Imports grow faster than exports. 1. Answer: C

1.5 Sectorial Contribution to Explanation: The tax-to-GDP ratio is a ratio of a nation's tax revenue relative to its gross GDP Growth

domestic product (GDP), or the market value of 22. Which one of the following is the correct goods and services a country produces. Taxes sequence in the decreasing order of and GDP are generally related. The higher the contribution of different sectors to the GDP, the more tax a nation collects. Gross Domestic Product of India? Conversely, countries with lower taxes produce a) Services- Industry - Agriculture a lower GDP. A low tax to GDP ratio b) Services - Agriculture - Industry indicates lower economic development and less c) Industry - Services - Agriculture equitable distribution of wealth. The tax-to-GDP d) Industry- Agriculture - Services ratio is an economic measurement that

23. With reference to the Indian economy compares the amount of taxes collected by a consider the following activities: government to the amount of income that

www.laex.in Page No. 4 https://elearn.laex.in

www.laex.in

ECONOMY UPSC

Previous Year Questions

country receives for its products. Hence both Educational objective: To learn about the key the statements are correct. factors which influences the economic growth in Educational Objective: To know about Tax to country.

GDP ratio.

4. Answer : D 2. Answer: B Explanation: Capital formation means Explanation: Economic growth results in higher increasing the stock of real capital in a country. disposable income available with the consumers In other words, capital formation involves which increases the overall demand along with making of more capital goods such as machines, the supply available for the consumers. This tools, factories, transport equipment, materials, increase in demand spurs inflation which electricity, etc., which are all used for future increases the general prices of goods and production of goods. services in the market, which eventually For making additions to the stock of Capital, becomes a necessary evil for a growing economy. saving and investment are essential. Capital Hence economic growth is usually coupled with output ratio is the amount of capital needed to inflation. produce one unit of output. For example, Deflation: reduction of the general level of suppose that investment in an economy, prices in an economy. investment is 32% (of GDP), and the economic Stagflation: persistent high inflation combined growth corresponding to this level of investment with high unemployment and stagnant demand is 8%. Here, a Rs 32 investment produces an in a country's economy. output of Rs 8. Capital output ratio is 32/8 or 4. Hyperinflation is very high and typically In other words, to produce one unit of output, 4 accelerating inflation. It quickly erodes the real unit of capital is needed. Hence, if the capital- value of the local currency, as the prices of all output ratio is high, there will not be significant goods increase. increase in output despite high savings and Educational Objective: To know about inflation investment. and its impact on economic growth. So, clearly with high savings growth or output

3. Answer : C may remain low because of high capitaloutput Explanation:Technical progress and volume of ratio. trade does not say anything about the country Educational objective: To learn and X, only the general world economy. understand about capital output ratio. If there is population growth without any 1.2 Economy Basics avenues for the economy to absorb them, it will 5. Answer : D have a deteoriarating effect. Explanation : A closed economy is one that has Capital formation in the country x is the most no trading activity with outside economies. The appropriate option, as whenever there is capital closed economy is therefore entirely self- formation in country it will lead to growth. sufficient, which means no imports come into the country and no exports leave the country. www.laex.in Page No. 5 https://elearn.laex.in

www.laex.in

ECONOMY UPSC

Previous Year Questions

The goal of a closed economy is to provide while demand starts to increase slowly and so domestic consumers with everything they need does investment and employment. from within the country's borders. The phases of trade cycle happen during the In reality, there are no nations that have long run and not in short periods. economies that are completely closed because Educational objective : To learn about raw materials, such as crude oil, play a vital role Business cycle and its phase

as inputs to final goods. Many countries do not 7. Answer : A have raw materials naturally and are forced to Explanation : Supply-side economics is a import these resources. macroeconomic theory arguing that economic Educational objective : To learn about types of growth can be most effectively created by economies. lowering taxes and decreasing regulation, by 6. Answer : A which it is directly opposed to demand-side Explanation : Business cycles are continuous economics. phases of good and bad changes that take place Demand-side economics is a macroeconomic in an economy. theory which maintains that economic growth A Business cycle /trade cycle has four phases: and full employment are most effectively created Boom: this phase is also known as prosperity or by high demand for products and services. peak. During this phase growth level is According to demand-side economics, output is maximum. Income, demand, investments and determined by effective demand. profits are high. Supply side economics lays greater emphasis on Recession: this phase comes after the phase of the point of view of the producer whereas the boom when economic activities have reached the demand side economics lays emphasis interest highest level it is then followed by a slow down. of the consumer. In this phase demand, investment and Educational objective : To learn about supply employment slows down. side and Demand side economics Depression: the phase of recession continues to 8. Answer : A the phase of depression. Explanation : Consumer Equilibrium is the There is complete slow down of all economic state of balance obtained by an end-user of activities. Consumers expect lower prices and products that refers to the number of goods and this causes a fall in demand, production, services they can buy given their existing level of employment. income and the prevailing level of cost prices. Recovery: after the phase of depression which Consumer equilibrium permits a customer to causes low level of economic activities, get the most satisfaction possible from their government may try to help and bring changes income. for the economy to function properly. After a Educational objective : To learn about Consumer Equilibrium. www.laex.in Page No. 6 https://elearn.laex.in

www.laex.in

ECONOMY UPSC

Previous Year Questions

9. Answer : B values, corporate losses etc. that hurt the Explanation : A product life cycle is the amount economy and lead to losses for investors. of time a product goes from being introduced Recession : When the GDP(Gross Domestic into the market until it's taken off the shelves. Product) of a country falls for two continuous The Product Life-Cycle (PLC) concept indicates quarters( a quarter is 3 months in a year), then as to what can be expected in the market for a it is assumed that the country is sliding into a new product at various stages i.e., recession. The five stages of the PLC are: (in order) Slow down : means that the pace of the GDP 1) Product development growth has decreased. 2) Market introduction During slowdown, the GDP growth is still 3) Growth positive but the rate of growth has decreased. 4) Maturity Educational objective : To learn about Melt 5) Decline down vs. Slow down vs. Recession The concept of product life cycle helps inform 1.3 Current Trends in GDP growth rate

business decision-making, from pricing and 12. Answer: B promotion to expansion or cost-cutting. Explanation: The rate of growth of Real Gross Educational objective : To know about Product Domestic Product has not steadily increased in Life-Cycle (PLC) concept.

the last decade. Hence statement 1 is incorrect 10. Answer : C Explanation : Human capital is a stock of skill and expertise of a nation at a particular point of time. It raises individual efficiency and productivity thereby raising the aggregate production and economic well being of a country. It is intangible as compared to physical wealth.

Sources of human capital formation are Statement 2 is correct as The Gross Domestic Expenditure on education, Expenditure on Product at market prices (in rupees) has steadily health, On the job training, Study programmes increased in the last decade as the current for adults, Migration and expenditure on market prices of commodities keep increasing information. over a period of time. Educational objective : To learn about Human Educational Objective: To know about Trends capital formation. in Gross Domestic Products.

11. Answer : A 13. Answer : D Explanation : Melt down : Refers to events like Explanation: Statement 1 is incorrect as GDP steep fall in stock markets, decline in asset of India in last 5 years has not grown steadily. www.laex.in Page No. 7 https://elearn.laex.in

www.laex.in

ECONOMY UPSC

Previous Year Questions

prosperity of a country based on its economic growth. Small, rich countries and more developed industrial countries tend to have the highest per capita GDP. Among the given options India has got the lowest GDP per capita. Educational Objective: Understanding about  Statement 2 is incorrect as the growth GDP per capita.

rate in per capita income has not steadily 1.4 Measuring National Income

increased in the last five years. 16. Answer : A Explanation: National income of a country can be defined as the total market value of all final goods and services produced in the economy in a year. Two things must be noted in regard to this meaning of national income. First, it measures

the market value of annual output. In other Educational Objective: To know about Trends words, national income is a monetary measure. in growth rate.

Educational Objective: To know about National 14. Answer : B income. Explanation: India’s GDP in 2008 was 1224.1 17. Answer : D billion USD and in 2018 is 2718.73 billion USD, Explanation: Real GDP per capita is a hence statement 1 is incorrect measurement of the total economic output of a Statement 2 is correct as The percentage share country divided by the number of people and of Public Sector in GDP has declined in the last adjusted for inflation. It's used to compare 10 years. the standard of living between countries and Educational Objective: To know about DP over time. This economic indicator consists of

15. Answer : B the following three concepts. You must Per capita gross domestic product (GDP) is a understand these first if you want to metric that breaks down a country's economic comprehend GDP per capita. output per person and is calculated by dividing The first concept is “gross domestic product.” the GDP of a country by its population. That measures everything that a country Per capita GDP is a global measure for gauging produces in a year. The components of the prosperity of nations and is used by GDP are personal consumption, business economists, along with GDP, to analyze the investment, government spending and exports minus imports. www.laex.in Page No. 8 https://elearn.laex.in

www.laex.in

ECONOMY UPSC

Previous Year Questions

The second is “real GDP,” which is GDP without 20. Answer: C the effect of price changes. Inflation makes Explanation: There are three ways of regular, “nominal” GDP higher, so real GDP is a calculating GDP - all of which in theory should more accurate measurement when you want to sum to the same amount: compare an economy over time. National Output = National Expenditure The third is “per capita,” which means “per (Aggregate Demand) = National Income person.” Real GDP is divided by the population The Expenditure Method - Aggregate Demand of a country to calculate real GDP per capita. It's (AD): the best way to compare economic indicators o The full equation for GDP using this like GDP for countries with very different approach is population sizes. Hence per capita real income o GDP = C + I + G + (X-M) where is most appropriate measure of a country's o C: Household spending on goods and economic growth services Educational Objective: To know about o I: Capital Investment spending Measurement of GDP and Difference in real and o G: Government spending nominal GDP. o X: Exports of Goods and Services

18. Answer : C o M: Imports of Goods and Services Explanation: Option A refers to Net national The Income Method – adding together factor product (NNP) incomes Refers to gross national product (GNP), i.e. the GDP is the sum of the incomes earned through total market value of all final goods and services the production of goods and services. This is : produced by the factors of production of a Income from people in jobs and in self- country or other polity during a given time employment (e.g. wages and salaries) + period, minus depreciation. Profits of private sector businesses + Rent Educational Objective: To know about Net income from the ownership of land Gross national product. Domestic product (by sum of factor incomes)

19. Answer : B Only those incomes that are come from the Explanation: The growth rate of per capita production of goods and services are included in income at current prices is higher than that of the calculation of GDP by the income approach. per capita income at constant prices, because We exclude: Transfer payments e.g. the state the latter takes into account the rate of inflation pension; income support for families on low (increase in price level of goods and services) incomes; the Jobseekers’ Allowance for the which is higher than the constant price which unemployed and other welfare assistance such makes the per capita income at current prices housing benefit and incapacity benefits more than constant price Private transfers of money from one individual Educational Objective: To know about Per to another capita income calculation. Income not registered with the tax authorities Every year, billions of pounds worth www.laex.in Page No. 9 https://elearn.laex.in

www.laex.in

ECONOMY UPSC

Previous Year Questions

of activity is not declared to the tax authorities. reflectors of overall economic development in the This is known as the shadow economy. society. Published figures for GDP by factor incomes will As the question clearly mentions effects on be inaccurate because much activity is not development, we must take into account impact officially recorded– including subsistence of poverty and unemployment of economic farming and barter transactions growth despite having good parameters of Gross Value Added and Contributions to a progress in growth, per capita income etc. nation’s GDP. Educational objective: To learn about the basic There are three main wealth-generating aspects of absolute and per capita real GNP.

sectors in an economy – manufacturing and 1.5 Sectorial Contribution to GDP construction, primary (including oil& Growth gas, farming, forestry & fishing) and a wide

range of service-sector industries. 22. Answer: A This measure of GDP adds together the value of Explanation: Contribution of different sectors output produced by each of the productive to national income is sectors in the economy using the concept  Service sector: 55.1% of value added. .  Industry : 26.4% Value added is the increase in the value of  Agriculture: 18.5% goods or services as a result of the production  Hence option A is the correct order process  Educational Objective: To know about Value added = value of production - value of Sectorial contribution to national income.

intermediate goods. 23. Answer : D Educational Objective: To know about Explanation: According to present figures the calculation of Gross value Added. correct order will be 4.3.2.1.

21. Answer : C According to present figures the correct Explanation An essential aspect of development order will be 4.3.2.1 is to enable the maximum number to experience Educational Objective: To know abou sectorial the fruits of development. Concepts of per capita contribution of GVA.

income (per capita GDP or per capita NSDP) are 24. Answer : A not able to capture this aspect of development. Explanation: The contribution of tertiary sector There may be a case wherein increase in to total GDP of India has shown an increasing absolute and per capita GNP is reflective of trend from 38.57% in 1979-80 to 57% in 2018, growth in income of a small section of society hence Option A is correct. and that majority of the population is poverty Educational Objective: To know about stricken and unemployed. Multi -dimensional Sectorial contribution to GDP. non -monetary social indicators are better

www.laex.in Page No. 10 https://elearn.laex.in

www.laex.in

ECONOMY UPSC

Previous Year Questions

2. Planning 3. In the Fifth Five-Year Plan, for the first time, the financial sector was included as 2.1 Five Year Planning

an integral part of the Plan. 1. In the context of India's Five Year Select the correct answer using the code Plans, a shift in the pattern of given below. industrialization, with lower emphasis a) 1 and 2 only on heavy industries and more on b) 2 only infrastructure begins in c) 3 only a) Fourth Plan d) 1, 2 and 3

b) Sixth Plan 5. The main objective of the 12th Five Year c) Eight Plan Plan is d) Tenth Plan a) Inclusive growth and poverty reduction 2. Inclusive growth as enunciated in the b) Inclusive growth and sustainable growth Eleventh-Five Year Plan does not c) Sustainable and inclusive growth to include which one of the following: reduce unemployment a) Reduction of poverty d) Faster, sustainable and more inclusive b) Extension of employment opportunities growth

c) Strengthening of capital market d) Reduction of gender inequality 6. Consider the following statements regarding Indian Planning: 3. During which Five Year Plan, was the 1) The Second Five-Year Plan emphasized on Emergency clamped, new elections took the establishment of heavy industries. place and the Janata Party was elected? 2) The Third Five-Year Plan introduced the a) Third concept of import substitution as a strategy b) Fourth for industrialization. c) Fifth Which of the statements given above is/are d) Sixth correct?

4. With reference to India's Five-Year a) 1 only Plans, which of the following b) 2 only statements is / are correct? c) Both 1 and 2 1. From the Second Five-Year Plan, there was d) Neither 1 nor 2 a determined thrust towards substitution 7. Five Year Plan in India is finally of basic and capital good industries. approved by 2. The Fourth Five-Year Plan adopted the a) Union Cabinet objective of correcting the earlier trend of b) President on the advice of Prime Minister increased concentration of wealth and c) Planning Commission economic power. d) National Development Council

www.laex.in Page No. 11 https://elearn.laex.in

www.laex.in

ECONOMY UPSC

Previous Year Questions

2.2 Related Institutions (Like NITI 2.3 Miscellaneous

AAYOG) 11. The planning process in the industrial 8. With reference to 'Financial Stability sector in India has assumed a relatively and Development Council', consider the less important position in the nineties as following statements: compared to that in the earlier period. 1) It is an organ of NITI Aayog. Which one of the following is not true in 2) It is headed by the Union Finance this regard? Minister. a) With the advent of liberalisation, industrial 3) It monitors macroprudential supervision of investments/development have largely been the economy. placed within the domain of private and multinational sectors Which of the statements given above is / are b) With markets assuming a central place, the correct? role of central planning in many sectors has a) 1 and 2 only been rendered redundant b) 3 only c) The focus of planning has shifted to sectors c) 2 and 3 only like human resource development, d) 1, 2 and 3 infrastructure, population control and

9. Which of the following are associated welfare with 'Planning' in India? d) The nation's priorities have shifted away 1) The Finance Commission from industrial development to rural 2) The National Development Council development

3) The Union Ministry of Rural Development 12. Which of the following can be said to be 4) The Union Ministry of Urban Development essentially the parts of 'Inclusive 5) The Parliament Governance'? Select the correct answer using the code 1) Permitting the Non-Banking Financial given below. Companies to do banking a) 1, 2 and 5 only 2) Establishing effective District Planning b) 1, 3 and 4 only Committees in all the districts c) 2 and 5 only 3) Increasing the government spending on d) 1, 2, 3, 4 and 5 public health 4) Strengthening the Mid-day Meal Scheme 10. The has Select the correct answer using the codes established NITI Aayog to replace the given below: a) Human Rights Commission a) 1 and 2 only b) Finance Commission b) 3 and 4 only c) Law Commission c) 2, 3 and 4 only d) Planning Commission d) 1, 2, 3, and 4

www.laex.in Page No. 12 https://elearn.laex.in

www.laex.in

ECONOMY UPSC

Previous Year Questions

2. Key and Explanations education of 18–23 years of age group by 2011-

12. It focused on distant education, convergence 2.1 Five Year Planning of formal, non-formal, distant and it education

1. Answer: B institutions. Explanation: The objectives of the sixth five Rapid and inclusive growth (poverty reduction). year plan India were mainly focused on Emphasis on social sector and delivery of increasing industrialization and reducing long- service therein. standing problems such as poverty and Empowerment through education and skill unemployment. Some of the highlights and development. predominant aims of the sixth five year plan Reduction of gender inequality. India are enumerated as under: Environmental sustainability.  To increase the growth rate of the economy To increase the growth rate in agriculture,  To concentrate on the promotion of industry and services to 4%, 10% and 9% efficient use of resources respectively.  To improve productivity level Reduce total fertility rate to 2.1.  To initiate modernization for achieving Provide clean drinking water for all by 2009. economic and technological self-reliance Increase agriculture growth to 4%.  To control poverty and unemployment Educational Objective: To learn about the  To develop indigenous energy sources and various objectives and goals set as per the five efficient energy usage year plans

 To promote improved quality of life of the 3. Answer: C citizens Explanation: In India, "the emergency" refers to  To introduce minimum needs program for an 18-month period from 1975 to 1977 when the poor and needy with an emphasis to the then Prime minister had a state of reduce the discrepancies in income and emergency declared across the country. wealth accumulation Officially issued by President under article 352  To initiate family planning programs in of the constitution because of the prevailing order to check the growing population "internal disturbance", the emergency was in trends effect from 26 June 1975 until its withdrawal in  To protect and improve ecological and January 1977. environmental assets The order bestowed upon the Prime minister the  To promote the education at all levels authority to rule by decree, allowing elections to Educational Objectives: To learn about themes be suspended and civil liberties to be curbed. and objectives of five year plans. Educational Objective: To have knowledge about the political background and how they 2. Answer: C affected the respective five year plans. Explanation: Eleventh FYP (2007-2012) It

aimed to increase the enrolment in higher www.laex.in Page No. 13 https://elearn.laex.in

www.laex.in

ECONOMY UPSC

Previous Year Questions

4. Answer: B  To ensure that 50% of the rural population Explanation: From 1947 to 2017, the Indian have accesses to proper drinking water. economy was premised on the concept  To increase green cover by 1 million of planning. This was carried through the five- hectare every year. year plans, developed, executed, and monitored  To provide access to banking services to by the planning commission (1951-2014) and 90% of households. the NITI aayog (2015-2017). With the prime Educational Objective: To have knowledge about minister as the ex-officio chairman, the the main objectives and targets set by each five commission has a nominated deputy chairman, year plan.

who holds the rank of a cabinet 6. Answer: C minister. Montek Singhahluwalia is the last Explanation: The second five-year plan was deputy chairman of the commission (resigned based on the Mahalanobis model of growth on 26 may 2014). The twelfth plan completed its which gave more emphasis on setting up of the term in March 2017. heavy industries. Hence option 1 is correct. Prior to the fourth plan, the allocation of state The third five-year plan gave importance for self resources was based on schematic patterns reliance. In order to fulfil this, it had objective of rather than a transparent and objective import substitution as a strategy for mechanism, which led to the adoption of industrialisation. Hence option 2 is also the Gadgil formula in 1969. Revised versions of correct. the formula have been used since then to Educational Objective: To have knowledge determine the allocation of central assistance for about the main objectives and targets set by state plans. each five year plan. Educational Objective: To have knowledge 7. Answer: D about the main objectives and targets set by Explanation: The National Development each five year plan.

Council (NDC) or Rashtriya VikasParishad is the 5. Answer: D apex body for decision creating and Explanation: The objectives of the twelfth five- deliberations on development matters in India, year plan were: presided over by the prime minister. It was set  To create 50 million new work up on 6 august 1952 to strengthen and mobilize opportunities in the non-farm sector. the effort and resources of the nation in support  To remove gender and social gap in school of the five year plans made by planning enrolment. commission, to promote common economic  To enhance access to higher education. policies in all vital spheres, and to ensure the  To reduce malnutrition among children balanced and rapid development of all parts of aged 0–3 years. the country. The council comprises the prime  To provide electricity to all villages. minister, the union cabinet ministers, chief ministers of all states or their substitutes,

www.laex.in Page No. 14 https://elearn.laex.in

www.laex.in

ECONOMY UPSC

Previous Year Questions

representatives of the union territories and the also includes the chairman of the insolvency members of the NITI aayog (erstwhile planning and bankruptcy board (IBBI). commission). The council deals, inter-alia, with issues relating Educational Objective: To have knowledge to financial stability, financial sector about the planning process in India and the development, inter–regulatory coordination, bodies that play a key role in planning of the financial literacy, financial inclusion and macro country prudential supervision of the economy including the functioning of large financial conglomerates. 2.2 Related Institutions (Like NITI No funds are separately allocated to the council AAYOG) for undertaking its activities. 8. Answer: C Educational Objective: To have knowledge Explanation: Financial stability and about various bodies dealing with economy of development council (FSDC) is an apex-level the country

body constituted by the government of India. 9. Answer: C The idea to create such a super regulatory Explanation: National development council was body was first mooted by the Raghuram Rajan set up post-independence along with the Committee in 2008.A body dealing with macro planning commission to have a planned prudential and financial regularities in the economy for the country. Hence option 2 is entire financial sector of India. correct. An apex-level fsdc is not a statutory body Finance commission deals with the devolution of envisages to strengthen and institutionalise the finances between the centre and states and not mechanism of maintaining financial in the planning of the country. Hence option 1 is stability, financial sector development, inter- wrong. regulatory coordination along with By elimination method we can arrive at the monitoring macro-prudential answer that is option c regulation of economy. No funds are separately Educational Objective: To have knowledge allocated to the council for undertaking its about various constitutional and non activities. constitutional bodies dealing with planned The council is chaired by the union finance economy in the country. minister and its members are governor, reserve

bank of India; finance secretary and/or 10. Answer: D secretary, department of economic affairs; Explanation: The NITI Aayog ( National secretary, department of financial services; chief Institution For Transforming India) is a economic adviser, ministry of finance; chairman, policy think tank of the government of India, securities and exchange board of India; established with the aim to achieve sustainable chairman, insurance regulatory and development goals with cooperative development authority and chairman, pension federalism by fostering the involvement of state fund regulatory and development authority. It

www.laex.in Page No. 15 https://elearn.laex.in

www.laex.in

ECONOMY UPSC

Previous Year Questions

governments of India in the economic policy- Option (4) is not directly linked to inclusive making process using a bottom-up approach. governance. It was established in 2015, by Educational Objective: To have knowledge the nda government, to replace the planning about the terms development, inclusive growth, commission which followed a top-down model. governance, inclusive governance etc. The NITI Aayog council comprises all the state chief ministers, along with the chief 3. Poverty ministers of Delhi and Puducherry, the 3.1 Poverty Line lieutenant governor of Andaman and Nicobar,

and a vice-chairman nominated by the prime 1. In a given year in India, official poverty minister. In addition, temporary members are lines are higher in some States than in selected from leading universities and research others because institutions. These members include a chief a) Poverty rates vary from State to State executive officer, four ex-official members, and b) Price levels vary from State to State two part-time members. c) Gross State Product varies from State to Educational Objective: To have knowledge State about the transformation from Planning d) Quality of public distribution varies from commission to NITI AAYOG. State to State.

2.3 Miscellaneous 2. Persons below the poverty line in India are classified as such based on whether 11. Answer: D a) They are entitled to a minimum prescribed Explanation: Statements (a), (b)&(c) are true food basket which explain that planning process in the b) They get work for a prescribed minimum industrial sector in India has assumed a number of days in a year relatively less important position in the nineties c) They belong to agricultural labourer Educational Objective: To have knowledge household and the scheduled caste/tribe about the planned economy that India followed social group post-independence and how it eventually got d) Their daily wages fall below the prescribed transformed from period to period.

minimum wages 12. Answer: D 3.2 Measures to Eradicate Poverty Explanation: Inclusive governance, increasing

accountability and trust in government 3. How does the National Rural Livelihood institutions. When national, regional, or local Mission seek to improve livelihood governments make a concerted effort to solicit options of rural poor? input from citizens and provide services to all of 1) By setting up a large number of new their constituents equally, they sow the seeds of manufacturing industries and stability and growth. And reach out to the last agribusiness centres in rural areas corner of the country. www.laex.in Page No. 16 https://elearn.laex.in

www.laex.in

ECONOMY UPSC

Previous Year Questions

2) By strengthening 'self-help groups' and a) Under the Targeted Public Distribution providing skill development System, the families Below Poverty Line 3) By supplying seeds, fertilizers, diesel are provided 50 kg of food grains per pump-sets and micro-irrigation equipment month per family at subsidized price free of cost to farmers b) Under Annapurna Scheme, indigent senior Select the correct answer using the codes citizens of 65 years of age or above eligible given below: for National Old Age Pension but not a) 1 and 2 only getting pension can get 10 kg of food b) 2 only grains per person per month free of cost c) 1 and 3 only c) Ministry of Social Justice and Empower- d) 1, 2 and 3 ment has scheme in which indigent people

4. How do District Rural Development living in welfare institutions like Agencies (DRDAS) help in the reduction orphanages are given 15 kg of food grains of rural ? per person per month of BPL rates 1) DRDAS act as Panchyati Raj Institutions d) Ministry of Human Resource Development in certain Specified backward regions of gives financial support to Mid-day Meal the country. Scheme for the benefit of class I to V 2) DRDAS undertake area-specific scientific students in Government or Government study of the causes of poverty and aided School

malnutrition and detailed remedial 6. With reference to the government's measures. welfare schemes, consider the following 3) DRDAS secure inter-sectoral and inter- statements: departmental coordination and 1) Under the Antyodaya Anna Yojana, the cooperation for effective implementation of food grains are available to the poorest of anti-poverty programmes. the poor families at Rs. 2 per kg for wheat 4) DRDAS watch over and ensure effective and Rs. 3 per kg for rice. utilization of the funds intended for anti- 2) Under the National Old Age Pension poverty programmes. Scheme, the old and destitute are provided Which of the statements given above is/are Rs. 75 per month as Central Pension, in correct? addition to the amount provided by most a) 1, 2 and 3 only State Governments. b) 3 and 4 only 3) Government of India has allocated 25 kg c) 4 only food grains per Below Poverty Line family d) 1, 2, 3 and 4 per month, at less than half the economic cost. 5. Which one of the following statements Which of these statements are correct? is not correct? a) 1 & 2 Only

www.laex.in Page No. 17 https://elearn.laex.in

www.laex.in

ECONOMY UPSC

Previous Year Questions

b) 1 & 3 Only Rural 2400 2400 2090 calori kcal kcal kcal 48g c) 2 & 3 Only es protein d) All of the above 26g fat Urba 2100 2100 2155

n kcal kcal kcal 3. Key and Explanations calor 50g

protein 28g fat 3.1 Poverty Line Ies Rural ₹446.68 ₹972 per 1. Answer: B Rupe per capita capita Explanation: Poverty line estimation in India is es per per month month based on Basket of consumption goods (monthly Urba ₹578.80 ₹ 1181 spending on education, health, electricity and n per capita per Rupe per capita transport also) (as per Suresh Tendulkar es month per month Committee)  And the prices of these goods change from Educational Objective: To have brief idea on state to state. This is the reason for having how poverty lines are estimated in India and the high official poverty lines in some states various committees setup for poverty line where as less in some states. estimation.

Educational Objective: To have basic idea 3.2 Measures to Eradicate Poverty about the calculation of poverty lines in India 3. Answer: A

2. Answer A Explanation: The DAY-NRLM is essentially a Explanation: poverty relief programme of the Central Y.K.A Saxen Lakda Tendulka Rangaraj government. It was launched as ‘Aajeevika – Com lagh a wala r an mitte Com Com National Rural Livelihoods Mission (NRLM)’ by Comm Committ Committ es mitte mitte ittee ee ee the GOI’s Ministry of Rural Development in the e e Year 1979 1993 2005 2009 2013 year 2011. It was renamed as DAY-NRLM in Cons Planni Planni Planning Minist Governm 2015. titut ng ng Commissi ry of ent of ed by Comm Commi on Expert Rural India Features of NRLM: ission ssion Group develo Expert pment  Universal social mobilisation: A minimum Group of one lady member of a rural poor Base Calori Calori Basket of Calories d On es es in consump (it household (with particular emphasis on Povert tion included the marginal sections) is to be brought in y line goods nutrition, based (monthly fat and the network of an SHG. on spending other house on essential  Participatory Identification of Poor hold education non-food  Community Funds as Resources in per , health, items) capita electricity Perpetuity: this is to strengthen the consu and financial management capacity of the poor mption transport) expend  Financial inclusion iture.

www.laex.in Page No. 18 https://elearn.laex.in

www.laex.in

ECONOMY UPSC

Previous Year Questions

 setting up a large number of new This programme is controlled by the Ministry of manufacturing industries and agribusiness Consumer Affairs, Government of India. TPDS centres in rural areas emphasizes on the implementation and  strengthening 'self-help groups' and identification of the poor for proper arrangement providing skill development and delivery of food grains.  Livelihoods: the mission focuses on TPDS, the beneficiaries were divided into two promoting and stabilising the existing categories livelihood structures of the poor through 1. Households below the poverty line (BPL) its three pillars 2. Households above the poverty line (APL) Educational Objective: To know about various Currently, allocation is done as per the National schemes and policies of Government of India Food Security Act, 2013 (NFSA). Under this act, towards the development of the Rural poor. beneficiaries of the Public Distribution System are entitled to 5 kilograms per person per month 4. Answer: B of cereals. Now the number of kgs of rice allotted Explanation: District Rural Development to a family has been raised to 30 kgs but Agencies previously it was 2 Over the years the District Rural Development Educational Objective: To have brief idea of all Agencies have emerged as the principal organs the schemes and initiatives of the government of at the district level to oversing the India to the vulnerable sections of the society implementation of different poverty alleviation like women, old, SC/ST/OBC etc. programmes.

The primary objective of the DRDA 6. Answer: D Administration Scheme is to professionalise the Explanation: Antyodaya Anna Yojana is a DRD As so that they are able to effectively Government of India sponsored scheme to manage the poverty alleviation programmes of provide highly subsidized food to millions of the the Ministry of Rural Development and interact poorest families provide with opportunity to purposively with other agencies. purchase up to 35kilograms of rice and wheat at The DRDAs are expected to (effectively) a highly subsidized cost of ₹3 per kilogram of coordinate with the Panchayati Raj Institutions. rice and ₹2 per kilogram of wheat. They ensure effective utilization of the funds National Old Age Pension Scheme (NOAPS) is intended for anti-poverty programmes. a non-contributory old age pension scheme that Educational Objective: To know about various covers Indians who are 60 years and above and schemes and policies of Government of India live below the poverty line. Under the scheme towards the development of the Rural poor. the old and destitute are provided Rs. 75 per month as Central Pension, in addition to the 5. Answer : A amount provided by most State Governments. Explanation: Targeted Public Distribution System

www.laex.in Page No. 19 https://elearn.laex.in

www.laex.in

ECONOMY UPSC

Previous Year Questions

Now the number of kgs of rice allotted to a Which of the statements given above is/are family has been raised to 30 kgs but previously correct? it was 25 kgs a) 1 only Educational Objective: To have brief idea of all b) 2 only the schemes and initiatives of the government of c) Both 1 and 2 India to the vulnerable sections of the society d) Neither 1 nor 2

like women, old, SC/ST/OBC etc. 4. Consider the following statements,

The objectives of the National Renewal 4. Employment Fund set up in February 1992 were

4.1 Basic Concepts 1. To give training and counseling for

1. Disguised unemployment generally workers affected by retrenchment or VRS. means 2. Redeployment of workers a) Large number of people remain Which of these statements is/are correct? unemployed a) Neither 1 nor 2 b) Alternative employment is not available b) Both 1 and 2 c) Marginal productivity of labour is zero c) 1 only d) Productivity of workers is low d) 2 only

2. To obtain full benefits of demographic 5. With reference to 'Stand Up India dividend, what should India do? Scheme', which of the following a) Promoting skill development statements is/are correct? b) Introducing more social security schemes 1) Its purpose is to promote entrepreneurship c) Reducing infant mortality rate among SC/ST and women entrepreneurs. d) Privatization of higher education 2) It provides for refinance through SIDBI.

4.2 Government Initiatives Select the correct answer using the code

given below. 3. Consider the following statements in a) 1 only respect of the National Rural b) 2 only Employment Guarantee Act, 2005: c) Both 1 and 2 1) Under the provisions of the Act, 100 days d) Neither 1 nor 2

of employment in a year to every 6. Pradhan Mantri MUDRA Yojana is aimed household whose adult members volunteer at, to do unskilled manual work has become a a) Bringing the small entrepreneurs into fundamental right. formal financial system 2) Under the provisions of the Act, women b) Providing loans to poor farmers for are to get priority to the extent that one- cultivating particular crops half of persons who are given employment c) Providing pensions to old and destitute are women who have asked for work. persons www.laex.in Page No. 20 https://elearn.laex.in

www.laex.in

ECONOMY UPSC

Previous Year Questions

d) Funding the voluntary organizations d) 1, 2, 3 and 4

involved in the promotion of skill 10. The Swarna Jayanti Shahari Rozgar development and employment generation Yojana which came into operation from

7. Among the following who are eligible to 1-12-1997 aims to provide gainful benefit from the "Mahatma Gandhi employment to the urban unemployed National Rural Employment Guarantee or underemployed poor but does not Act"? include a) Adult members of only the scheduled caste a) Nehru Rozgar Yojana and scheduled tribe households b) Urban Basic Services Programme b) Adult members of below poverty line (BPL) c) Prime Minister's Integrated Urban Poverty households Eradication Programme c) Adult members of households of all d) Prime Minister's Rozgar Yojana

backward communities 11. The Employment Assurance Scheme d) Adult members of any household envisages financial assistance to rural areas for guaranteeing employment to 8. What is the name of the scheme which at least provides training and skills to women in a) 50% of the men and women seeking jobs traditional and non-traditional trades? in rural areas a) Kishori Shakti Yojna b) 50 per cent of the men seeking jobs in b) Rashtriya MahilaKosh rural areas c) Swayamsiddha c) one man and one woman in a rural family d) Swawlamban living below the poverty line

9. With reference to National Rural Health d) one person in a rural landless household Mission, which of the following are the living below the poverty line

jobs of 'ASHA', a trained community 12. With reference to Power Sector in India, health worker? consider the following statements: 1) Accompanying women to the health facility 1) Rural electrification has been treated as a for antenatal care checkup Basic Minimum Service under the Prime 2) Using pregnancy test kits for early Minister's Gramodaya Yojana detection of pregnancy 2) 100 percent Foreign Direct Investment in 3) Providing information on nutrition and power is allowed without upper limit immunization 3) The Union Ministry of Power has signed a 4) Conducting the delivery of baby Memoranda of Understanding with 14 Select the correct answer using the codes States given below: Which of these statements is/are correct? a) 1, 2 and 3 only a) 1 only b) 2 and 4 only b) 1 and 2 c) 1 and 3 only www.laex.in Page No. 21 https://elearn.laex.in

www.laex.in

ECONOMY UPSC

Previous Year Questions

c) 2 and 3 The Phillips curve shows the relationship d) 3 only between unemployment and inflation in an

economy. 13. Consider the following statements: Educational Objective: To know about various 1) The manages and types of Un-employments.

services Government of India Securities but 2. Answer : A not any State Government Securities. Explanation: Demographic dividend is 2) Treasury bills are issued by the Government economic growth brought on by a change in the of India and there are no treasury bills structure of a country’s population, usually a issued by the State Governments. result of a fall in fertility and mortality rates. 3) Treasury bills offer are issued at a discount The demographic dividend comes as there’s an from the par value. increase in the working population's Which of the statements given above is/are productivity, which boosts per capita income. correct? Demographic dividends can be found with a) 1 and 2 only savings, labor supply, human capital, and b) 3 only economic growth. c) 2 and 3 only Educational Objective: To know about d) 1, 2 and 3 Demographic Dividend and its components.

4. Key and Explanations 4.2 Government Initiatives

4.1 Basic Concepts 3. Answer: D

1. Answer : C Explanation: Right to work or livelihood is not a Explanation: Disguised unemployment- fundamental right, but is enshrined in the marginal productivity of labour is zero. Directive Principles in the Constitution of India. Example-Indian Agriculture is suffering from The MGNREG Act provides 33 percent provision disguised unemployment. for women and pays them equal wages. Other features of MGNREGA- Points to know:  Adult members of a rural household, willing to do unskilled manual work, may apply for registration in writing or orally to the local Gram Panchayat  The Gram Panchayat after due verification will issue a Job Card. The Job Card will bear the photograph of all adult members of the household willing to work under MGNREGA and is free of cost  The Job Card should be issued within 15 days of application. www.laex.in Page No. 22 https://elearn.laex.in

www.laex.in

ECONOMY UPSC

Previous Year Questions

 Employment will be given within 15 days of the workers who are likely to be affected by application for work, if it is not then daily technological up-gradation and unemployment allowance as per the Act, has modernisation in the Indian industry. to be paid liability of payment of The objectives were to be achieved as unemployment allowance is of the States. follows:  Wages are to be paid according to the By providing assistance to firms to cover the Minimum Wages Act 1948 for agricultural costs of retraining and redeployment of labourers in the State, unless the Centre employees arising as a result of modernisation notifies a wage rate which will not be less and technological upgradation of existing than Rs. 60/- per day. Equal wages will be capacities and from Industrial restructuring. provided to both men and women.  By providing funds for compensation to  Work site facilities such as crèche, drinking employees affected by restructuring or water, shade have to be provided. closure of industrial units, both in the  The shelf of projects for a village will be public and private sectors. recommended by the gram sabha and  By providing funds for employment approved by the Zilapanchayat. generation schemes in the organised and  At least 50% of works will be allotted to unorganised sectors in order to provide a Gram Panchayats for execution social safety net for labour.  The Central Government bears the 100  The government included two schemes percent wage cost of unskilled manual under this fund viz. Voluntary Retirement labour and 75 percent of the material cost Scheme (VRS) for central public sector including the wages of skilled and semi- undertakings; and re-training scheme for skilled workers rationalised workers in organised sector.  Social Audit has to be done by the Gram NRF was abolished in 2000. Sabha Educational Objective: To know about National  Grievance redressal mechanisms have to be Renewal Fund.

put in place for ensuring a responsive 5. Answer: C implementation process. Explanation: Stand-Up India Scheme is for Educational Objective: To know about financing SC/ST and/or Women Entrepreneur. MGNREGA, the world's largest public works The scheme is anchored by Department of programme. Financial Services (DFS), Ministry of

4. Answer : B Finance, Government of India. Explanation: National Renewal Fund was set  Composite loan (inclusive of term loan and up in February 1992 working capital) between 10 lakh and Objectives of National Renewal Fund upto 100 lakh.  The main objective of the National Renewal  The rate of interest would be lowest Fund was to provide a social safety net to applicable rate of the bank for that category

www.laex.in Page No. 23 https://elearn.laex.in

www.laex.in

ECONOMY UPSC

Previous Year Questions

(rating category) not to exceed (base rate (MUDRA) loans under Pradhan Mantri Mudra (MCLR) + 3%+ tenor premium. Yojana (PMMY).  The loan is repayable in 7 years with a Under the aegis of PradhanMantri MUDRA maximum moratorium period of 18 months. Yojana, MUDRA has already created the  Refinance window through Small Industries following products / schemes. Development Bank of India (SIDBI) with an Shishu: covering loans upto 50,000/- initial amount of Rs.10,000 crore. Kishor: covering loans above 50,000/- and upto  Creation of a corpus of Rs. 5,000 crore for 5 lakh credit guarantee through NCGTC. Tarun: covering loans above 5 lakh and upto 10  Applicants of the loan under this scheme lakh. should be at least 18 years of age. Educational Objective: To know about MUDRA  At least 51 percent of the shareholding and Yojana

control should be owned by the borrower of 7. Answer : D the loan under this scheme. Explanation: Eligibility for MGNREGA Educational Objective: To know about Stand Adult members of a rural household, willing to Up India Scheme do unskilled manual work, may apply for registration in writing or orally to the local Gram 6. Answer : A Panchayat. Explanation: Pradhan Mantri Mudra Yojana “Household” means the members of a family (PMMY) is a flagship scheme of Government of related to each other by blood, marriage or India to “fund the unfunded” by bringing such adoption and normally residing together and enterprises to the formal financial system and sharing meals or holding a common ration card. extending affordable credit to them. Job Card is a key document that records workers’  It enables a small borrower to borrow from all entitlements under MGNREGA. It legally empowers Public Sector Banks such as PSU Banks, the registered households to apply for work, Regional Rural Banks and Cooperative Banks, ensures transparency and protects workers Private Sector Banks, Foreign Banks, Micro against fraud. Finance Educational Objective: To know about  Institutions (MFI) and Non Banking Finance MGNREGA, the world's largest public works Companies (NBFC) for loans up to Rs.10 lakhs programme.

for non-farm income generating activities. 8. Answer : D  Any Indian Citizen who has a business plan for Explanation: Swavlamban' is the name of the a non-farm sector income generating activity scheme which provides training and skills to such as manufacturing, processing, trading or women in traditional and non-traditional trades. service sector and whose credit need is less Swawlamban (NORAD) Scheme is being than Rs 10 lakh can approach either a Bank, implemented by the Department of Women and MFI, or NBFC for availing of Micro Units Child Development with partial assistance from Development & Refinance Agency Ltd. Norway since 1982. www.laex.in Page No. 24 https://elearn.laex.in

www.laex.in

ECONOMY UPSC

Previous Year Questions

Kishori Shakti Yojana- Kishori Shakti Yojana marginalized promoting healthy behaviours (KSY) seeks to empower adolescent girls, so as to mobilizing for collective action for better health enable them to take charge of their lives. It is outcomes and meeting curative care needs as viewed as a holistic initiative for the development appropriate to the organization of service of adolescent girls. The programme through its delivery in that area and compatible with her interventions aims at bringing about a difference training and skills. in the lives of the adolescent girls. It seeks to Conducting the delivery of baby is done by provide them with an opportunity to realize their Medical Officer full potential. Educational Objective: To know about ASHA.

Rashtriya MahilaKosh (RMK), established in 1993 is a national level organization as 10. Answer : D an autonomous body under the aegis Explanation: Swarna Jayanti Shahari of the Ministry of Women &Child Development, Rozgar Yojana (SJSRY) is a Centrally Sponsored for socio-economic empowerment of women. Scheme strives to provide gainful employment to The operating model currently followed by RMK the urban unemployed and underemployed poor, is that of a facilitating through encouraging the setting up of self- agency wherein RMK provides loans to NGO- employment ventures or provision of wage MFIs termed as Intermediary Organizations (IMO) employment. which on-lend to Self Help Groups (SHGs) of The Swarna Jayanti Shahari Rozgar Yojana women. (SJSRY) was launched on 01.12.1997 after Swayamsiddha- It is a flagship programme of the subsuming the earlier three schemes for urban Ministry of Women and Child Development (WCD), poverty alleviation, viz., Nehru Rozgar Yojana Government of India. It is an integrated women (NRY), Urban Basic Services for the Poor (UBSP) empowerment programme (IWEP) initiated in 2001 and the Prime Minister’s Integrated Urban Poverty by merging Mahila Samriddhi Yojana and Eradication Programme (PMIUPEP). The scheme recasting Indira Mahila Yojana (IMY was the first SJSRY has been restructured renaming it as Self Help Group based women’s empowerment National Urban Livelihood Mission (NULM) since programme of Ministry of WCD launched in 1995- the year 2014-15 96) and including other sectoral programmes Educational objective: To learn about Swarna meant for women empowerment. Jayanti Shahari Rozgar Yojana.

Educational Objective: To know about 11. Answer : C schemes related to Women Empowerment. Explanation: Employment Assurance Scheme was launched on 2nd October, 1993 for 9. Answer: A implementation in 1778 identified backward Explanation: The role of an ASHA is that of a blocks of different States. The block selected community level care provider. were in the drought prone areas, desert areas, This includes a mix of tasks: tribal areas and hilly areas. Facilitating access to health care services Objective As its name suggests, the primary building awareness about health care objective of the EAS is to provide gainful entitlements especially amongst the poor and www.laex.in Page No. 25 https://elearn.laex.in

www.laex.in

ECONOMY UPSC

Previous Year Questions

employment during the lean agricultural season 13. Answer : C in manual work to all able bodied adults in rural Explanation: The Reserve Bank of India areas who are in need and desirous of work, but manages public debt and issues Indian currency cannot find it. denominated loans on behalf of the central and Status The programme is implemented as a the state governments under the powers derived Centrally Sponsored Scheme on cost sharing from the Reserve Bank of India Act. The RBI is basis between the Centre and the States in the the debt manager for both the Central ratio of 75:25. In the case of Union Territories, Government and the State Governments. RBI the Centre provides entire funds under the manages the debt of state governments on the Scheme. basis of separate agreements. Hence statement Target Group The EAS would be open to all 1 is not correct. adult rural poor who are in need of wage employment. A maximum of two adults per Treasury Bills are short term (up to one year) family (the generally expected number of adults borrowing instruments of the Government of in a family) would be provided wage India or by a central authority of any country employment, when there is demand during lean which enable investors to park their short term agricultural season, subject to availability of surplus funds while reducing their market risk. funds. Hence option c is correct. Individuals, Firms, Trusts, Institutions and Educational objective: To learn about the banks can purchase T-Bills. Treasury bills or T- Employment Assurance Scheme and its bills, which are money market instruments, are features. short term debt instruments issued by the Government of India and are presently issued in 12. Answer : B three tenors, namely, 91 days, 182 days and Explanation: PMGY was implemented from 364 days. 2000 - 2001 with the objective of achieving The State governments do not issue any sustainable human development at the village treasury bills. Interest on the treasury bills is level. It is a component of Basic Minimum determined by market forces. Services (BMS) which was started in 1996 – 97. Hence statement 1 is correct. They are auctioned by the Reserve Bank of India (RBI) at regular intervals and issued at a Up to 100% FDI allowed in respect of projects discount to face value. Hence statement 2 and 3 relating to electricity generation, transmission are correct. and distribution, other than atomic reactor power plants. There is no limit on the project Educational objective: To learn about Reserve cost and quantum of foreign direct investment. bank of India and treasury bills. Hence statement 2 is correct. Educational objective: To learn about Prime Minister's Gramodaya Yojana and Foreign Direct Investment. www.laex.in Page No. 26 https://elearn.laex.in

www.laex.in

ECONOMY UPSC

Previous Year Questions

www.laex.in Page No. 27 https://elearn.laex.in

www.laex.in

ECONOMY UPSC

Previous Year Questions

5. Monetary Policy a) 2, and 3 only b) 1 and 2 only 5.1 RBI c) 1 and 3 only 1. Consider the following statements : d) 1, 2 and 3 The Reserve Bank of India's recent

directives relating to "Storage of Payment 3. Consider the following statements: System Data', popularly known as data 1) Reserve Bank of India was nationalized on diktat, command the payment system 26 January, 1950 providers that, 2) The borrowing programme of the 1) They shall ensure that entire data relating Government of India is handled by the to payment systems operated by them are Department of Expenditure, Ministry of stored in a system only in India Finance 2) They shall ensure that the systems are Which of the statements given above owned and operated by public sector is/are correct? enterprises a) 1 only 3) They shall submit the consolidated system b) 2 only audit report to the Comptroller and Auditor c) Both 1 and 2 General of India by the end of the calendar d) Neither 1 nor 2

year Which of the statements given above is/are 4. The National Housing Bank was set up in correct? India as a wholly- owned subsidiary of which one of the following? a) 1 only a) State Bank of India b) 1 and 2 only b) Reserve Bank of India c) 3 only c) ICICI Bank d) 1, 2 and 3

d) Life Insurance Corporation of India

2. The Reserve Bank of India (RBI) acts as 5. The accounting year of the Reserve Bank a bankers' bank. This would imply which of India is of the following? a) April-March 1) Other banks retain their deposits with the RBI. b) July-June 2) The RBI lends funds to the commercial c) October-September banks in times of need. d) January-December

3) The RBI advises the commercial banks on 6. The Reserve Bank of India regulates the monetary matters. commercial banks in matters of Select the correct answer using the codes 1) Liquidity of assets given below: www.laex.in Page No. 28 https://elearn.laex.in

www.laex.in

ECONOMY UPSC

Previous Year Questions

2) Branch expansion a) 1 only 3) Merger of banks b) 1 and 2 only 4) Winding-up of banks c) 3 only Select the correct answer using the codes d) 2 and 3 only

given below: 9. With reference to Indian economy, a) 1 and 4 only consider the following b) 1, 2 and 3 only c) 2, 3 and 4 only 1) Bank rate d) 1, 2, 3 and 4 2) Open market operations

7. Consider the following statements: 3) Public revenue 1) Global Trust Bank has been amalgamated 4) Public debt with the Punjab National Bank. Which of the above is/are component/ 2) The second report of the Kelkar Committee components of Monetary Policy? dealing with direct and indirect taxes has a) 1 only maintained its original recommendations b) 2, 3 and 4 including the abolition of exemptions c) 1 and 2 relating to housing loans. d) 1, 3 and 4 Which of the statements given above is/are

correct? 10. When the Reserve Bank of India reduces a) 1 only the Statutory Liquidity Ratio by 50 basis b) 2 only points, which of the following is likely to c) Both 1 and 2 happen? d) Neither 1 nor 2 a) India's GDP growth rate increases

drastically 5.2 Credit & Monetary Policy b) Foreign Institutional Investors may bring 8. Which of the following statements is/are more capital into our country correct regarding the Monetary Policy c) Scheduled Commercial Banks may cut their Committee (MPC) ? lending rates 1) It decides the RBI's benchmark interest

rates, d) It may drastically reduce the liquidity to the banking system 2) It is a 12-member body including the

Governor of RBI and is reconstituted every 11. The terms 'Marginal Standing Facility year. Rate' and 'Net Demand and Time 3) It functions under the chairmanship of the Liabilities', sometimes appearing in Union Finance Minister. news, are used in relation to Select the correct answer using the code a) Banking operations given below: b) Cmmunication networking www.laex.in Page No. 29 https://elearn.laex.in

www.laex.in

ECONOMY UPSC

Previous Year Questions

c) Military strategies Government/RBI takes to stop the slide d) Supply and demand of agricultural of ? products a) Curbing imports of non-essential goods and promoting exports 12. Consider the following statements: b) Encouraging Indian borrowers to issue 1) The repo rate is the rate at which other rupee denominated Masala Bonds banks borrow from the Reserve Bank of c) Easing conditions relating to external India. commercial borrowing 2) A value of 1 for Gini Coefficient in a country d) Following an expansionary monetary policy implies that there is perfectly equal income

for everyone in its population. 16. In the context of Indian economy, which Which of the statements given above is/are of the following is / are the correct? purpose/purposes of Statutory Reserve a) 1 only Requirements'? b) 2 only 1) To enable the Central Bank to control the c) Both 1 and 2 amount of advances the banks can create d) Neither 1 nor 2 2) To make the people's deposits with banks

safe and liquid 13. The banks are required to maintain a 3) To prevent the commercial banks from certain ratio between their cash in hand making excessive profits and total assets. This is called 4) To force the banks to have sufficient vault- a) SBR (Statutory Bank Ratio) cash to meet their day-to-day requirements b) SLR (Statutory Liquid Ratio) Select the correct answer using the code c) CBR (Central Bank Reserve) given below. d) CLR (Central Liquid Reserve) a) 1 only

14. In the context of Indian economy, 'Open b) 1 and 2 only Market Operations' refers to c) 2 and 3 only a) borrowing by scheduled banks from the RBI d) 1, 2, 3 and 4 b) lending by commercial banks to industry 17. An increase in the Bank Rate generally and trade indicates that the c) purchase and sale of government securities a) Market rate of interest is likely to fall by the RBI b) Central Bank is no longer making loans to d) None of the above commercial banks

15. Which one of the following is not the c) Central Bank is following an easy money most likely measure the policy

www.laex.in Page No. 30 https://elearn.laex.in

www.laex.in

ECONOMY UPSC

Previous Year Questions

d) Central Bank is following a tight money d) Management of concerned bank policy 22. With reference to the governance of

18. The lowering of Bank Rate by the public sector , consider Reserve Bank of India leads to the following statements: a) More liquidity in the market 1) Capital infusion into public sector banks by b) Less liquidity in the market the Government of India has steadily c) No change in the liquidity in the market increased in the last decade. d) Mobilization of more deposits by commercial 2) To put the public sector banks in order, the banks merger of associate banks with the parent

State Bank of India has been affected. 19. When the Reserve Bank of India Which of the statements given above is/are announces an increase of the Cash correct ? Reserve Ratio, what does it mean? a) 1 only a) The commercial banks will have less money b) 2 only to lend c) Both 1 and 2 b) The Reserve Bank of India will have less money to lend d) Neither 1 nor 2

c) The Union Government will have less money 23. Which of the following terms indicates a to lend mechanism used by commercial banks d) The commercial banks will have more for providing credit to the government? money to lend a) Cash Credit Ratio

5.3 Banking Sector b) Debt Service Obligation

c) Liquidity Adjustment Facilities 20. Which of the following is not included in d) Statutory Liquidity Ratio the assets of a commercial bank in

India? 24. With reference to the Non-banking a) Advances Financial Companies (NBFCs) in India, consider the following statements: b) Investments 1) They cannot engage in the acquisition of c) Deposits securities issued by the government d) Money at call and short notice

2) They cannot accept demand deposits like 21. The Chairmen of public sector banks are Savings Account selected by the Which of the statements given above is/are a) Banks Board Bureau correct? b) Reserve Bank of India a) 1 only c) Union Ministry of Finance b) 2 only

www.laex.in Page No. 31 https://elearn.laex.in

www.laex.in

ECONOMY UPSC

Previous Year Questions

c) Both 1 and 2 29. Consider the following statements: d) Neither 1 nor 2 The functions of commercial banks in India include 25. The basic aim of Lead Bank Scheme is that 1) Purchase and sale of shares and securities on behalf of customers. a) Big banks should try to open offices in each district 2) Acting as executors and trustees of wills. b) There should be stiff competition among the Which of the statements given above is/ are various nationalized banks correct? c) Individual banks should adopt particular a) 1 only districts for intensive development b) 2 only d) All the banks should make intensive efforts c) Both 1 and 2 to mobilize deposits d) Neither 1 nor 2

26. The Service Area Approach was 30. In India, the first bank of limited liability implemented under the purview of managed by Indians and founded in 1881 a) Integrated Rural Development Programme was b) Lead Bank Scheme a) Hindustan Commercial Bank c) Mahatma Gandhi National Rural b) Oudh Commercial Bank Employment Guarantee Scheme c) Punjab National Bank d) National Skill Development Mission d) Punjab and Sind Bank

27. Which of the following committees 5.4 Lending Rates examined and suggested Financial Sector 31. What is/are the purpose/purposes of the reforms? 'Marginal Cost of Funds based Lending a) Abid Hussain Committee Rate (MCLR)' announced by RBI? b) Bhagwati Committee 1) These guidelines help improve the c) Chelliah Committee transparency in the methodology followed d) by banks for determining the interest rates

on advances. 28. In India, the interest rate on savings 2) These guidelines help ensure availability of accounts in all the nationalized bank credit at interest rates which are fair commercial banks is fixed by to the borrowers as well as the banks. a) Union Ministry of Finance Select the correct answer using the code b) Union Finance Commission given below. c) Indian Banks' Association a) 1 only d) None of the above b) 2 only

www.laex.in Page No. 32 https://elearn.laex.in

www.laex.in

ECONOMY UPSC

Previous Year Questions

c) Both 1 and 2 b) 2 only d) Neither 1 nor 2 c) 1 and 2 only

d) Neither 1 nor 2 5.5 Finacial Inclusion

34. Priority Sector Lending by banks in India 32. The establishment of 'Payment Banks' is constitutes the lending to being allowed in India to promote financial inclusion. Which of the a) Agriculture following statements is/are correct in b) Micro and small enterprises this context? c) Weaker sections 1) Mobile telephone companies and d) All of the above supermarket chains that are owned and 35. Which of the following grants/grant controlled by residents are eligible to be direct credit assistance to rural promoters of Payment Banks. households? 2) Payment Banks can issue both credit cards 1) Regional Rural Banks and debit cards. 2) National Bank for Agriculture and Rural 3) Payment Banks cannot undertake lending Development activities. 3) Land Development Banks Select the correct answer using the code given below. Select the correct answer using the codes given below. a) 1 and 2 only a) 1 and 2 only b) 1 and 3 only b) 2 only c) 2 only c) 1 only d) 1, 2 and 3

d) 1, 2 and 3

33. What is/are the facility/facilities the beneficiaries can get from the services of 36. With reference to India, consider the Business Correspondent (Bank Saathi) in following: branchless areas? 1) Nationalization of Banks 1) It enables the beneficiaries to draw their 2) Formation of Regional Rural Banks subsidies and social security benefits in 3) Adoption of villages by Bank Branches their villages. Which of the above can be considered as 2) It enables the beneficiaries in the rural steps taken to achieve the "financial areas to make deposits and withdrawals. inclusion" in India? Select the correct answer using the code a) 1 and 2 only given below. b) 2 and 3 only a) 1 only c) 3 only

www.laex.in Page No. 33 https://elearn.laex.in

www.laex.in

ECONOMY UPSC

Previous Year Questions

d) 1, 2 and 3 c) Promoting financial inclusion in the country d) Providing financial help to the marginalized 37. With reference to the schemes launched communities by the Union Government, consider the

following statements: 40. Regarding 'Atal Pension Yojana', which of 1) Ministry of Health and Family Welfare the following statements is/are correct? launched the Rashtriya Swasthya Bima 1) It is a minimum guaranteed pension Yojana. scheme mainly targeted at unorganized 2) Ministry of Textiles launched the Rajiv sector workers. Gandhi Shilpi Swasthya Bima Yojana. 2) Only one member of a family can join the Which of the statements given above is/are scheme. incorrect? 3) Same amount of pension is guaranteed for a) 1 only the spouse for life after subscriber's death. b) 2 only Select the correct answer using the code c) Both 1 and 2 given below. d) Neither 1 nor 2 a) 1 only

b) 2 and 3 only 38. Which of the following can aid in c) 1 and 3 only furthering the Government's objective of d) 1, 2 and 3 inclusive growth?

1) Promoting Self-Help Groups 41. Consider the following: 2) Promoting Micro, Small and Medium 1) Hotels and restaurants Enterprises 2) Motor transport undertakings 3) Implementing the Right to Education Act 3) Newspaper establishments Select the correct answer using the codes 4) Private medical institutions given below: The employees of which of the above can a) 1 only have the "Social Security' coverage under b) 1 and 2 only Employees' State Insurance Scheme? c) 2 and 3 only a) 1, 2 and 3 only d) 1, 2 and 3 b) 4 only c) 1, 3 and 4 only 39. 'Pradhan Mantri Jan-Dhan Yojana' has

been launched for d) 1, 2, 3 and 4 a) Providing housing loan to poor people at cheaper interest rates b) Promoting women's Self-Help Groups in backward areas www.laex.in Page No. 34 https://elearn.laex.in

www.laex.in

ECONOMY UPSC

Previous Year Questions

5.6 Payment Systems & NPCI 45. Which one of the following statements

42. Consider the following statements: correctly describes the meaning of legal tender money? 1) National Payments Corporation of India (NPCI) helps in promoting the financial a) The money which is tendered in courts of inclusion in the country. law to defray the fee of legal cases 2) NPCI has launched RuPay, a card payment b) The money which a creditor is under scheme. compulsion to accept in settlement of his claims Which of the statements given above is/are c) The bank money in the form of cheques, correct? drafts, bills of exchange, etc. a) 1 only d) The metallic money in circulation in a b) 2 only country

c) Both 1 and 2 d) Neither 1 nor 2 5.7 NPAs & Stressed Assets

43. Which one of the following best describes 46. What was the purpose of Inter-Creditor the term "Merchant Discount Rate" Agreement signed by Indian banks and sometimes seen in news? financial institutions recently? a) The incentive given by a bank to a merchant a) To lessen the Government of India's for accepting payments through debit cards perennial burden of fiscal deficit and pertaining to that bank. current account deficit b) The amount paid back by banks to their b) To support the infrastructure projects of customers when they use debit cards for Central and State Governments financial transactions for purchasing goods c) To act as independent regulator in case of or services. applications for loans of Rs. 50 crore or c) The charge to a merchant by a bank for more accepting payments from his customers d) To aim at faster resolution of stressed through the bank's debit cards. assets of Rs. 50 crore or more which are d) The incentive given by the Government, to under consortium lending

merchants for promoting digital payments 47. Which of the following statements best by their customers through Point of sale describes the term 'Scheme for (PoS) machines and debit cards. Sustainable Structuring of Stressed

44. Which one of the following links all the Assets (S4A)', recently seen in the news? ATMS in India? a) It is a procedure for considering ecological a) Indian Banks' Association costs of developmental schemes formulated b) National Securities Depository Limited by the Government. c) National Payments Corporation of India d) Reserve Bank of India www.laex.in Page No. 35 https://elearn.laex.in

www.laex.in

ECONOMY UPSC

Previous Year Questions

b) It is a scheme of RBI for reworking the d) Transfer technology from developed financial structure of big corporate entities countries to poor countries to enable them facing genuine difficulties. to replace the use of chlorofluorocarbons in c) It is a disinvestment plan of the refrigeration with harmless chemicals Government regarding Central Public Sector 50. Basel II relates to which one of the Undertakings. following d) It is an important provision in The a) International standards for safety in civil Insolvency and Bankruptcy Code' recently aviation implemented by the Government.

b) Measures against cyber crimes 5.8 Basel Norms & Capital Adequacy c) Measures against drug abuse by Ratio sportspersons 48. Consider the following statements: d) International standards for measuring the adequacy of a bank's capital 1) Capital Adequacy Ratio (CAR) is the amount

that banks have to maintain in the form of 5.9 Money Supply & Inflation their own funds to offset any loss that 51. Which one of the following is likely to be banks incur if the account-holders fail to the most inflationary in its effect? repay dues. a) Repayment of public debt 2) CAR is decided by each individual bank. b) Borrowing from the public to finance a Which of the statements given above is/are budget deficit correct? c) Borrowing from banks to finance a budget a) 1 only deficit b) 2 only d) Creating new money to finance a budget c) Both 1 and 2 deficit d) Neither 1 nor 2

52. Consider the following statements: 49. 'Basel III Accord' or simply 'Basel III', often seen in the news, seeks to 1) Inflation benefits the debtors. 2) Inflation benefits the bondholders. a) Develop national strategies for the conservation and sustainable use of Which of the statements given above is/are biological diversity correct? b) Improve banking sector's ability to deal with a) 1 only financial and economic stress and improve b) 2 only risk management c) Both 1 and 2 c) Reduce the greenhouse gas emissions but d) Neither 1 nor 2

places a heavier burden on developed 53. A rapid increase in the rate of inflation is countries sometimes attributed to the "base effect". What is "base effect"?

www.laex.in Page No. 36 https://elearn.laex.in

www.laex.in

ECONOMY UPSC

Previous Year Questions

a) It is the impact of drastic deficiency in d) gold price has gone up 3-3 times supply due to failure of crops 57. Consider the following: b) It is the impact of the surge in demand due 1) Currency with the public to rapid economic growth 2) Demand deposits with banks c) It is the impact of the price levels of 3) Time deposits with banks previous year on the calculation of inflation Which of these are included in Broad Money rate (M3) in India? d) None of the statements (a), (b) and (c) given above is correct in this context a) 1 and 2 b) 1 and 3 54. A rise in general level of prices may be caused by c) 2 and 3 1) An increase in the money supply d) 1, 2 and 3

2) An decrease in the aggregate level of output 58. With reference to the Wholesale Price 3) An increase in the effective demand Index (WPI), consider the following Select the correct answer using the codes statements: given below 1) The new WPI series with base 1993-94=100 a) 1 only became effective from April 1998. b) 1 and 2 Only 2) In the new WPI series, the weight for c) 2 and 3 only primary articles has gone down by 10 d) 1, 2 and 3 percentage points. 55. Supply of money remaining the same 3) The weight for electricity has increased in when there is an increase in demand for the new WPI series. money, there will be Which of these statements are correct? a) A fall in the level of prices a) 1, 2 and 3 b) An increasé in the rate of interest b) 2 and 3 c) A decrease in the rate of interest c) 1 and 3 d) An increase in the level of income and d) 1 and 2 employment

59. The new series of Wholesale Price Index 56. The current price index (base 1960) is (WPI) released by the Government of nearly 330. This means that India is with reference to the base prices a) all items cost 3-3 times more than what of they did in 1960. a) 1981-82 b) the prices of certain selected items have b) 1990-91 gone upto 3-3 times c) 1993-94 c) weighted mean of prices of certain items d) 1994-95 has increased 3-3 times

www.laex.in Page No. 37 https://elearn.laex.in

www.laex.in

ECONOMY UPSC

Previous Year Questions

60. Consider the following actions by the Select the correct answer using the codes Government: given below: 1) Cutting the tax rates a) 1 only 2) Increasing the government spending b) 2 and 4 only 3) Abolishing the subsidies c) 1 and 3 In the context of economic recession, which d) 2, 3 and 4

of the above actions can be considered a part 63. With reference to India, consider the of the "“fiscal stimulus" package? following statements: a) 1 and 2 only 1) The Wholesale Price Index (WPI) in India is b) 2 only available on a monthly basis only c) 1 and 3 only 2) As compared to Consumer Price Index for d) 1, 2 and 3 Industrial Workers (CPI(IW)), the WPI gives

61. With reference to , less weight to food articles which of the following statements is Which of the statements given above is/ are correct? correct? a) Controlling the inflation in India is the a) 1 only responsibility of the Government of India b) b) 2 only only c) Both 1 and 2 b) The Reserve Bank of India has no role in d) Neither1 nor 2 controlling the inflation 64. Which of the following brings out the c) Decreased money circulation helps in 'Consumer Price Index Number for controlling the inflation Industrial Workers ? d) Increased money circulation helps in a) The Reserve Bank of India controlling the inflation b) The Department of Economic Affairs

62. Which of the following measures would c) The Labour Bureau result in an increase in the money supply d) The Department of Personnel and Training

in the economy? 5.10 Miscellaneous 1) Purchase of government securities from the 65. The money multiplier in an economy public by the Central Bank increase one of the following? 2) Deposit of currency in commercial banks by a) Increase in the cash reserve ratio the public b) Increase in the banking habit of the 3) Borrowing by the government from the population Central Bank c) Increase in the statutory liquidity ratio 4) Sale of government securities to the public d) Increase in the population of the country by the Central Bank

www.laex.in Page No. 38 https://elearn.laex.in

www.laex.in

ECONOMY UPSC

Previous Year Questions

66. What is/are the purpose/purposes of 69. With reference to the institution of Government's "Sovereign Gold Bond Banking Ombudsman in India, which one Scheme' and 'Gold Monetization of the statements is not correct? Scheme'? a) The Banking Ombudsman is appointed by 1) To bring the idle gold lying with Indian the Reserve Bank of India households into the economy b) The Banking Ombudsman can consider 2) To promote FDI in the gold and jewellery complaints from Non-Resident Indians sector having accounts in India 3) To reduce India's dependence on gold c) The orders passed by the Banking imports Ombudsman are final and binding on the Select the correct answer using the code parties concerned given below. d) The service provided by the Banking a) 1 only Ombudsman is free of any fee b) 2 and 3 only 70. The average rate of domestic savings c) 1 and 3 only (gross) for the Indian economy is d) 1, 2 and 3 currently estimated to be in the range of

67. If the interest rate is decreased in an (2000) economy, it will a) 15 to 20 per cent a) decrease the consumption expenditure in b) 20 to 25 per cent the economy c) 15 to 30 per cent b) increase the tax collection of the d) 30 to 35 per cent

Government c) increase the investment expenditure in the 71. Consider the following financial economy institutions of India: d) increase the total savings in the economy 1) Industrial Finance Corporation of India

(IFCI). 68. Consider the following liquid assets : 2) Industrial Credit and Investment 1) Demand deposits with the banks Corporation of India (ICICI). 2) Time deposits with the banks 3) Industrial Development Bank of India 3) Savings deposits with the banks (IDBI). 4) Currency 4) National Bank for Agriculture and Rural The correct sequence of these assets in the Development (NABARD) decreasing order of liquidity is The correct chronological sequence of the a) 1-4-3-2 establishment of these institutions is b) 4-3-2-1 c) 2-3-1-4 a) 1, 2, 3, 4 d) 4-1-3-2 b) 2, 3, 4, 1

www.laex.in Page No. 39 https://elearn.laex.in

www.laex.in

ECONOMY UPSC

Previous Year Questions

c) 3, 4, 1, 2 a) 1 only d) 4, 1, 2, 3 b) 1 and 4 only c) 2 and 3 only 72. Consider the following statements: d) 1, 2, 3 and 4 1) The National Housing Bank, the apex institution of housing finance in India, was 75. Why is the offering of "teaser loans" by set up as a wholly- owned subsidiary of the commercial banks a cause of economic Reserve Bank of India concern? 2) The Small Industries Development Bank of 1) The teaser loans are considered to be an India was established as a wholly-owned aspect of sub-prime lending and banks may subsidiary of the Industrial Development be exposed to the risk of defaulters in Bank of India future. 2) In India, the teaser loans are mostly given Which of the statements given above is/are to inexperienced entrepreneurs to set up correct? manufacturing or export units. a) 1 only Which of the statements given above is/are b) 2 only correct? c) Both 1 and 2 a) 1 only d) Neither 1 nor 2 b) 2 only

73. In the context of independent India's c) Both 1 and 2 economy, which one of the following was d) Neither 1 nor 2 the earliest event to take place? a) Nationalization of Insurance companies 5. Key and Explanations b) Nationalization of State Bank of India 5.1 RBI c) Enactment of Banking Regulation Act 1. Answer : A d) Introduction of First Five-Year Plan Explanation : On April 6, 2018 RBI released

74. Microfinance is the provision of financial guidelines on ‘Storage of Payment System Data’ services to people of low-income groups. advising all system providers to ensure that, This includes both the consumers and within a period of six months, the entire data the self-employed. The service/services relating to payment systems operated by them rendered under microfinance is/are: to be stored in India. 1) Credit Facilities The directions are applicable to all Payment 2) Saving Facilities System providers authorised / approved by the 3) Fund Transfer Facilities Reserve Bank of India (RBI) to set up and 4) Insurance Facilities operate a payment system in India under the Payment and Settlement Systems Act, 2007. Select the correct answer using the codes given below:

www.laex.in Page No. 40 https://elearn.laex.in

www.laex.in

ECONOMY UPSC

Previous Year Questions

The guidelines doesn’t require the payment 3. Answer : D systems to be regulated only by the public Explanation : sector enterprises . The RBI was Nationalised on 1st January, 1949. The data should include end-to-end transaction The borrowing programme of the Government of details and information pertaining to payment India is handled by the Public Debt or settlement transaction that is gathered / Management Cell (PDMC) as interim measure transmitted / processed as part of a payment under the Dept. of Economic Affairs , Ministry of message / instruction. Finance. There is a proposal to set up an For cross border transaction data, consisting of independent debt management body – Public a foreign component and a domestic component, Debt Management Agency(PDMA). a copy of the domestic component may also be Educational Objective: Government Debt stored abroad, if required. management(Government Budget and The System Audit Report (SAR), from a CERT-In financing).

empanelled Auditor, should inter-alia include 4. Answer : B Data Storage, Maintenance of Database, Data Explanation : NHB is an All India Financial Backup Restoration, Data Security, etc. Institution (AIFl), set up in 1988, under the Educational Objective: Important National Housing Bank Act, 1987 India as a developments in the Indian Banking Sector. wholly- owned subsidiary of the RBI.

2. Answer : D It is an apex agency established to operate as a Explanation : The Reserve Bank of India (RBI) principal agency to promote housing finance acts as a bankers' bank implies : institutions both at local and regional levels and Other banks retain their deposits with the RBI. to provide financial and other support incidental The RBI lends funds to the commercial banks in to such institutions and for matters connected times of need. The RBI advises the commercial therewith. banks on monetary matters. The Reserve Bank of India (RBI) has sold its Other Functions of RBI Include : entire stakes in National Housing Bank (NHB) to  Issuer of Bank Notes the Central Government for Rs 1,450 crore in  Banker to Government April ,2019. As a result NHB is wholly owned by  Custodian of Cash Reserves of the Central Government. Commercial Banks Educational Objective: Awareness about the All  Custodian of Country’s Foreign Currency India Financial Institutions.

Reserves 5. Answer : A  Central Clearance and Accounts Explanation : The accounting year of the Settlement Reserve Bank of India is April-March, which is Educational Objective: Understand the in sync with the fiscal year in India. Functions of the Central Bank. Educational Objective: Facts about RBI’s functioning.

www.laex.in Page No. 41 https://elearn.laex.in

www.laex.in

ECONOMY UPSC

Previous Year Questions

6. Answer : D 5.2 Credit & Monetary Policy

Explanation : RBI is called the banker's bank 8. Answer : A and regulates the banking sector in India. By Explanation : The RBI has a government- using mechanisms like CRR, SLR etc, it keeps a constituted Monetary Policy Committee (MPC) check on liquidity of assets of the banks. Moreover, which is tasked with framing monetary policy RBI also sets rules and regulations concerning using tools like the repo rate, reverse repo rate, merger of banks, their winding-up operations and bank rate, cash reserve ratio (CRR). branch expansion. Amalgamation /Merger of two It has been instituted by the Central bank require RBI’s approval. Government of India under Section 45ZB of the Educational objective: To learn about Reserve RBI Act 1934 and as per recommendations of bank of India’s operations. Dr. Urjit Patel committee. Composition of MPC: 7. Answer : B The committee will have six members. Of the six Explanation : Global Trust Bank has been members, the government will nominate three. amalgamated with the Oriental Bank of The other three members would be from the RBI Commerce. The merger took place on August with the governor being the ex-officio 14, 2004. Hence statement 1 is wrong. chairperson. Deputy governor of RBI in charge KELKAR TASK FORCE ON FRBM : MAJOR/ of the monetary policy will be a member, as also IMPORTANT RECOMMENDATIONS an executive director of the central bank. Direct Taxes Selection and term of members: a) Income tax Explanation : The government nominees to the i. No tax on income up to Rs.1 Lakh MPC will be selected by a Search-cum-Selection ii. 20 percent tax on income between Rs.1 Committee under Cabinet Secretary with RBI Lakh to Rs. 4 Lakh Governor and Economic Affairs Secretary and iii. 30 percent tax on income over Rs. 4 three experts in the field of economics or Lakh. banking or finance or monetary policy as its iv. Scrap all exemption, except those meant members. for housing loans, senior citizens and Term: Members of the MPC will be appointed for women. a period of four years and shall not be eligible b) Corporation Tax. for reappointment. i. Reduce tax rate from 35.875 percent to How decisions are made? 30 percent. Decisions will be taken by majority vote with ii. Reduce depreciation rate from 25 each member having a vote. percent to 15 percent. RBI governor’s role: The RBI Governor will Educational objective: To learn about recent chair the committee. The governor, however, will merging of banks and Kelkar committee’s not enjoy a veto power to overrule the other important recommendations.

www.laex.in Page No. 42 https://elearn.laex.in

www.laex.in

ECONOMY UPSC

Previous Year Questions

panel members, but will have a casting vote in 11. Answer : A case of a tie. Explanation : Deposits are liabilities of the Educational Objective: Understand Monetary banks. Liabilities are classified into Time and Policy making Framework. Demand liabilities.

9. Answer : C Time liabilities : Money deposited in Fixed Explanation : Monetary policy instruments are deposits (FD) Cash certificates gold deposits. of two types namely qualitative instruments and Demand liabilities : Money deposited in savings quantitative instruments. account Money deposited in current account The list of quantitative instruments includes Demand drafts unclaimed deposits; Open Market Operations, Bank Rate, Repo Rate, Marginal standing facility (MSF) is a window for Reverse Repo Rate, Cash Reserve Ratio, banks to borrow from the Reserve Bank of India Statutory Liquidity Ratio, Marginal standing in an emergency situation when inter-bank facility and Liquidity Adjustment Facility (LAF). liquidity dries up completely. Qualitative Instruments refer to direct action, Educational Objective : Understand Monetary change in the margin money and moral suasion. Policy Tools.

Educational Objective : Understand the tools 12. Answer : A of Monetary policy. Explanation : Repo rate: It is the rate at which

10. Answer : C Reserve Bank of India lends short term loan to Explanation : Statutory Liquidity Ratio or SLR commercial banks in the event of any shortfall is the minimum percentage of deposits that a of funds. Repo rate is used by monetary commercial bank has to maintain in the form of authorities to control inflation. liquid cash, gold or other securities. It is The Gini index or Gini coefficient is a statistical basically the reserve requirement that banks are measure of distribution developed by the Italian expected to keep before offering credit to statistician Corrado Gini in 1912. It is often customers. used as a gauge of economic inequality, The SLR is fixed by the RBI and is a form of measuring income distribution or, less control over the credit growth in India. Any commonly, wealth distribution among a reduction in SLR frees up cash for banks to lend population. as a result the banks may reduce lending rates A value of 1 for Gini Coefficient in a country The government uses the SLR to regulate implies that there is perfectly unequal income inflation and fuel growth. Increasing the SLR distribution and a value of Zero indicates will control inflation in the economy while perfectly equality. decreasing the statutory liquidity rate will cause growth in the economy. Educational Objective : Understand what is the purpose of SLR.

www.laex.in Page No. 43 https://elearn.laex.in

www.laex.in

ECONOMY UPSC

Previous Year Questions

15. Answer : D Explanation : Value of Indian currency is in general depends on demand and supply of other currencies vis-à-vis rupee. Any shortage in supply or excess of demand of a currency will lead to slide in the value of Rupee in relation to that currency.  In Other words Market forces decide the

Educational Objective : Understand the value of Currencies vis-à-vis Rupee. meaning of Repo rate and Gini coefficient.  Curbing of Imports will cause a decrease in the rate of reduction in Forex. 13. Answer : B Encouraging Indian borrowers to issue Explanation :Statutory Liquidity Ratio or SLR is rupee denominated Masala Bonds will the minimum percentage of deposits that a shield Indian companies from commercial bank has to maintain in the form of fluctuations in Dollar or other currency’s liquid cash, gold or other securities. It is rates. Hence any increase n Dollar rate basically the reserve requirement that banks are will not burden Indian companies with expected to keep before offering credit to paying more in dollars. customers.  Easing conditions relating to external Educational Objective: To understand SLR

commercial borrowing will increase the 14. Answer : C supply of foreign currency in the Explanation : Open market operations is the domestic market. sale and purchase of government securities and  Following an expansionary monetary treasury bills by RBI or the central bank of the policy will lead to inflation in the country. economy. The objective of OMO is to regulate the money Educational Objective: To understand How supply in the economy. currency rate varies.

RBI carries out the OMO through commercial 16. Answer : B banks and other financial institutions and does Explanation : The CRR/SLR is maintained to not directly deal with the public. ensure that banks have sufficient liquidity in Features: When the RBI wants to increase the order to handle any rush of bank withdrawals money supply in the economy, it purchases the and is more of a safety measure. The RBI government securities from the market and it increases the CRR/SLR when it wants to suck sells government securities to suck out liquidity out liquidity from the banking system and from the system. reduce lending capacity. Educational Objective: To understand OMO Educational Objective: To understand

objectives of monetary policy tools.

www.laex.in Page No. 44 https://elearn.laex.in

www.laex.in

ECONOMY UPSC

Previous Year Questions

17. Answer : D 5.3 Banking Sector

Explanation : Tight, or contractionary monetary 20. Answer : C policy is a course of action undertaken by a Explanation : Deposits of the bank form central bank slow down overheated economic liabilities Loans, Advances, Investment and growth, to constrict spending in an economy finance etc. form assets .Assets generate that is seen to be accelerating too quickly, or to revenue for the bank. curb inflation when it is rising too fast by Educational Objective: To understand basic reducing liquidity. banking concepts. Bank rate is the lending rate at which 21. Answer : A commercial banks can borrow from the RBI Explanation : BBB is an autonomous body of without providing any security. eminent professionals and officials for public Rising Bank rates increases the cost of sector banks (PSBs). borrowing for banks and interest rates raises It was announced by Union Government in thus liquidity decreases as loans become August 2015 as part of seven point expensive. Indradhanush Mission to revamp PSBs and Educational Objective: To understand how started functioning in April 2016. monetary policy tools effect liquidity.

It had replaced Appointments Board of 18. Answer : A Government as recommended by P J Nayak Explanation : Bank rate is the lending rate at committee. which commercial banks can borrow from the Functions RBI without providing any security. Lowering  Give recommendations for appointment of Bank rates decreases the cost of borrowing for full-time Directors as well as non-Executive banks and interest rates fall thus liquidity Chairman of PSBs. increases as loans become cheaper.  Give advice to PSBs in developing Educational Objective: To understand how differentiated strategies for raising funds monetary policy tools effect liquidity. through innovative financial methods and

19. Answer : A instruments and to deal with issues of Explanation : Cash Reserve Ratio (CRR) is a stressed assets. specified minimum fraction of the total deposits  Guide banks on mergers and consolidations of customers, which commercial banks have to and governance issues to address bad loans hold as reserves either in cash or as deposits problem among other issues. with the central bank. Composition: BBB comprises of three ex-officio Educational Objective: To understand members (from government) and three expert monetary policy tools. members, two of which are from private sector

in addition to Chairman.

www.laex.in Page No. 45 https://elearn.laex.in

www.laex.in

ECONOMY UPSC

Previous Year Questions

Educational Objective: To learn about SLR component has Govt. Securities,by buying functions of Important bodies associated with Govt. securities Banks provide to the Indian Banking sector. government.

22. Answer : C Educational Objective: To Understand Explanation : Recaptialisation of PSBs concepts of Monetary tools.

According to the Comptroller and Auditor 24. Answer : B General’s Report (2017), the government had Explanation : Non-Banking Financial infused Rs 1,18,724 crore in PSBs between Companies (NBFCs): 2008-09 to 2016-17. Recapitalistion effort These are establishments that provide financial announced by the government in 2017 services and banking facilities without meeting The government has announced a major the legal definition of a Bank. Recapitalisation drive in 2017 by utilizing three Hence they are frequently referred to as “shadow channels – budget, market borrowings and issue banks. of recapitalisation bonds. They are covered under the Banking regulations 1) Budgetary allocations: government will buy laid down by the Reserve Bank of India and Rs 18,000 crore worth of public sector bank provide banking services like loans, credit shares. facilities, TFCs, retirement planning, investing 2) Market Borrowings: PSBs will mobilise Rs and stocking in money market. 58,000 crore from the market through However, they are restricted from taking any borrowings. form of deposits from the general public. 3) Recapitalisation Bonds: Government will However, certain types of Deposits taking issue Bank Recapitalization Bonds worth Rs NBFCs are allowed to take deposits. 1,35,000 crore which will be used to buy Educational Objective: To Understand the more shares in public sector banks. functions of NBFCs.

State Bank of Bikaner and Jaipur (SBBJ), State 25. Answer : C Bank of Hyderabad (SBH), State Bank of Mysore Explanation : The Lead Bank Scheme was (SBM), State Bank of Patiala (SBP) and State introduced in 1969, to guide commercial, Bank of Travancore (SBT), besides Bharatiya regional, rural and Co-operatives to improve Mahila Bank (BMB), merged with SBI with effect their facilities in rural areas and deliver effective from in April 1,2017 services,on the basis of the study group under Educational Objective: To learn about Bank the Chairmanship of Prof. D.R. Gadgil. recapitalization and Merger of SBI associate A “district credit” plan combined with banks. government schemes is prepared and monitored

23. Answer : D by the lead bank. Explanation : Statutory Liquidity Ratio or SLR Objectives of The Lead Bank Scheme: is the minimum percentage of deposits that a  To identify unbanked and underbanked commercial bank has to maintain in the form of centres in districts and to evaluate their liquid cash, gold or Government securities. www.laex.in Page No. 46 https://elearn.laex.in

www.laex.in

ECONOMY UPSC

Previous Year Questions

physiographic, agro climatic end Socio- determined internally by the bank depending on economic conditions through economic the period left for the repayment of a loan. RBI survey. introduced MCLR in 2016  To help in removing regional imbalances Beginning from April 1st,2019 RBI has asked through appropriate credit deployment; lenders to link all floating rate loans to external  To extend banking facilities to unbanked bench marks like Repo rate, T-bill yield etc. areas;Etc Educational Objective: To understand Banking Educational Objective: To Learn about Lead concepts.

bank Scheme. 29. Answer : C

26. Answer : B Explanation : Explanation : The aim of the Lead Bank The functions of commercial banks in India scheme is to provide adequate banking and include credit in rural areas through an ‘service area 1. Collection of Cheques, Dividends, Interests approach’, with one bank assigned for one area. 2. Payment of Subscription, Rent, Insurance Educational Objective: To Learn about Lead Premium etc. bank Scheme. 3. Conduct of Stock Exchange Transactions on

27. Answer : D behalf of their customers. Explanation : First Narasimhan Committee 4. Acting as Executor, Trustees, Attorneys Report – 1991 5. Preparation of Income Tax Returns To promote the healthy development of the 6. Conducting Foreign Exchange Transactions financial sector Educational Objective: To understand 2nd Narasimham Committee Report-1998 : It is functions of a commercial bank

better known as the Banking Sector Committee. 30. Answer : B It was told to review the banking reform Explanation : The first bank of limited liability progress and design a programme for further managed by Indians and founded in 1881 was strengthening the financial system of India. Oudh Commercial Bank. Educational Objective: To Learn about Oldest bank but liquidated: Bank of important committees on financial sector Hindustan(1770-1832) reforms. First bank purely managed by Indian :Punjab 28. Answer : D National Bank(1895) Explanation : Commercial banks fix Interest Educational Objective: To Know basic history rates based on cost of funds measured by about Banking in India.

MCLR(Marginal cost of lending rates). 5.4 Lending Rates The marginal cost of funds-based lending rate 31. Answer : C (MCLR) is the minimum interest rate that a Explanation : The marginal cost of funds- bank can lend at. MCLR is a tenor-linked based lending rate (MCLR) is the minimum internal benchmark, which means the rate is interest rate that a bank can lend at. MCLR is a www.laex.in Page No. 47 https://elearn.laex.in

www.laex.in

ECONOMY UPSC

Previous Year Questions

tenor-linked internal benchmark, which means  The entities eligible to start their business as the rate is determined internally by the bank per the RBI directives as Payment Banks depending on the period left for the repayment includes non-banking prepaid instrument of a loan. RBI introduced MCLR in 2016. issuers, mobile telecom companies, Non- These guidelines help improve the transparency Banking Finance Companies(NBFC), in the methodology followed by banks for corporate business correspondents, co- determining the interest rates on advances and operatives and supermarket chains. NBFC’s, help ensure availability of bank credit at interest microfinance institutions, individuals with rates which are fair to the borrowers as well as experience of banking and finance societies the banks. for ten years and local area banks qualify for Educational Objective: To understand Banking starting Small Banks. concepts-MCLR. Educational Objective: To understand

5.5 Finacial Inclusion functions of Payment banks.

33. Answer : C 32. Answer : B Explanation : Banking Correspondents (BCs) Explanation : Payment banks is a new model are individuals/entities engaged by a bank in of banks conceptualised by the Reserve Bank of India (commercial banks, Regional Rural Banks India (RBI). On the basis of Nachiket Mor (RRBs) and Local Area Banks (LABs)) for committee recommendations. providing banking services in unbanked / 1) These banks can accept a restricted deposit, under-banked geographical territories. which is currently limited to ₹ 100,000 per A banking correspondent works as an agent of customer and may be increased further. the bank and substitutes for the brick and 2) These banks cannot issue loans and credit mortar branch of the bank. cards. Banking correspondents were allowed by RBI 3) Both the current account and savings vide a circular dated 25 January 2006. The accounts can be operated by such banks. concept of Banking Correspondent stemmed 4) Payment banks can issue services like ATM from a report of H R Khan, Dy Governor of RBI. cards, debit cards, net-banking and mobile Committee on Financial Inclusion Chaired by banking. Dr. C. Rangarajan which in its report submitted Regulations under Payment Bank had also recommended for the expansion of the  The minimum capital requirement is 100 BC model. crore. Functions:  For the first five years, the stake of the  Identification of borrowers; promoter should remain at least 40%. The  Collection and preliminary processing of loan foreign shareholding will be allowed in these applications including verification of primary banks as per the rules for FDI in private information/data; banks in India.

www.laex.in Page No. 48 https://elearn.laex.in

www.laex.in

ECONOMY UPSC

Previous Year Questions

 Creating awareness about savings and other loans to state sponsored organisations for products and education and advice on SC/ST. managing money and debt counseling; Educational Objective: To Learn about Priority  Processing and submission of applications to Sector Lending as Qualitative tool of monetary banks; policy.

 Promoting, nurturing and monitoring of Self- 35. Answer : C Help Groups/ Joint Liability Groups/Credit Explanation : Regional Rural Banks(RRBs) Groups/others; grant loans to individuals , firms , Households  Post-sanction monitoring; etc. in rural areas.  Follow-up for recovery, NABARD is a regulating and refinancing agency  Disbursal of small value credit, of rural financial institutions.  Recovery of principal / collection of interest Land Development Banks lend funds to large  Collection of small value deposits scale projects in rural areas.  Sale of micro insurance/ mutual fund products/ pension products/ other Companies registered under the Indian Companies Act, 2013 with large and widespread retail outlets Educational Objective: To understand functions of Banking Correspondents

34. Answer : D

Explanation : Priority Sector refers to those sectors of the economy which may not get timely Educational Objective: To understand rural and adequate credit. Priority Sector Lending is financing ecosystem.

an important role given by the Reserve Bank of 36. Answer: D India (RBI) to the banks for providing a specified Explanation : Financial inclusion is defined as portion of the bank lending to few specific the availability and equality of opportunities to sectors. access financial services. It refers to a process As per the RBI there are eight broad categories by which individuals and businesses can access of the Priority Sector Lending. appropriate, affordable, and timely financial They are: (1) Agriculture (2) Micro, Small and products and services. These include banking, Medium Enterprises (3) Export Credit (4) loan, equity, and insurance products. Education (5) Housing (6) Social Infrastructure Nationalization of Banks,Formation of Regional (7) Renewable Energy (8) Others. Rural Banks,Adoption of villages by Bank The others category includes personal loans to Branches etc are steps towards financial weaker section, loans to distressed persons, inclusion.

www.laex.in Page No. 49 https://elearn.laex.in

www.laex.in

ECONOMY UPSC

Previous Year Questions

Educational Objective: To understand financial  Beneficiaries get a biometric-enabled smart inclusion with examples. card containing their fingerprints and

37. Answer : D photographs Explanation : Rashtriya Swasthya Bima Ministry of Textiles launched the Rajiv Yojana: Gandhi Shilpi Swasthya Bima Yojana Ministry/Department: Ministry of Health & Scheme is aimed towards providing financial aid Family Welfare (Earlier it was under Ministry of to the artisan community to access the best of Labour and Employment) healthcare facilities in the country.

Objective: The scheme includes the following criterion for those artisans who wish to avail it-  To provide financial protection against catastrophic health costs i) Handicraft artisans ii) Annual limit per family (1+4) (includes self,  To improve access to quality health care for spouse and three dependent family members), below poverty line households and other all craft members whether male or female vulnerable groups in the unorganized sector between the age of group of one day to 80 years,  Scheme: all craft persons will be eligible to be covered  A centrally sponsored health insurance under the scheme subject to the conditions as scheme applicable.  The premium cost for enrolled beneficiaries Educational Objective: To learn about the under the scheme is shared by Government social security schemes.

of India and the State Governments

 It was initially designed to target only the 38. Answer : D Below Poverty Line (BPL) households, but Explanation : As per OECD (Organisation for has been expanded to cover other defined Economic Co-operation and Development), categories of unorganised workers, covering inclusive growth is economic growth that is construction workers, street vendors etc. distributed fairly across society and creates opportunities for all.  The beneficiaries under RSBY are entitled to Elements of Inclusive Growth hospitalization coverage up to Rs. 30,000/- per annum on family floater basis, for most  Skill Development of the diseases that require hospitalization.  Financial Inclusion  The coverage extends to maximum five  Promoting Self-Help Groups members of the family which includes the  Technological Advancement head of household, spouse and up to three  Quality and affordable Education to all dependents.  Implementing the Right to Education Act  Provision to pay transport expense is also  Quality and affordable Health to all there subject to limits  Micro, Small and Medium Enterprises Etc.  The beneficiaries need to pay only Rs. 30/- Educational Objective : To understand as registration fee for a year Inclusive growth.

www.laex.in Page No. 50 https://elearn.laex.in

www.laex.in

ECONOMY UPSC

Previous Year Questions

39. Answer : C Administered By: Pension Fund Regulatory and Explanation : Pradhan Mantri Jan Dhan Development Authority through National Yojana (PMJDY) was introduced with a target of Pension System (NPS). opening bank accounts for around 7.5 crores Eligibility: Any citizen of India can join the APY uncovered households in India by 26th of scheme. The age of the subscriber should be January, 2015. This scheme was successful in between 18-40 years. The contribution levels opening around 12.54 crore accounts by 31st would vary and would be low if a subscriber January 2015 with deposits exceeding Rs. joins early and increases if she joins late. 10,000 crores. Benefits:  The scheme covers both the urban as well as  It provides a minimum guaranteed pension the rural areas and provides an indigenous ranging from Rs 1000 to Rs 5000 on Debit Card (RuPay card) to each of its attaining 60 years of age. account holders.  The amount of pension is guaranteed for  No minimum balance is required for opening lifetime to the spouse on death of the an account under this scheme. The subscriber. beneficiary can open his/her account in any  In the event of death of both the subscriber bank branch or Business Correspondent and the spouse, the entire pension corpus is outlet at zero balance. paid to the nominee  It ensures to provide on Core Banking Educational Objective: To learn about the System (CBS) along with mobile banking Government schemes.

using USSD facilities. 41. Answer : D  Rs. 5,000 overdraft facility for Aadhar-linked Explanation : Employees’ State Insurance accounts. Corporation (ESIC) scheme  RuPay debit card with inbuilt Rs. 1 lakh  It functions under the Ministry of Labour & accident insurance cover is one of the main Employment. features provided by this scheme.  It protects the employees against financial Educational Objective: To learn about the distress arising out of events of sickness, Government schemes. disablement or death due to employment

40. Answer : C injuries. Explanation : Atal Pension Yojana The scheme  ESI provides for direct cash compensation was launched on 9th May, 2015, with the for the above cases. objective of creating a universal social security  Employees’ State Insurance Corporation system for all Indians, especially the poor, the (ESIC) is responsible for the administration under-privileged and the workers in the of ESI Scheme. unorganised sector.  ESIC is a statutory corporate body set up under the ESI Act, 1948.  Eligibility:

www.laex.in Page No. 51 https://elearn.laex.in

www.laex.in

ECONOMY UPSC

Previous Year Questions

All Organised sector Employees whose monthly and has been issued by 300 cooperative banks wages are or below Rs 21000. and Regional Rural Banks (RRBs) in India. What are the benefits? It was created to fulfill the Reserve Bank of Medical Benefit - Full medical care to the India’s desire to have a domestic, open loop, and insured person and his family members with no multilateral system of payments in India. RuPay ceiling on expenditure of the treatment. facilitates electronic payment at all Indian Sickness Benefit - In the form of cash banks and financial institutions, and competes compensation at the rate of 70 per cent of with MasterCard and Visa in India. wages. Educational objective : To learn about NPCI & Maternity Benefit - For confinement/pregnancy RUPAY

is payable for 26 weeks, which is extendable by 43. Answer : C further one month on medical advice. Disablement Benefit - Explanation : Merchant Discount Rate(MDR) is Temporary disablement benefit (TDB) the cost paid by a merchant to a bank for Permanent disablement benefit (PDB) accepting payment from their customers via Dependants Benefit - Paid in the form of digital means. It is expressed in percentage of monthly payment to the dependants in cases the transaction amount. where death is due to employment injury or Presently, it is applicable for online transactions occupational hazards. and QR-based transactions. Educational Objective: To learn about the The amount that the merchant pays for every Government schemes. transaction gets distributed among three

5.6 Payment Systems & NPCI stakeholders--the bank that enables the

transaction, vendor that installs the point of 42. Answer : C sale (PoS) machine and the card network Explanation : National Payments Corporation provider such as Visa, MasterCard, RuPay. of India (NPCI) is an umbrella organisation for Educational objective : To learn about operating retail payments and settlement Merchant Discount Rate

systems in India. It is an initiative of Reserve Bank of India (RBI) and Indian Banks’ 44. Answer : C Association (IBA) under the provisions of the Explanation : National Payments Corporation of Payment and Settlement Systems Act, 2007. India(NPCI) was founded on 2008 for the It has been incorporated as a “Not for Profit” operation of retail payments in India. It is the Company under the provisions of Section 25 of apex authority for maintaining a nationwide link Companies Act 1956 (now Section 8 of of all the ATMs in India. Companies Act 2013). NPCI exercises this role through its wing IDRBT NPCI launched RuPay, a domestic card scheme (Institute for Development of Research in of India that has a magnetic stripe along with an Banking Technology). IDRBT created a massive EMV chip. The card is now accepted at all ATMs www.laex.in Page No. 52 https://elearn.laex.in

www.laex.in

ECONOMY UPSC

Previous Year Questions

network called National Financial Switch in on all lenders. The dissenting creditors will, the year 2004 for linking all the ATMs in India. however, have the option to sell their loans to Other functions are Immediate Payment System other lenders at a discount of 15% to the (IMPS), Affiliation of RuPay Cards, National liquidation value, or buy the entire portfolio Automatic Clearing House (ACH), Aadhaar paying 125% of the value agreed under the debt Enabled Payments System (AEPS), Operation of resolution plan by other lenders. Cheque Truncation System. Educational objective : To learn about Inter- Educational objective : To learn about Creditor Agreement.

functions of NPCI. 47. Answer : B

45. Answer : B Explanation : The Reserve Bank of India (RBI) Explanation : Every currency note bears on its has issued guidelines called Scheme for face a promise from the Governor of RBI that if Sustainable Structuring of Stressed Assets (S4A) someone produces the note to RBI, or any other as an optional framework to manage bad loans. commercial bank, RBI will be responsible for Under this scheme, large ticket loans are giving the person purchasing power equal to the restructured by separating a sustainable loan value printed on the note. The same is also true from an unsustainable loan. of coins. They are also called legal tenders as The lenders are required to make this they cannot be refused by any citizen of the classification. Sustainable level of debt is one country for settlement of any kind of transaction which the banks think the stressed borrower . Hence, B is the most appropriate answer. can service with its current cash flows. Educational objective : To learn about legal Educational objective : To learn about Scheme tender money for Sustainable Structuring of Stressed Assets

5.7 NPAS & Stressed Assets (S4A)

46. Answer : D 5.8 Basel Norms & Capital Adequacy Ratio Explanation : Inter-Creditor Agreement (ICA) is

aimed at the resolution of loan accounts with a 48. Answer : A size of ₹ 50 crore and above that are under the Explanation : Capital Adequacy Ratio (CAR) is control of a group of lenders. It is part of the the ratio of a bank’s capital in relation to its risk “Sashakt” plan approved by the government to weighted assets and current liabilities. It is address the problem of resolving bad loans. measured as: The agreement is based on a recommendation Capital Adequacy Ratio = (Tier I + Tier II + Tier by the Sunil Mehta committee that looked into III (Capital funds)) /Risk weighted assets resolution of stressed assets. The risk weighted assets take into account The agreement says if 66% of lenders by value credit risk, market risk and operational risk. agree to a resolution plan, it would be binding

www.laex.in Page No. 53 https://elearn.laex.in

www.laex.in

ECONOMY UPSC

Previous Year Questions

It is decided by central banks and bank Explanation : Basel guidelines refer to broad regulators to prevent commercial banks from supervisory standards formulated by group of taking excess leverage and becoming insolvent central banks- called the Basel Committee on in the process. Banking Supervision (BCBS). The set of Educational objective : To learn about Capital agreement by the BCBS, which mainly focuses Adequacy Ratio (CAR) on risks to banks and the financial system are

called Basel accord. 49. Answer : B Basel is a city in Switzerland which is also the Explanation : Basel guidelines refer to broad headquarters of Bureau of International supervisory standards formulated by group of Settlement (BIS). central banks- called the Basel Committee on The purpose of the accords is to ensure that Banking Supervision (BCBS). The set of financial institutions have enough capital on agreement by the BCBS, which mainly focuses account to meet obligations and absorb on risks to banks and the financial system are unexpected losses. called Basel accord. BASEL-II: Basel is a city in Switzerland which is also the headquarters of Bureau of International Published in 2004. Settlement (BIS). Banks should maintain a minimum capital The purpose of the accords is to ensure that adequacy requirement of 8% of risk assets. financial institutions have enough capital on Banks were needed to develop and use better account to meet obligations and absorb risk management techniques in monitoring and unexpected losses. managing all the three types of risks that is Basel III: credit and increased disclosure requirements. The three types of risk are- operational risk, In 2010, Basel III guidelines were released. market risk, capital risk. These guidelines were introduced in response to the financial crisis of 2008. Educational objective : To learn about Basel Norms Basel III norms aim at making most banking

activities such as their trading book activities 5.9 Money Supply & Inflation more capital-intensive. 51. Answer : D The guidelines aim to promote a more resilient Deficit financing always brings inflationary banking system by focusing on four vital pressure whatever be the means but 'creation' banking parameters viz. capital, leverage, (printing) of new money has the highest funding and liquidity. inflationary impact. Because it increases money Educational objective : To learn about Basel supply without any increase in production of Norms goods and services

50. Answer : D Educational objective : To learn about Effects of Inflation www.laex.in Page No. 54 https://elearn.laex.in

www.laex.in

ECONOMY UPSC

Previous Year Questions

52. Answer : A current inflation. Ex. If inflation in march 2018 Explanation : Inflation is the general increase was 1% and absolute increase in Price index in in prices and fall in the purchasing value of march 2019 was say 4%, then, inflation in money. Inflation decreases the value of money march 2019 will be low i.e. 3% over time. Money loses value when its Educational objective : To learn about Base purchasing power falls. effect. Inflation harms creditors, as they lose in real 54. Answer : D terms. It is because the money they get paid Explanation : Increase in the supply of money back has less purchasing power than the money leads to more spending and debt. This leads to a they loaned out. steady increase in demand, which means higher Inflation benefits the Debtor as they gain in real prices. Hence, statement 1 and 3 are correct. terms. Inflation reduces the value of money. A decrease in the aggregate level of output is interest rate that a borrower pays is effectively majorly due to rise in production cost. This lower thanks to inflation. leads to price rise and inflation. Hence, Educational objective : To learn about Effects statement 2 is correct. of Inflation Educational objective : To learn about impacts

of inflation on demand and supply 53. Answer : C

Explanation : The base effect refers to the 55. Answer : B impact of the rise in price level (i.e. last year’s Explanation : Supply remaining the same, inflation) in the previous year over the increase in demand for money will lead to more corresponding rise in price levels in the current options to lend money for the banks. Thus, year (i.e., current inflation) banks will lend money at greater rate of interest If the price index had risen at a high rate in the to earn more money as there are more corresponding period of the previous year customers for the same amount of money. leading to a high inflation rate, some of the Hence, statement B is correct. potential rise is already factored in, therefore a Educational objective : To learn about impact similar absolute increase in the Price index in of inflation on demand and supply of economy the current year will lead to a relatively lower 56. Answer : C inflation rates. Ex. If inflation in march 2018 Explanation : A price index is a normalized was 8% and absolute increase in Price index in average (typically a weighted average) of price march 2019 was say 9%, then, inflation in relatives for a given class of goods or services in march 2019 will be low i.e. 1% a given region, during a given interval of time. It On the other hand, if the inflation rate was too is a statistic designed to help to compare how low in the corresponding period of the previous these price relatives, taken as a whole, differ year, even a relatively smaller rise in the Price between time periods or geographical locations. Index will arithmetically give a high rate of The current price index (base 1960) is nearly www.laex.in Page No. 55 https://elearn.laex.in

www.laex.in

ECONOMY UPSC

Previous Year Questions

330 which implies that the price if the weighted Ministry of Commerce and Industry. WPI figures mean of price of certain items has increased 3.3 are published on a monthly basis. times. In India, wholesale price index is divided into Educational objective : To learn about current three groups: Fuel and Power (13.2 percent), price index Primary Articles (22.6 percent of total weight) and Manufactured Products (654.2 percent). 57. Answer : D Food Articles from the Primary Articles Group Explanation : Reserve Money (M0): It is also account for 15.2 percent of the total weight. The known as High-Powered Money, monetary base, most important components of the base money etc. Manufactured Products Group are Basic Metals M0 = Currency in Circulation + Bankers’ (9.7 percent of total weight); Food products (9.1 Deposits with RBI + Other deposits with RBI. percent); Chemicals and Chemical products (6.5 It is the monetary base of the economy. percent) and Textiles (4.9 percent). . Narrow Money (M1): Educational objective : To learn about WPI M1 = Currency with public + Demand deposits 59. Answer : C with the Banking system (current account, The WPI is average change of prices of a fixed saving account) + Other deposits with RBI basket of goods at the first point of bulk sale in M2 = M1 + Savings deposits of post office the domestic market over a given period of time savings banks Wholesale Price Index (WPI) series in India has Broad Money (M3) undergone six revisions in 1952-53, 1961-62, M3 = M1 + Time deposits with the banking 1970-71, 1981-82, 1993-94 and 2004-05 so system far. The base year of All-India WPI has been M4 = M3 + All deposits with post office savings revised from 2004-05 to 2011-12 on 12 May banks 2017 to align it with the base year of other Educational objective : To learn about macroeconomic indicators like the Gross Measures of Money Supply Domestic Product (GDP) and Index of Industrial 58. Answer : B Production (IIP). The current series is the seventh revision. Wholesale Price Index (WPI) Educational objective : To learn about WPI The WPI is average change of prices of a fixed

basket of goods at the first point of bulk sale in 60. Answer : A the domestic market over a given period of time Explanation : Fiscal stimulus refers to policy WPI measures the inflation at wholesale level. measures undertaken by a government that The present base year of WPI is 2011-12 typically reduce taxes or regulations—or Nodal Office for Compilation and release of WPI increase government spending—in order to is the Office of Economic Adviser, Department boost economic activity. Hence, statements 1 &2 for Promotion of Industry and Internal Trade, are correct. www.laex.in Page No. 56 https://elearn.laex.in

www.laex.in

ECONOMY UPSC

Previous Year Questions

Abolishing the subsidies reduces the However, deposit of currency in commercial government spending. Hence, statement 3 is banks by public doesn't lead to increase in wrong. money supply as the money just gets Educational objective : To learn about fiscal transferred from public to commercial banks. stimulus. Sale of government securities to the public by the Central Bank leads to decrease in money 61. Answer : C supply as RBI absorbs the liquidity with the Explanation : Inflation targeting is a monetary Public. Hence , statement 4 is wrong. policy strategy used by Central Banks for Educational objective : To learn about Money maintaining price level at a certain level or supply and role of RBI in money supply

within a range. It indicates the primacy of price stability as the key objective of monetary policy. 63. Answer : C The Reserve Bank of India and Government of The WPI is average change of prices of a fixed India signed a Monetary Policy Framework basket of goods at the first point of bulk sale in Agreement according to which RBI would aim the domestic market over a given period of time to contain consumer price inflation within 6 The present base year of WPI is 2011-12 percent and within 4 percent with a band of WPI figures are published on a monthly basis. (+/-) 2 percent. Hence, statements 1&2 are In India, wholesale price index is divided into wrong. three groups: Fuel and Power (13.2 percent), Increasing the money supply faster than the Primary Articles (22.6 percent of total weight) growth in real output will cause inflation. The and Manufactured Products (654.2 percent). reason is that there is more money chasing the Food Articles from the Primary Articles Group same number of goods. Therefore, the increase account for 15.2 percent of the total weight. in monetary demand causes firms to put up Consumer Price Index (CPI) is based on the final prices. Hence, statement 3 is correct. prices of goods at the retail level. In India, RBI Educational objective : To learn about Inflation uses CPI (combined) released by CSO for 62. Answer : C inflation purpose with base year as 2012. Explanation : Money supply increases when The number of items in CPI basket include 448 money flows out of the RBI. Purchase of in rural and 460 in urban. Government securities from the public by the The weightage for food and beverages in CPI Central bank leads to transfer of money to the is 54.18. public thereby leading to increase in money Educational objective : To learn about WPI

supply in the economy. Similarly, borrowing by 64. Answer : C the government from Central bank leads to Explanation : Consumer Price Index (CPI) increased money supply. Hence , statement 1 and 3 are correct. It measures price changes from the perspective of a retail buyer.

www.laex.in Page No. 57 https://elearn.laex.in

www.laex.in

ECONOMY UPSC

Previous Year Questions

It measures changes over time in the level of The sovereign gold bond scheme was launched retail prices of selected goods and services on in November 2015. Its objective is to reduce the which consumers of a defined group spend their demand for physical gold and shift a part of the incomes. domestic savings (used for the purchase of gold) Four types of CPI are as follows: into financial savings. 1. CPI for Industrial Workers (IW). Gold Monetisation Scheme 2. CPI for Agricultural Labourer (AL). The scheme was launched in November 2015 3. CPI for Rural Labourer (RL). along with sovereign gold bonds and India gold 4. CPI (Rural/Urban/Combined). coins. Of these, the first three are compiled by the It facilitates the depositors of gold to earn Labour Bureau in the Ministry of Labour and interest on their metal accounts. Once the gold Employment. Fourth is compiled by the Central is deposited in metal account, it starts earning Statistical Organization (CSO) in the Ministry of interest on the same. Statistics and Programme Implementation. Under the scheme, a depositor gets 2.25% Base Year for CPI is 2012. interest annually for a short-term deposit of one Educational objective : To learn about CPI year to three years. Medium- and long-term

deposits get 2.5% interest rate. 5.10 Miscellaneous

Objective: To mobilize the gold held by 65. Answer : B households and institutions in the country to Explanation : Money multiplier measures the put this gold into productive use and in the long amount of money that the banks are able to run to reduce the current account deficit by create in the form of deposits with each unit of reducing the country’s reliance on imports of money it keeps as gold to meet the domestic demand. reserves. Its value determined in ratio of total Educational objective : To learn about money supply to the stock of the high powered Sovereign Gold Bond Scheme' and 'Gold money in an economy. Thus the money Monetization Scheme' multiplier increases the banking habit of the 67. Answer : C population. Explanation : Higher interest rates mean higher Educational objective : To learn about Money borrowing costs, people will eventually start multiplier in the economy spending less which implies lower savings and

66. Answer : C lower tax collections. Explanation : Sovereign gold bonds are issued Conversely, falling interest rates means by the RBI on behalf of the government. They borrowing money becomes cheaper; this entices are government securities denominated in people to start spending again. This leads to grams of gold. They are substitutes for holding investments. Hence, statement 3 is correct. physical gold.

www.laex.in Page No. 58 https://elearn.laex.in

www.laex.in

ECONOMY UPSC

Previous Year Questions

68. Answer : D Educational objective : To learn about Banking Explanation : Narrow Money (M1) = Currency Ombudsman scheme

with the Public + Demand Deposits with the 70. Answer : C Banking System + ‘Other’ deposits with the RBI. Explanation : source : RBI M2 = M1 + Savings Deposits of Post office Savings Banks. Broad Money (M3) = M1 + Time Deposits with the Banking System. M4 = M3 + All deposits with Post Office Savings Banks (excluding National Savings Certificates). Liquidity-As we move from M1 to M4 the liquidity of the money goes on decreasing. Hence, option D is the correct answer. Educational objective : To learn about savings Educational objective : To learn about rate in India monetary aggregates 71. Answer : A

69. Answer : C Explanation : IFCI, previously Industrial Explanation : Banking ombudsman is a quasi Finance Corporation of India, is a Non-Banking judicial authority Finance Company in the public sector. It is created to resolve customer complaints Established in 1948 as a statutory corporation. against banks relating to certain services ICICI was formed in 1955 at the initiative of the provided by them. World Bank, the Government of India and The Ombudsman is a senior official, who has representatives of Indian industry. The principal been appointed by the Reserve Bank of India to objective was to create a development financial address grievances and complaints from institution for providing medium-term and long- customers, pertaining deficiencies in banking term project financing to Indian businesses. services. Industrial Development Bank of India (IDBI) It covers all kinds of banks including public was constituted under Industrial Development sector banks, Private banks, Rural banks as Bank of India Act, 1964 as a Development well as co-operative banks. Financial Institution (DFI) and came into being The Banking Ombudsman does not charge any as on July 01, 1964 vide GoI notification dated fee for filing and resolving customers’ June 22, 1964. complaints. NABARD was established on the The Appellate Authority is vested with a Deputy recommendations of B. Sivaraman Committee, Governor of the RBI. on 12 July 1982 to implement the National Bank for Agriculture and Rural Development Act 1981. www.laex.in Page No. 59 https://elearn.laex.in

www.laex.in

ECONOMY UPSC

Previous Year Questions

Educational objective : To learn about major  It co-ordinates functions of institutions financial institutions in India. engaged in similar activities.

 Currently, the shares of SIDBI are held by 72. Answer : C Central Government and 29 other Explanation : NHB It was established as a institutions including public sector banks wholly owned subsidiary of Reserve Bank of (PSBs), insurance companies owned and India (RBI). Now, its owned by GoI. controlled by Central Government.  It was set up on 9 July 1988 under the  It was established as a wholly-owned National Housing Bank Act, 1987 subsidiary of the Industrial Development  NHB is an apex financial institution for Bank of India housing. Educational objective : To learn about NHB &  NHB has been established with an objective SIDBI.

to operate as a principal agency to promote housing finance institutions both at local 73. Answer : C and regional levels and to provide financial Explanation : An Ordinance was issued on and other support incidental to such 19th January, 1956 nationalising the Life institutions and for matters connected Insurance sector and Life Insurance Corporation therewith. came into existence in the same year. The LIC  NHB registers, regulates and supervises absorbed 154 Indian, 16 non-Indian insurers as Housing Finance Company (HFCs), keeps also 75 provident societies—245 Indian and surveillance through On-site & Off-site foreign insurers in all. The LIC had monopoly till Mechanisms and co-ordinates with other the late 90s when the Insurance sector was Regulators. reopened to the private sector. SIDBI State Bank of India (SBI) was nationalised in  It is the principal development financial July 1955 under the SBI Act of 1955. institution for promotion, financing and Nationalisation of Seven State Banks of India development of Micro, Small and Medium (formed subsidiary) took place on 19th July, Enterprises (MSME) sector in India. 1960.  It was established on April 2, 1990 through Banking regulation act was passed in 1949. an Act of Parliament (thus, it is statutory Initially, the law was applicable only to banking body). companies. But, 1965 it was amended to make  It is headquartered in Lucknow, Uttar it applicable to cooperative banks and to Pradesh. introduce other changes.  SIDBI aims to facilitate and strengthen The concept of economic planning in India is credit flow to MSMEs and address both derived from the Russia (then USSR). India has financial and developmental gaps in MSME launched 12 five year plans so far. First five year eco-system across the country. plan was launched in 1951.

www.laex.in Page No. 60 https://elearn.laex.in

www.laex.in

ECONOMY UPSC

Previous Year Questions

Educational objective : To know about major sub-prime lending. These home buyers would events in Indian economy initially pay the loan installments for few years but eventually default in case of an economy 74. Answer : D downturn which might bring about jobs Explanation : Microfinance is the practice of disruption and their earning capacities . This extending a small loan or other form of credit, ultimately may lead to a scary situation for the savings, checking, or insurance products to entire banking system . The correct option is A. individuals who do not have access to this type Educational objective: To learn and of capital. This allows individuals who are living understand about teaser loans.

in poverty to work on becoming financially

independent so they can work their way into 6. Fiscal Policy better living conditions. 6.1 Taxation It helps help disadvantaged households and entrepreneurs gain access to affordable financial 6.1.1 Direct Tax

services to help them finance income-generating 1. Under which of the following activities, accumulate assets through savings, circumstances may ‘capital gains’ provide for family needs, and protect themselves arise? against the risks of daily life, such as illness, 1) When there is an increase in the sales of a death, theft, natural disasters. product The micro-credit industry in India is Inspired by 2) When there is a natural increase in the the success of the Grameen Bank model in value of the property owned Bangladesh. 3) When you purchase a painting and there is a growth in its value due to increase in Today, the industry reaches over 50 million low- its popularity. income women customers across 600 districts, Select the correct answer using the codes with an average loan size of ₹ 30,000 and given below: aggregate loan outstanding of ₹ 2 trillion, as a) 1 only estimated. b) 2 and 3 only Educational objective : To know about c) 2 only Microfinance

d) 1, 2 and 3

75. Answer : A 2. Consider the following statements: Explanation : A Teaser loan refers to a loan In India, taxes on transactions in Stock which is provided for a fixed period and could Exchanges and Futures Markets are then be withdrawn. It comes with a lower rate of 1) levied by the Union interest during the initial years and then start 2) collected by the States charging normal interest rates. Which of the statements given above is/are These loans are usually targeted at low-income correct? home buyers ,they are considered an aspect of a) 1 only www.laex.in Page No. 61 https://elearn.laex.in

www.laex.in

ECONOMY UPSC

Previous Year Questions

b) 2 only 6. The term 'Base Erosion and Profit c) Both 1 and 2 Shifting' is sometimes seen in the news d) Neither 1 nor 2 in the context of

3. Consider the following: a) Mining operation by multinational 1) Fringe Benefit Tax companies in resource-rich but backward 2) Interest Tax areas 3) Securities Transaction Tax b) Curbing of the tax evasion by Which of the above is/are Direct Tax/Taxes? multinational companies a) 1 only c) Exploitation of genetic resources of a b) 1 and 3 only country by multinational companies. c) 2 and 3 only d) Lack of consideration of environmental d) 1, 2 and 3 planning and developmental costs in the implementation of projects.

4. Which of the following is not a

recommendation of the task force on 6.1.3 Indirect Tax

direct taxes under the Chairmanship of 7. Consider the following items: Dr. Vijay Kelkar in the year 2002? 1) Cereal grains hulled a) Abolition of Wealth Tax 2) Chicken eggs cooked b) Increase in the exemption limit of personal 3) Fish processed and canned income to Rs 1.20 lakh for widows 4) Newspapers containing advertising c) Elimination of standard deduction material d) Exemption from tax on dividends and Which of the above items is/are exempted capital gains from the listed equity under GST (Goods and Services Tax)?

5. Which one of the following statements a) 1 only regarding the levying, collecting and b) 2 and 3 only distribution of Income Tax is correct? c) 1, 2 and 4 only a) The Union levies, collects and distributes d) 1, 2, 3 and 4 the proceeds of income tax between itself 8. What is/are the most likely advantages and the states of implementing “Goods and Services

b) The Union levies, collects and keeps all the Tax (GST)”? proceeds of income tax itself 1) It will replace multiple taxes collected by

c) The Union levies and collects the tax but multiple authorities and will thus create a all the proceeds are distributed among the single market in India. states 2) It will drastically reduce the 'Current

d) Only the surcharge levied on income tax is Account Deficit of India and will enable it shared between the Union and the states to increase its foreign exchange reserves.

6.1.2 Tax Avoidance and Tax evasion

www.laex.in Page No. 62 https://elearn.laex.in

www.laex.in

ECONOMY UPSC

Previous Year Questions

3) It will enormously increase the growth and c) 1 and 3 size of and will enable it d) 2 and 3

to overtake China in the near future. 12. The sales tax you pay while purchasing Select the correct answer using the code a toothpaste is a given below: a) Tax imposed by the Central Government a) 1 only b) Tax imposed by the Central Government b) 2 and 3 only but collected by the State Government c) 1 and 3 only c) Tax imposed by the State Government but d) 1, 2 and 3 collected by the Central Government.

6.1.4 Other Indirect Taxes d) Tax imposed and collected by the State Government 9. The Standing Committee of State

Finance Ministers recommended in 13. Which one of the following is not a feature January, 2000 uniform rates across the of "Value Added Tax"? states in respect of a) It is a multi-point destination-based system a) Value-added tax of taxation b) Sales tax b) It is a tax levied on value addition at each c) Stamp duty and registration fees stage of transaction in the production- d) Agricultural income tax distribution chain

c) It is a tax on the final consumption of goods 10. Which one of the following is the or services and must ultimately be borne by correct statement? the consumer Service tax is a/an d) It is basically a subject of the Central a) Direct tax levied by the Central Government and the State Governments are Government. only a facilitator for its successful b) Indirect tax levied by the Central implementation

Government. 14. In India, the tax proceeds of which one c) Direct tax levied by the State Government. of the following as a percentage of gross d) Indirect tax levied by the State tax revenue has significantly declined Government. in the last five years? (2010)

11. Consider the following taxes: a) Service tax 1) Corporation tax b) Personal income tax 2) Customs duty c) Excise duty 3) Wealth tax d) Corporation tax

4) Excise duty

Which of these is/are indirect taxes?

a) 1 only

b) 2 and 4 www.laex.in Page No. 63 https://elearn.laex.in

www.laex.in

ECONOMY UPSC

Previous Year Questions

6.2 Expenditure 3) The upward trend in the ratio of Fiscal

Deficit to GDP in recent years has an 15. With reference to Union Budget, which adverse effect on private investments. of the following is / are covered under 4) Interest payments is the single largest Non-Plan Expenditure? component of the non-plan revenue 1) Defence expenditure expenditure of the Union Government. 2) Interest payments Which of these statements are correct? 3) Salaries and pensions a) 1, 2 and 3 4) Subsidies b) 1 and 4 Select the correct answer using the code c) 2, 3 and 4 given below. d) 1, 2, 3 and 4 a) 1 only

b) 2 and 3 only 18. India's increased from US c) 1, 2, 3 and 4 $98,158 million as at the end of March d) None 2000 to US $ 100,225 million as at the

6.3 Borrowing / Debt end of March 2001 due to increase in, 16. Consider the following statements: a) Multilateral and bilateral debt 1) Reserve Bank of India was nationalized on b) Rupee debt 26 January, 1950 c) Commercial borrowings and NRI deposits 2) The borrowing programme of the d) Borrowings from International Monetary Government of India is handled by the Fund

Department of Expenditure, Ministry of 19. A country is said to be in a debt trap if, Finance a) It has to abide by the conditionalities Which of the statements given above is/are imposed by the International Monetary correct? Fund a) 1 only b) It has to borrow to make interest payments b) 2 only on outstanding loans c) Both 1 and 2 c) It has been refused loans or aid by d) Neither 1 nor 2 creditors abroad

17. With reference to the Indian Public d) The World Bank charges a very high rate Finance, consider the following of interest on outstanding as well as new statements: loans

1) External liabilities reported in the Union 20. Consider the following: Budget are based on historical exchange 1. Market borrowing rates. 2. Treasury bills 2) The continued high borrowing has kept the 3. Special securities issued to RBI real interest rates high in the economy. Which of these is/are component(s) of internal debt? www.laex.in Page No. 64 https://elearn.laex.in

www.laex.in

ECONOMY UPSC

Previous Year Questions

a) 1 only Expenditure Total b) 1 and 2 Receipts less c) 2 only Borrowings and d) 1, 2 and 3 Interest payments Codes: 6.4 Deficit and Measures to a) A-3; B-1; C-2; D-4 Reduce Deficit b) A-4; B-3; C-2; D-1

21. There has been a persistent deficit c) A-1; B-3; C-2; D-4 budget year after year. Which d) A-3; B-1; C-4; D-2

action/actions of the following can be 23. A: Fiscal deficit is greater than taken by the Government to reduce the budgetary deficit. deficit? R: Fiscal deficit is the borrowing from 1) Reducing revenue expenditure the Reserve Bank of India plus other 2) Introducing new welfare schemes liabilities of Government to meet its 3) Rationalizing subsidies expenditure. 4) Reducing import duty a) Both, A and R, are true and R is the Select the correct answer using the code correct explanation of A given below. b) Both, A and R, are true but R is not the a) 1 only correct explanation of A b) 2 only c) If A is true but R is false, but A is false but c) 1 and 3 only R is true d) Neither 1 nor 2

d) Both A and R is false .

22. Match List I with List II and select the 24. In India, deficit financing is used for correct answer using the codes given raising resources for below the Lists:

a) Economic development List-I List-II b) Redemption of public debt A. Fiscal deficit 1. Excess of total c) Adjusting the balance of payments Expenditure over d) Reducing the foreign debt

Total Receipts

B. Budget deficit 2. Excess of Revenue 25. Consider the following statements: Expenditure over 1) Tax revenue as a percent of GDP of India Revenue Receipts has steadily increased in the last decade. C. Revenue deficit 3. Excess of Total 2) Fiscal deficit as a percent of GDP of India Receipts less has steadily increased in the last decade. Expenditure over Which of the statements given above is/are borrowings correct? D. Primary deficit 4. Excess of Total a) 1 only b) 2 only www.laex.in Page No. 65 https://elearn.laex.in

www.laex.in

ECONOMY UPSC

Previous Year Questions

c) Both 1 and 2 28. Which one of the following statements d) Neither 1 nor 2 is correct?

6.5 Budget and FRBM Act Fiscal Responsibility and Budget Management Act (FRBMA) concerns 26. Consider the following statement: 1) The Fiscal Responsibility and Budget a) Fiscal deficit only. Management (FRBM) Review Committee b) Revenue deficit only. Report has recommended a debt to GDP ratio c) Both fiscal deficit and revenue deficit. of 60% for the general (combined) d) Neither fiscal deficit nor revenue deficit.

government by 2023, comprising 40% for the 29. Which of the following is/are included Central Government and 20% for the State in the capital budget of the Government Governments. of India? 2) The Central Government has domestic 1. Expenditure on acquisition of assets like liabilities of 21% of GDP as compared to that roads, buildings, machinery, etc. of 49% of GDP of the State Governments. 2. Loans received from foreign governments 3) As per the Constitution of India, it is 3. Loans and advances granted to the States mandatory for a State to take the Central and Union Territories Government's consent for raising any loan if the former owes any outstanding liabilities to Select the correct answer using the code the latter. given below. Which of the statements given above is/are a) 1 only correct? b) 2 and 3 only a) 1 only c) 1 and 3 only b) 2 and 3 only d) 1, 2 and 3

c) 1 and 3 only 30. Consider the following statements : d) 1, 2 and 3 1. Most of India's external debt is owed by

27. Which one of the following was not governmental entities. stipulated in the Fiscal Responsibility 2. All of India's external debt is denominated and Budget Management Act, 2003? in US dollars. a) Elimination of revenue deficit by the end of Which of the statements given above is / are the fiscal year 2007-08 correct? b) Non-borrowing by the central government a) 1 only from Reserve Bank of India except under b) 2 only certain circumstances c) Both 1 and 2 c) Elimination of primary deficit by the end of d) Neither 1 nor 2 the fiscal year 2008-09 d) Fixing government guarantees in any financial year as a percentage of GDP

www.laex.in Page No. 66 https://elearn.laex.in

www.laex.in

ECONOMY UPSC

Previous Year Questions

6.6 Finance Commission a) It is a massive investment by the

Government in manufacturing sector to 31. With reference to the Fourteenth ensure the supply of goods to meet the Finance Commission, which of the demand surge caused by rapid economic following statements is/are correct? growth 1) It has increased the share of States in the b) It is an intense affirmative action of the central divisible pool from 32 percent to 42 Government to boost economic activity in percent. the country 2) It has made recommendations concerning c) It is Government's intensive action on sector- specific grants. financial institutions to ensure Select the correct answer using the code disbursement of loans to agriculture and given below. allied sectors to promote greater food a) 1 only production and contain food inflation b) 2 only d) It is an extreme affirmative action by the c) Both 1 and 2 Government to pursue its policy of d) Neither 1 nor 2

financial inclusion. 32. Which of the following is/are among the 34. Consider the following actions by the noticeable features of the Government: recommendations of the Thirteenth 1) Cutting the tax rates Finance Commission? 2) Increasing the government spending 1) A design for the Goods and Services Tax, 3) Abolishing the subsidies and a compensation package linked to In the context of economic recession, which adherence to the proposed design of the above actions can be considered a part 2) A design for the creation of lakhs of jobs in of the “fiscal stimulus” package? the next ten years in consonance with a) 1 and 2 only India's demographic dividend b) 2 only 3) Devolution of a specified share of central c) 1 and 3 only taxes to local bodies as grants d) 1, 2 and 3 Select the correct answer using the codes

given below: 6. Key and Explanations

a) 1 only b) 2 and 3 only 6.1 Taxation System c) 1 and 3 only 6.1.1 Direct Tax

d) 1, 2 and 3 1. Answer: B

6.7 Fiscal Stimulus Explanation: Capital gains tax is a tax imposed

33. Which one of the following statements on gains derived from the disposition of capital appropriately describes the "fiscal assets. It means an increase in the value of stimulus"? a capital asset (investment or real estate) that www.laex.in Page No. 67 https://elearn.laex.in

www.laex.in

ECONOMY UPSC

Previous Year Questions

gives it a higher worth than the purchase price. (i) above. more 36 months. It is the increase in the market value of an asset than 36 or set of assets, between two dates. months.

Capital assets mean properties of any kind held Hence increase in sales of a product does not by a person. Usually, capital gain is income constitute capital gains and hence, statement 1 derived from the sale of an investment. For is incorrect. example, suppose we purchased some asset at Educational Objective: To know about Rs 100 in the past and sold it at Rs 150 today; capital Gains tax. the capital gain is Rs 50. 2. Answer: A The tax rate depends upon whether long term or Explanation: In India, taxes are divided into six short term. Gains are considered long term if groups: the assets are held for more than 36 months. (A) Taxes Levied, Collected and Retained by This period may be reduced to more than 12 the Centre: months in the case of shares, specified These taxes are belonging to the centre securities/bonds and units of mutual funds. exclusively. In other words, no part of the The minimum holding period for long term proceeds of these taxes can be assigned to the capital gains in respect of shares of a company states. The following taxes fall under this that is not listed on a recognized stock exchange category: or units of a mutual fund is more than 36  Corporation Tax (Corporate tax) moths.  Customs Duties. Capital assets are classified as Long Term or  Surcharge on Income Tax. Short Term with reference to the period of  Taxes on capital value of assets of holding of the assets till it is transferred. The individual and companies. classification is made on the following basis.  Fees on matters of the Union list. Short Long Term Term (B) Taxes Levied by the Centre but Nature of Asset Capital Capital Collected and Appropriated by the Asset Asset States: (i) Shares in a The following taxes are included in this company or any category: other security listed Held for in a recognised stock  Stamp duties on bills of exchange, not Held for exchange in India or cheques, promissory notes and others. more more than a unit of a Unit than 12 12 months.  Excise duties on medicinal and toilet Trust of India or months. preparation containing alcohol. a unit of a mutual fund specified under (C) Taxes Levied and Collected by the section 10(23D). Centre but Assigned to the States: (ii) Assets other than Held for Held for This category includes the following duties and assets mentioned in not more than taxes: www.laex.in Page No. 68 https://elearn.laex.in

www.laex.in

ECONOMY UPSC

Previous Year Questions

 Duties on succession to property (other Under this category falls the excise duties than agricultural land). included in the Union list except those on  Estate duty on property (other than medicinal and toilet preparations. These are agricultural land). levied and collected by the centre. The net  Terminal taxes on goods and passengers proceeds of such duties can be paid to states carried by railways, sea and airways. out of the consolidated Fund of India only if the  Taxes on railway fares a freights. parliament so provides.  Taxes on transaction in stock exchanges (F) Taxes Levied and Collected and and future markets (other than stamp Retained by States: duties). The following taxes and duties exclusively  Taxes on the sale or purchase of belong to states. They are mentioned in the newspapers and taxes on advertisements State list. Every state is entitled to levy, collect published in them. and appropriate these taxes. The taxes are  Taxes on the sale or purchase of goods in  Duty on succession to agricultural land. course of inter-state trade or commerce  Estate duty on agricultural land. (other than newspaper).  Land revenue.  Taxes on the consignment of goods in the  Tax on agricultural income. course of inter-state trade or commerce.  Tax on land and buildings The net proceeds of these duties and taxes  Capitation taxes. are assigned to states in accordance with the  Tax on mineral rights. principles laid down by the parliament.  Tax on the consumption or sale of (D) Taxes Levied and Collected by the electricity. Centre and Compulsorily Distributed  Tax on vehicles. between the Centre and the States:  Tax on the sales and purchase of goods Taxes on income (other than agricultural income (other than newspaper) for e.g. Sales tax. and corporation tax) shall be levied and  Tolls collected by the centre but compulsorily  Tax on professions, trades and distributed between the centre and the states in employment. such manner as prescribed by the president on Hence taxes on stock exchanges and futures the recommendations of the Finance markets are Levied and Collected by the Centre Commission. The obligatory sharing of income but Assigned to the States: tax is provided by Article 270 of the Educational Objective: To know about taxes on Constitution. transactions.

(E) Taxes Levied and Collected by the 3. Answer: D Centre and may be distributed between Explanation: Direct Tax versus Indirect Tax the Centre and the States:

www.laex.in Page No. 69 https://elearn.laex.in

www.laex.in

ECONOMY UPSC

Previous Year Questions

As the incidence and impact of the above taxes Basis Direct Tax Indirect Tax viz, fringe benefit tax, interest tax and STT fall The tax that is on the same person, they fall under direct tax The tax that is levied by the Educational Objective: To know about levied by the government on difference between direct and indirect tax.

government one entity directly on the (Manufacturer of 4. Answer: B Meaning individuals or goods), but is Explanation: Increase in exemption limit to Rs. corporations are passed on to the 1lakh for general category tax payers. A higher called Direct final consumer by exemption limit of 1.5 lakh for widows and Taxes. the manufacturer. senior citizens was one of the recommendations The incidence of vijaykelkar committee in the year 2002. hence The incidence and impact of and impact of the Option B is in correct. Incidence the direct tax tax fall on Educational Objective: To know about direct fall on the same different persons. person. taxes in India

VAT, Service tax, 5. Answer: A Income Tax, GST, Excise duty, Explanation: In India, taxes are divided into six Example Corporation Tax entertainment tax groups: and Wealth Tax. and Customs Duty. (A) Taxes Levied, Collected and Retained by They are They are the Centre: Nature progressive in regressive in These taxes are belonging to the centre nature. nature. exclusively. In other words, no part of the Both Social and Only Economical. proceeds of these taxes can be assigned to the Economical. When an indirect Social objective tax is levied on a states. The following taxes fall under this of direct tax is product, both category: the distribution rich and poor  Corporation Tax (Corporate tax) of income. A must pay at the  Customs Duties. person earning same rate. A more should person earning  Surcharge on Income Tax. Objective contribute more 10 lakh a month  Taxes on capital value of assets of in the provision pays the same individual and companies. of public service tax on the Wheat by paying more purchase as the  Fees on matters of the Union list. tax. This person earning (B) Taxes Levied by the Centre but provision is also 3000 Re a month. Collected and Appropriated by the known as This principle is States: progressive called regressive taxation. taxation. The following taxes are included in this category: Not at all Impact Is inflationary. Inflationary.  Stamp duties on bills of exchange, cheques, promissory notes and others. www.laex.in Page No. 70 https://elearn.laex.in

www.laex.in

ECONOMY UPSC

Previous Year Questions

 Excise duties on medicinal and toilet tax is provided by Article 270 of the preparation containing alcohol. Constitution. (C) Taxes Levied and Collected by the (E) Taxes Levied and Collected by the Centre but Assigned to the States: Centre and may be distributed between This category includes the following duties and the Centre and the States: taxes: Under this category falls the excise duties  Duties on succession to property (other included in the Union list except those on than agricultural land). medicinal and toilet preparations. These are  Estate duty on property (other than levied and collected by the centre. The net agricultural land). proceeds of such duties can be paid to states  Terminal taxes on goods and passengers out of the consolidated Fund of India only if the carried by railways, sea and parliament so provides. airways.Taxes on railway fares a freights. (F) Taxes Levied and Collected and  Taxes on transaction in stock exchanges Retained by States: and future markets (other than stamp The following taxes and duties exclusively duties). belong to states. They are mentioned in the  Taxes on the sale or purchase of State list. Every state is entitled to levy, collect newspapers and taxes on advertisements and appropriate these taxes. The taxes are published in them.  Duty on succession to agricultural land.  Taxes on the sale or purchase of goods in  Estate duty on agricultural land. course of inter-state trade or commerce  Land revenue. (other than newspaper).  Tax on agricultural income.  Taxes on the consignment of goods in the  Tax on land and buildings course of inter-state trade or commerce.  Capitation taxes. The net proceeds of these duties and taxes are  Tax on mineral rights. assigned to states in accordance with the  Tax on the consumption or sale of principles laid down by the parliament. electricity. (D) Taxes Levied and Collected by the  Tax on vehicles. Centre and Compulsorily Distributed  Tax on the sales and purchase of goods between the Centre and the States: (other than newspaper) for e.g. Sales tax. Taxes on income (other than agricultural income  Tolls and corporation tax) shall be levied and  Tax on professions, trades and collected by the centre but compulsorily employment. distributed between the centre and the states in Hence Option A is correct where in income tax is such manner as prescribed by the president on levied, collected and distributed by centre. the recommendations of the Finance Educational Objective: To know about tax Commission. The obligatory sharing of income devolution to states.

www.laex.in Page No. 71 https://elearn.laex.in

www.laex.in

ECONOMY UPSC

Previous Year Questions

6.1.2 Tax Avoidance and Tax Evasion Option 1 Cereal grains hulled are exempted from GST, other options are taxed. 6. Answer : B Educational Objective: To know about Explanation: Base Erosion and Profit Shifting commodities under GST. (BEPS) is a tax avoidance strategy used by

multinational companies by exploiting gaps and 8. Answer : A mismatches in tax rules to artificially shift Explanation : statement 2 is incorrect GST has profits to low or no-tax locations. Here, profits nothing to do with forex reserves as it is an are shifted from jurisdictions that have high indirect tax imposed on the goods and services taxes (such as the United States and many within the country Western European countries) to jurisdictions Statement 3 is incorrect as GST will not help that have low (or no) taxes (so-called tax India to overtake China as the GDP difference havens). between the 2 countries is huge Educational Objective: To know about Base Statement 1 is correct as one of the objectives of Erosion and profit shifting. implementing GST is to replace multiple indirect

taxes with one single tax across the nation. 6.1.3 Indirect Tax Educational Objective: To know about various

7. Answer : A advantages of GST.

Explanation: 6.1.4 Other Indirect Taxes

9. Answer : B Explanation : The Empowered Committee of State Finance Ministers (EC) was originally set up on 17th July, 2000 by the Government of India with the Hon’ble State Finance Ministers of West Bengal, Karnataka, Madhya Pradesh, Maharashtra, Punjab, Uttar Pradesh, Gujarat, Delhi and Meghalaya as members with an objective to monitor the implementation of uniform floor rates of sales tax by States and Union territories, to monitor the phasing out of the sales-tax based incentive schemes, to decide milestones and methods of States to switch over to VAT and to monitor reforms in the Central Sales Tax system existing in the country. Educational Objective: To know about role of standing committee of state finance.

www.laex.in Page No. 72 https://elearn.laex.in

www.laex.in

ECONOMY UPSC

Previous Year Questions

10. Answer : B 13. Answer : D Explanation: Service tax is a tax levied by the Explanation: A value-added tax (VAT), is a type government on service providers on certain of tax that is assessed incrementally. It is levied service transactions, but is actually borne by on the actual transaction value of a product or the customers. It is categorized under Indirect service at each stage of production, distribution Tax and came into existence under the Finance or sale to the end consumer, except where a Act, 1994. Hence, Option B is correct. business is the end consumer which will reclaim Educational Objective: To know about service this input value. It has similarities and is often tax erroneously compared to a sales tax. The

11. Answer : B amount of VAT is decided by the state as Explanation: Indirect tax is a type of tax where percentage of the end-market price. As its name the incidence and impact of taxation does not suggests, value-added tax is designed to tax fall on the same entity. In the case of indirect only the value added by a business on top of the tax, the burden of tax can be shifted by the services and goods it can purchase from the taxpayer to someone else. Indirect tax has the market. effect to raising the price of the products on Hence option D is incorrect. which they are imposed. Customs duty, central Educational Objective: To know about Value excise, service tax and value added tax are added tax.

examples of indirect tax. Hence in the above 14. Answer : C four options customs duty and excise duty are Explanation: The following points highlight the classified under indirect taxes. two main sources of government revenue in Educational Objective: To know about various India. A. Tax Revenue: indirect taxes.  Union Excise Duties

12. Answer : D  Customs Explanation: Tax imposed and collected by the  Income Tax State Government Taxes on toothpaste come  Corporation Tax under CST Act , which is administered by State  Gift Tax Government. Sales Tax is paid to the Sales  Capital Gains Tax Tax Authority in the state from where the goods  Hotel Expenditure Tax are moved, i.e. the state from where the  Tax on Foreign Travel movement of goods begin. (This question B. Non-Tax Revenue: appeared in CSE prelims before implementation  Interest Receipts GST)  Surplus Profits of the Reserve Bank of Educational Objective: To know about tax India (RBI) collection on various commodities.  Currency, Coinage and Mint

 Railways  Profits of Public Enterprises www.laex.in Page No. 73 https://elearn.laex.in

www.laex.in

ECONOMY UPSC

Previous Year Questions

C.Other Non-Tax sources of Revenue: The borrowing programme of India is handled by In India, the tax proceeds of central excise as a Reserve Bank of India and not Finance Ministry. percentage of gross tax revenue has Hence statement 2 is incorrect. significantly declined in the last five years Educational Objective: To know about basic Educational Objective: To know about Tax information about RBI.

revenue of the government. 17. Answer : C

6.2 Expenditure Explanation: The External liabilities reported in the Union Budget are based on current 15. Answer : C exchange rates and not historical exchange Explanation: There are two components of rates. Hence statement 1 is incorrect. expenditure - plan and non-plan.Non-plan Educational Objective: To know about Indian revenue expenditure is accounted for by public finance.

interest payments, subsidies (mainly on food 18. Answer : A and fertilisers), wage and salary payments to government employees, grants to States and Explanation: The external debt of India is the total debt the country owes to foreign creditors. Union Territories governments, pensions, police, The debtors can be the Union government, state economic services in various sectors, other general services such as tax collection, social governments, corporations or citizens of India. services, and grants to foreign governments. The debt includes money owed to Non-plan capital expenditure mainly includes private commercial banks, foreign governments, defence, loans to public enterprises, loans to or international financial institutions such as States, Union Territories and foreign the International Monetary Fund (IMF) governments. and World Bank. Hence all the above are a part of non-plan Multilateral debt is the money India owes to expenditure international financial institutions such as the Asian Development Bank (ADB), Educational Objective: To know about the International Development Component of Expenditure Association (IDA), the International Bank for 6.3 Borrowing/Debt Reconstruction and Development (IBRD),

16. Answer: D the International Fund for Agricultural Explanation: The Reserve Bank of India was Development (IFAD) and others. Borrowing from nationalised with effect from 1st January, 1949 the International Monetary Fund (IMF) are not on the basis of the Reserve Bank of India included under multilateral debt, and are (Transfer to Public Ownership) Act, 1948. All instead classified separately under the IMF shares in the capital of the Bank were deemed head. transferred to the Central Government on Bilateral debt is the money India owes to foreign payment of a suitable compensation. Hence governments. statement 1 is incorrect

www.laex.in Page No. 74 https://elearn.laex.in

www.laex.in

ECONOMY UPSC

Previous Year Questions

Rupee denominated debt refers to that part of  Treasury-Bills: Zero coupon securities India's total external debt that is denominated that are issued at a discount and in India's domestic currency, the Rupee redeemed in face value at maturity. External commercial borrowing (ECBs) These are issued to address short term are loans in India made by non-resident lenders receipt-expenditure mismatches under in foreign currency to Indian borrowers. the auction program of the Government. Educational Objective: To know about These are primarily issued in three External debt of India. tenors, 91,182 and 364 day, 14 Day 19. Answer: B Treasury Bills. Explanation: A debt trap is a situation in which  Securities issued to International a borrower is led into a cycle of re-borrowing, or Financial Institutions: Securities rolling over, their loan payments because they issued to institutions viz. IMF, IBRD, are unable to afford the scheduled payments of IDA, ADB, IFAD etc. for India’s interests on the principal of a loan. These traps contributions to these institutions etc. are usually caused by high-interest rates and  Securities issued against ‘Small short terms. Savings’: All deposits under small Educational Objective: To know about Debt savings schemes are credited to the trap. National Small Savings Fund (NSSF).

20. Answer : D The balance in the NSSF (net of Explanation: Public Debt in India includes only withdrawals) is invested in special Internal and External Debt incurred by the Government securities. Central Government. Internal Debt includes  Market Stabilization Scheme (MSS) liabilities incurred by resident units in the Bonds: Governed by a MoU between the Indian economy to other resident units, GoI and the RBI, MSS was created to while External Debt includes liabilities incurred assist the RBI in managing its by residents to non-residents sterilization operations. GoI borrows The major components covered under Internal under this scheme from the RBI, while Debt are as follows: proceeds from such borrowings are  Dated Securities: Primarily fixed coupon maintained in a separate cash account securities of short, medium and long with the latter and is used only for term maturity which have a specified redemption of T-bills /dated securities redemption date. These are the single- raised under this scheme. most important component of financing Hence, all the 3 options are parts of internal the fiscal deficit of the Central debt Government (around 91 % in 2010-11) Educational Objective: To know about Internal with average maturity of around 10 debt.

years.

www.laex.in Page No. 75 https://elearn.laex.in

www.laex.in

ECONOMY UPSC

Previous Year Questions

6.4 Deficit and Measures to Reduce Revenue deficit = Revenue expenditure – Deficit Revenue Receipts

21. Answer : C Fiscal deficit is the distinction between the Explanation: Revenue Expenditure is that part government’s total expend and its total receipts of government expenditure that does not result and this excludes borrowing. in the creation of assets. Payment of salaries, Gross fiscal deficit = Total expenditure – wages, pensions, subsidies and interest fall in (Revenue receipts + Non-debt creating capital this category as revenue expenditure examples. receipts) Also, note that revenue expenses are incurred A primary deficit is the amount of money that by the government for its operational needs. the government requires to borrow apart from The Union government’s revenue expenditure the interest payments on the formerly borrowed comprises money spent on revenue account — loans the amount spent on running its elaborate Gross primary deficit = Gross fiscal deficit – machinery. All grants given to state Net interest liabilities governments and Union territories are also A budget deficit occurs when expenses exceed treated as revenue expenditure, even if some of revenue and indicate the financial health of a these grants may be used for the creation of country. Budget deficit = total receipts- total capital assets. expenditure In India, the payment of subsidies is also Hence the correct option is Option A included in revenue expenditure. The central Educational Objective: To know about various government pays subsidy under three major types of deficit related to government finance.

heads – food subsidy, fertiliser subsidy and fuel 23. Answer : C subsidy. Hence statement 1 and 3 are correct Explanation: Budget deficit = total receipts- Statement 2 is incorrect as introducing new total expenditure. Fiscal deficit is the sum total welfare schemes will further increase the of budget deficit + borrowings and other revenue expenditure liabilities. Hence, fiscal deficit is greater than Educational Objective: To know about Revenue budgetary deficit. Hence A is true but R is false expenditure. Educational Objective: To know about Budget 22. Answer: A deficit

Explanation: Deficit is the amount by which the 24. Answer: A expends in a budget overreach the earnings. The Explanation: Deficit spending is the amount by Government Deficit is the amount of money in which spending exceeds revenue over a the budget set by which the government expend particular period of time, also called simply overreaches the government earning amount. deficit, or budget deficit The revenue deficit mentions to the surplus of Purposes of Deficit Financing; government’s revenue expenditure over the revenue receipts.  To overcome the problem of lack of funds for speeding up the country's economic www.laex.in Page No. 76 https://elearn.laex.in

www.laex.in

ECONOMY UPSC

Previous Year Questions

development. Hence option 1 is revenue, expenditure and budgetary deficit correct trends.  Promote additional investment in the 6.5 Budget and FRBM Act

country to side away the adverse impacts 26. Answer : C of depression period of the country. Explanation :The total domestic liabilities of  To arrange fund for ensuring the holistic centre is 45.84% in 2017, hence statement 2 is development of the country. incorrect As per article 293 of the Constitution  To arrange fund for the unforeseen events of India, it is mandatory for a State to take the and arrange resources for wartime Central Government's consent for raising any expenditure. loan if the former owes any outstanding  To upgrade the infrastructure of the liabilities to the latter hence statement 3 is country so that the taxpayers of the correct country are convinced that the tax paid Following are the main recommendations of the by them is spent on the right things. NK Singh Committee. Educational Objective: to know about deficits (A). Public debt to GDP ratio should be of the budget and how they are financed considered as a medium-term anchor

25. Answer : D for fiscal policy in India. The combined debt-to-GDP ratio of the centre and states should be brought down to 60 per cent by 2023 (comprising of 40 per cent for the Centre and 20% for states) as against the existing 49.4 per cent, and 21per cent respectively. The above graph represents the tax revenue as (B). Fiscal deficit as the operating percentage of GDP. It is clear that 1st statement target: The Committee advocated fiscal is wrong deficit as the operating target to bring down public debt. For fiscal consolidation, the centre should reduce its fiscal deficit from the current 3.5% (2017) to 2.5% by 2023. Justifying the target of 2.5% to be realized in the next six years, the Committee observed that debt sustainability analysis (DSA) conducted for the central From the above graph it is clear that 2nd government suggests such a target (for statement is also wrong. fiscal deficit) will help to achieve the public Educational Objective: To have knowledge debt target of 40% for the centre by 2023. about the regular trends in the GDP, tax www.laex.in Page No. 77 https://elearn.laex.in

www.laex.in

ECONOMY UPSC

Previous Year Questions

(C). Revenue deficit target deficit target if some economic instabilities The Committee also recommends that the like an external crisis affects the Indian central government should reduce its economy. For example, the government revenue deficit steadily by 0.25 percentage has to spend more during the time of a (of GDP) points each year, to reach 0.8% recession and hence it need not restrict its by 2023, from a projected value of 2.3% in borrowing to keep the fiscal deficit target. 2017. Hence, the committee advocates The Committee advised government to countercyclical covers in fiscal policy while follow the golden rule here ie., not to following the FRBM. finance government’s day to day Here, the committee recommends fiscal expenditure through borrowings. Revenue flexibilities to go above or below the fiscal deficit implies financing of government’s deficit targets in the form of ‘escape day today activities from borrowings. clauses’. The Committee set 0.5% as Table: Fiscal roadmap for 2023 and the escape clause for fiscal deficit target. targets for the Centre (figures are as a Educational Objective: To know about FRBM percent of GDP) FD is fiscal deficit and RD Act.

is revenue deficit. 27. Answer: C Year Debt/GDP FD RD Explanation: Targets and fiscal indicators as 2017 49.4 3.5 2.3 per the FRBM Act 2023 38.7 2.5 0.80  Revenue deficit to be eliminated by the (D). Formation of Fiscal Council to advice 31st of March 2009. A minimum annual the government. reduction of 0.5% of GDP. The Committee advocated formation of  Fiscal Deficit to be brought down to at institutions to ensure fiscal prudence in least 3% of GDP by 31st of March 2008. accordance with the FRBM spirit. It A minimum annual reduction – 0.3% of recommended setting up an independent GDP. Fiscal Council. The Council will provide  Total Debt to be reduced to 9% of the GDP several advisory functions. It will forecast (a target increased from the original 6% key macro variables like real and nominal requirement in 2004–05). An annual GDP growth, tax buoyancy, commodity reduction of – 1% of GDP. prices. Similarly, it will do a monitoring  The purchase of government bonds by RBI role, besides advising about the use of must cease from 1 April 2006. escape clause and also specify a path of Hence, option C Elimination of primary deficit return. by the end of the fiscal year 2008-09 was not (E). Escape Clause to accommodate counter stipulated in FRBM 2003 cyclical issues: Educational Objective: To know about FRBM The NK Singh Committee points out that act. there are disadvantages with set fiscal www.laex.in Page No. 78 https://elearn.laex.in

www.laex.in

ECONOMY UPSC

Previous Year Questions

28. Answer : C Educational Objective: To know about capital Explanation: Fiscal Responsibility and Budget account.

Management Act (FRBM) concerns both fiscal 30. Answer : D and revenue deficit and hence option C is Explanation: From RBI press release 2018 it is correct. clear that Commercial borrowings continued to Educational Objective: To know about FRBM be the largest component of external debt with a act. share of 38.2 per cent, followed by NRI deposits

29. Answer : D (23.8 per cent) and short-term trade credit (19.0 Explanation: per cent). Most of India's external debt is not The capital budget is an account of assets as owed by governmental entities. Hence statement well as liabilities of the central government. 1 is wrong. Capital Receipts: All those receipts of the India’s external debt is held in multiple government which either creates liability or currencies, the largest of which is the United reduces financial asset are capital receipts. States dollar. As on 31 December 2017, 48.2% Examples: Market borrowings by the of the country’s debt was held in U.S. dollars. government from the public, Borrowings from The rest of the debt is held in Indian rupees the RBI, Borrowings from commercial banks or (37.3%), (5.7%), Japanese financial institutions through the sale of T- yen (4.6%), Euros (3.2%) and other currencies BILLS, loans received from foreign governments (1%). Hence statement 2 is also wrong. or international financial institutions, post office Educational objective: To learn about India’s savings, post office saving certificates and PSU’s external debt and the major developments Disinvestment. relating to India’s external debt.

Capital Expenditure: All those expenditures of 6.6 Finance Commission the government which either result in the 31. Answer : A creation of physical/financial assets or Explanation: Article 280 of the Constitution of reduction in financial liabilities. India requires the Constitution of a Finance Examples: Purchase of land, machinery, Commission every five years, or earlier The building and equipment’s; investment in shares; Finance Commission is required to recommend loans and advances by the central government the distribution of the net proceeds of taxes of to state governments and UTs the Union between the Union and the States Hence the capital budget includes (commonly referred to as vertical devolution); Expenditure on acquisition of assets like roads, and the allocation between the States of the buildings, machinery, etc, Loans received from respective shares of such proceeds (commonly foreign governments and Loans and advances known as horizontal devolution). The 14th granted to the States and Union Territories, finance commission recommended increasing therefore all the three statements are correct. the share of States in the central divisible pool from 32 percent to 42 percent, but it did not www.laex.in Page No. 79 https://elearn.laex.in

www.laex.in

ECONOMY UPSC

Previous Year Questions

make any sector specific recommendations, stimulus will see the central bank expanding hence statement 1 is correct and statement 2 is money supply or reducing the cost of money incorrect. (interest rates), to spur consumer spending. A Educational Objective: To know about finance fiscal stimulus entails the Government spending commission recommendations. more from its own coffers or slashing tax rates

32. Answer: A to put more money in the hands of consumers Explanation: One of the main recommendations During recession the Government can take of 13th finance commission was A design for the steps increasing government spending and Goods and Services Tax, and a compensation cutting tax rates, hence statement 1 and 2 are package linked to adherence to the proposed correct. design hence statement 1 is correct but it did Educational Objective: To know about Fiscal not recommend A design for the creation of stimulus.

lakhs of jobs in the next ten years in 7. Stock Market consonance with India's demographic dividend

and Devolution of a specified share of central 1. Which one of the following pairs is not taxes to local bodies as grants hence statement correctly matched? 2 and 3 are incorrect. a) Japan : Nikkei Educational Objective: To know about finance b) Singapore : Shcomp commission recommendations. c) UK : FTSE

6.7 Fiscal Stimulus d) USA : Nasdaq

33. Answer: B 2. Which of the following pairs are Explanation: A ‘stimulus’ is an attempt by correctly matched: policymakers to kickstart a sluggish economy 1) Dow Jones : New York through a package of measures. A monetary 2) Hang-Seng : Seoul stimulus will see the central bank expanding 3) FTSE-100 : London money supply or reducing the cost of money Select the correct answer using the codes (interest rates), to spur consumer spending. A given below: fiscal stimulus entails the Government spending a) 1, 2 and 3 more from its own coffers or slashing tax rates b) 2 and 3 to put more money in the hands of c) 1 and 2 consumersHence option B is Correct. d) 1 and 3

Educational Objective: To know about Fiscal 3. In India, which of the following is stimulus. regulated by the Forward Markets 34. Answer : A Commission? Explanation: A ‘stimulus’ is an attempt by a) Currency Futures Trading policymakers to kickstart a sluggish economy b) Commodities Futures Trading through a package of measures. A monetary c) Equity Futures Trading www.laex.in Page No. 80 https://elearn.laex.in

www.laex.in

ECONOMY UPSC

Previous Year Questions

d) Both Commodities Futures and Financial d) Directors

Futures Trading 8. Among the following major stock

4. What does S & P 500 relate to? exchanges of India, the exchange which a) Super computer recorded highest turnover during the b) A New technique in e-business year 2000-01 is c) A new technique in bridge building a) d) An index of stocks of large companies b) Calcutta Stock Exchange

5. What is Indo Next which was launched c) Delhi Stock Exchange in January, 2005? d) National Stock Exchange

a) A new scheme to promote Indian tourism 9. A rise in 'SENSEX' means b) A new scheme to promote export of Indian a) A rise in prices of shares of all companies handicrafts registered with Bombay Stock Exchange c) An association of the Non-Resident Indians b) A rise in prices of shares of all companies to organize Pravasi Bhartiya Divas every registered with National Stock Exchange year in India. c) An overall rise in prices of shares of group d) An alternative trading platform being of companies registered with Bombay promoted by the Bombay Stock Exchange Stock Exchange and Regional Stock Exchanges b) A rise in prices of shares of all companies

6. Consider the following statements belonging to a group of companies 1) Sensex is based on 50 of the most registered with Bombay Stock Exchange.

important stocks available on the Bombay 10. In the parlance of financial Stock Exchange (BSE). investments, the term 'bear' denotes 2) For calculating the Sensex, all the Sensex a) An investor who feels that the price of a stocks are assigned proportional particular security is going to fall weightage. b) An investor who expects the price of 3) New York Stock Exchange is the oldest particular shares to rise stock exchange in the world. c) A shareholder or a bondholder who has an Which of the statements given above is/are interest in a company, financial or correct? otherwise. a) 2 only d) Any lender whether by making a loan or b) 1 and 3 buying a bond. c) 2 and 3 11. In the context of global economy, which d) None

one of the following pairs is not 7. Debenture holders of a company are its correctly matched? a) Shareholders a) JP Morgan Chase : Financial b) Creditors Services c) Debtors www.laex.in Page No. 81 https://elearn.laex.in

www.laex.in

ECONOMY UPSC

Previous Year Questions

b) Roche Holding AG : Financial 7. Key and Explanations

Services c) WL Ross & Co. : Private 1. Answer : B Equity Firm Self Explanatory d) Warburg Pincus : Private Educational Objective: To know about various Equity Firm global stock markets

12. Gilt-edged market means 2. Answer: D a) bullion market Explanation: b) market of Government securities A stock exchange, securities exchange or c) market of guns bourse [note 1] is a facility where stockbrokers d) market of pure metals and traders can buy and sell securities, such as shares of stock and bonds and other financial 13. Resurgent India Bonds were issued in instruments. US dollar, Pound Sterling and Dow Jones INDEX a) An index is an indicator or measure of b) Deutsche Mark something, and in finance, it typically refers to a c) Euro statistical measure of change in a securities d) French Franc

market. 14. From the balance sheet of a company, it New York stock exchange (USA): it was is possible to founded in 1817 and is located on wall street in a) judge the extent of profitability of the New York. It is the largest stock exchange of the company world by market capitalisation. b) assess the profitability and size of the NASDAQ (USA): it is the second largest stock company exchange of the world founded in 1971 and c) determine the size and composition of the located in New York. assets and liabilities of the company Tokyo stock exchange (JAPAN): it is the third d) determine the market share, debts and largest stock exchange of the world and the assets of the company largest stock exchange in Asia. Nikkei INDEX

15. What does venture capital mean? Shanghai Stock Exchange (CHINA). SHCOMP a) A short-term capital provided to industries INDEX b) A long-term start-up capital provided to Hong Kong Stock Exchange: Hang-Seng new that the entrepreneurs INDEX c) Funds provided to industries at times of London stock exchange (UK)- FTSE-100 incurring losses INDEX d) Funds provided for replacement and Shenzhen Stock Exchange (CHINA): it is one renovation of industries of the two stock exchanges of China based in the city of Shenzhen, China.

www.laex.in Page No. 82 https://elearn.laex.in

www.laex.in

ECONOMY UPSC

Previous Year Questions

Toronto Stock Exchange (CHINA): it is the 4. Answer: D largest stock exchange in China Explanation: The S&P 500 Index or the Bombay Stock Exchange (BSE): it was founded Standard & Poor's 500 Index is a market- in 1875 and it is the first stock exchange of capitalization-weighted index of the 500 largest Asia. It is the 10th largest stock exchange of the U.S. publicly traded companies. world with the overall market capitalisation of The S&P is a float-weighted index, meaning USD 2.3 trillion in April 2018. The headquarters company market capitalizations are adjusted by of this stock exchange is located at Mumbai, the number of shares available for public Maharashtra, India.(SENSEX) trading. National Stock Exchange of India (NSE): it is The index is widely regarded as the best gauge the leading Stock Exchange of India, located in of large-cap U.S. equities. As a result, there are Mumbai, Maharashtra, India. It was established many funds designed to track the performance in 1992 and it's market capitalisation reached of the S&P. USD 2.27 trillion in April 2018.(NIFTY) Educational Objective: To have brief idea Educational Objective: To have knowledge about various indices of stock exchanges.

about various stock exchanges across the world 5. Answer: D and India. Also have brief idea about various Explanation: It is a new initiative launched by indices that are used in the stock exchanges Bombay stock exchange (BSE)

3. Answer : B  In do next is a separate trading platform Explanation: Established in 1953 under the for medium and small-cap companies, provisions of the Forward Contracts (Regulation) which are listed on regional stock Act, 1952, it consists of not less than two but exchanges and the BSE itself. not exceeding four members appointed by the  The project is aimed at improving liquidity Central Government. in such companies. Commodities Futures Trading is its major  The BSE Indo Next, a separate trading function platform under the BOLT trading system, Since futures traded in India are traditionally on is an initiative of the BSE and the food commodities, the agency was originally Federation of Indian Stock overseen by Ministry of Consumer Affairs, Food Exchanges.(FISE) and Public Distribution (India).  The FISE has 18 regional stock exchanges In September 2013, the commission as its members. responsibility was moved to the Ministry of  The members of the BSE and subsidiaries Finance to reflect that futures trading was of regional stock exchanges, which are becoming more and more a financial activity. members of the BSE, can trade on the Educational Objective: To have idea about Indo Next various organisations and commissions that Educational Objective: To know the initiatives regulate the stock market. taken by stock exchanges like BSE and NSE

www.laex.in Page No. 83 https://elearn.laex.in

www.laex.in

ECONOMY UPSC

Previous Year Questions

and also the government for effective  Debentures have a fixed rate of interest, functioning of stock exchanges and such interest amount is payable

6. Answer : D yearly or half-yearly Explanation: Amsterdam Stock Exchange, 1602  Debenture holders do not get any voting According to Guinness World Records, it is the rights. This is because they are not oldest in the world. instruments of equity, so debenture Currently known as Euronext Amsterdam, it holders are not owners of the company, was founded in 1602 by the Dutch East India only creditors Company to deal in its printed stocks and  The interest payable to these debenture bonds. It was the first exchange to formally start holders is a charge against the profits of trading in stocks the company. So these payments have to The BSE Sensex (also known as the S&P be made even in case of a loss. Bombay Stock Exchange Sensitive Index or Educational Objective: To have understanding simply the Sensex) is a free-float market- about shares, how and why are they issued, weighted stock market index of 30 well- how a company can raise the capital in share established and financially sound companies market.

listed on Bombay Stock Exchange 8. Answer : A Educational Objective: To have idea about Explanation: Bombay stock Exchange had history of evolution of Stock Exchanges across recorded the highest turnover during the year India and the world 2000-01

7. Answer: B Educational Objective: To have knowledge on Explanation: Debentures are written how stock exchanges are functioning and their instruments of debt that companies issue under performance of the year.

their common seal. They are similar to a loan 9. Answer : C certificate. Explanation: BSE Sensex  Debentures are issued to the public as a The BSE SENSEX (also known as the S&P contract of repayment of money Bombay Stock Exchange Sensitive Index or borrowed from them. simply the SENSEX) is a free-float market-  Debentures are instruments of debt, which weighted stock market index of 30 well- means that debenture holders become established and financially sound companies creditors of the company listed on Bombay Stock Exchange  They are a certificate of debt, with the date The BSE Sensex has been on a growth curve of redemption and amount of repayment since India opened up its economy in 1991. mentioned on it. This certificate is issued Most of its growth has occurred the 21st under the company seal and is known as century. a Debenture Deed 2001 BSE launched DOLLEX-30, a dollar-linked version of the SENSEX.

www.laex.in Page No. 84 https://elearn.laex.in

www.laex.in

ECONOMY UPSC

Previous Year Questions

If the Sensex value increases it means that there the sixth largest bank in the world by total is a general increase in the prices of shares assets. Hence pair 1 is correct. Whereas, if the Sensex decreases it means there Roche Holding AG is is a general decrease in the price of shares. a Swiss multinational healthcare company that NIFTY or Nifty 50 operates worldwide under two Nifty, also called NIFTY 50, is the market index divisions: Pharmaceuticals and Diagnostics. consisting of 50 well-established and financially Its holding company. Hence pair 2 is wrong. sound companies listed on National Stock WL Ross & Co is a private equity company Exchange of India (NSE). founded and based in New York by Wilbur Educational Objective: To have knowledge Ross in April 2000. The company focuses on about structure and functioning of stock investments in financially distressed companies exchanges like BSE and NSE with undervalued stocks, in the $100 to $200

10. Answer : A million range, usually—but not exclusively—in Explanation: The terms bull and bear market are the United States, Asia, Korea, Ireland, Japan, used to describe how stock markets are doing in France and China. Hence pair 3 is correct. general Warburg Pincus LLC is a New York- A bull market is a market that is on the rise and is based private equity firm focused on growth economically sound, while a bear market is a investing with offices in the United market that is receding, where most stocks are States, Europe, Brazil, China, Southeast declining in value. Asia and India. It has been a private equity Although some investors are "bearish," the investor since 1966. Hence pair 4 is correct. majority of investors are "bullish." The stock Educational objective: To learn about global market, as a whole, has always posted returns. economy and stock markets. A bear market is more dangerous to invest in, as 12. Answer : B many equities lose value. Since it is hard to time a Explanation :The gilt-edged market is the market market bottom, most investors withdraw their in government securities or the securities money from the markets and sit on cash until guaranteed (as to both principal and interest) by the trend reverses the government. The former include securities of Educational Objective: To have basic knowledge the Government of India and of the slate about the terminologies that are frequently used in governments; the latter are securities issued by the Stock market.

Local authorities (like city corporations, 11. Answer : B municipalities, and port trusts) and autonomous Explanation: JPMorgan Chase & Co. is an government undertakings like development hanks, American multinational investment state electricity boards, etc. bank and financial services holding The term gilt-edged means ‘of the best quality”. It company headquartered in New York City. has come to be reserved for government securities JPMorgan Chase is ranked by S&P Global as as they do not suffer from the risk of default. the largest bank in the United States and Besides, government securities are highly liquid,

www.laex.in Page No. 85 https://elearn.laex.in

www.laex.in

ECONOMY UPSC

Previous Year Questions

as they can be easily sold in the market at their perspective. This capital is known as venture going market price. capital and the investors are called venture Educational objective: To learn basics about gilt- capitalists. edged market. Such investments are risky as they are illiquid, but

13. Answer : B are capable of giving impressive returns if invested Explanation: The Resurgent India Bonds (RIBs) in the right venture. are bank instruments issued by SBI representing Educational objective: To learn about venture foreign currency denominated deposits in the form capital and its features.

of promissory notes. Offered to Non-Resident Indians and Overseas Corporate Bodies. 8. Agriculture & Food Management Bonds being denominated in foreign currency i.e., 8.1 Procurement Price Or Support Price in US dollar, Pound Sterling and Deutsche Mark,

bonds are free from forex risk with a tenure of 5 1. The economic cost of food grains to the years. Food Corporation of India is Minimum Educational objective: To learn basics about Support Price and bonus (if any) paid to Resurgent India Bonds (RIBs). the farmers plus

14. Answer : C a) Transportation cost only Explanation : The balance sheet is one of the b) Interest cost only three fundamental financial statements and is key c) Procurement incidentals and distribution to both financial modeling and accounting. The cost balance sheet displays the company’s total assets, d) Procurement incidentals and charges for and how these assets are financed, through either godowns

debt or equity. 2. Consider the following: It can also be referred to as a statement of net 1. Areca nut 2. Barley worth, or a statement of financial position. The 3. Coffee 4. Finger millet balance sheet is based on the fundamental 5. Groundnut 6. Sesamum equation: Assets = Liabilities + Equity. 7. Turmeric This means that assets, or the means used to The Cabinet Committee on Economic Affairs, operate the company, are balanced by a company's has announced the Minimum Support Price financial obligations, along with the equity for which of the above? investment brought into the company and its retained earnings. a) 1, 2, 3 and 7 only Educational objective: To learn basics on balance b) 2,4,5, and 6 only sheet and its significance. c) 1,3,4, 5 and 6 only

15. Answer : B d) 1,2, 3, 4,5, 6 and 7

Explanation: Start up companies with a potential 3. Consider the following statements: to grow need a certain amount of investment. 1) The Commission for Agricultural Costs Wealthy investors like to invest their capital in and Prices recommends the Minimum such businesses with a long-term growth Support Prices for 32 crops. www.laex.in Page No. 86 https://elearn.laex.in

www.laex.in

ECONOMY UPSC

Previous Year Questions

2) The Union Ministry of Consumer Affairs, Which of the statements given above is/are Food and Public Distribution has launched correct? the National Food security Mission. a) 1 and 2 Which of the statements given above is/are b) 2 only correct? c) 1 and 3 a) 1 only d) 3 only

b) 2 only 7. Consider the following statements: c) Both 1 and 2 1) The Union Government fixes the Statutory d) Neither 1 nor 2 Minimum Price of sugarcane for each

4. The prices at which the government sugar season. purchases food grains for maintaining 2) Sugar and sugarcane are essential the public distribution system and for commodities under the Essential building up buffer-stock is known as, Commodities Act. a) Minimum support prices Which of the statements given above is/are b) Procurement prices correct? c) Issue prices a) 1 only d) Ceiling prices b) 2 only

5. The Fair and Remunerative Price (FRP) c) Both 1 and 2 of sugarcane is approved by the d) Neither 1 nor 2

a) Cabinet Committee on Economic Affairs 8.2 Trends in Indian Agriculture b) Commission for Agricultural Costs and Production

Prices c) Directorate of Marketing and Inspection, 8. With reference to the cultivation of Ministry of Agriculture. Kharif crops in India in the last five d) Agricultural Produce Market Committee. years, consider the following

6. Consider the following statements: statements: 1) Regarding the procurement of food grains, 1) Area under rice cultivation is the highest. Government of India follows a 2) Area under the cultivation of jowar is more procurement target rather than an open- than that of oilseeds. ended procurement policy 3) Area of cotton cultivation is more than that 2) Government of India announces minimum of sugarcane. support prices only for cereals 4) Area under sugarcane cultivation has 3) For distribution under Targeted Public steadily decreased. Distribution System (TPDS), wheat and Which of the statements given above are rice are issued by the Government of India correct? at uniform Central issue prices to the a) 1 and 3 only States/Union Territories b) 2, 3 and 4 only

www.laex.in Page No. 87 https://elearn.laex.in

www.laex.in

ECONOMY UPSC

Previous Year Questions

c) 2 and 4 only a) 1 only d) 1,2, 3 and 4 b) 2 only

9. Which of the following pairs about c) Both 1 and 2 India's economic indicator and d) Neither 1 nor 2

agricultural production (all in rounded 12. Consider the following statements: figures) are correctly matched? India continues to be dependent on imports to 1) GDP per capita (current prices): Rs 37,000 meet the requirement of oilseeds in the country 2) Rice : 180 million tons because 3) Wheat : 75 million tons 1) Farmers prefer to grow food grains with Select the correct answer using the code highly remunerative support prices given below: 2) Most of the cultivation of oilseed crops Codes: continues to be dependent on rainfall a) 1, 2 and 3 3) Oils from the seeds of tree origin and rice b) 1 and 2 only bran have remained unexploited c) 2 and 3 only 4) It is far cheaper to import oilseeds than to d) 1 and 3 only cultivate the oilseed crops

10. Consider the following statements: Which of the statements given above are 1) India ranks first in the world in fruit correct? production. a) 1 and 2 2) India ranks second in the world in the b) 1,2 and 3 export of tobacco c) 3 and 4 Which of these statements is/are correct? d) 1,2,3 and 4

a) Only 1 8.4 Inflation

b) Only 2 13. India has experienced persistent and c) Both 1 and 2 high food inflation in the recent past. d) Neither 1 nor 2 What could be the reasons?

8.3 Edible Oil 1) Due to a gradual switchover to the cultivation of commercial crops, the area 11. Consider the following statements: under the cultivation of food grains has 1) The quantity of imported edible oils is steadily decreased in the last five years by more than the domestic production of about 30%. edible oils in the last five years. 2) As a consequence of increasing incomes, 2) The Government does not impose any the consumption patterns of the people customs duty on all the imported edible have undergone a significant change. oils as special case. 3) The food supply chain has structural Which of the statements given above is/are constraints. correct

www.laex.in Page No. 88 https://elearn.laex.in

www.laex.in

ECONOMY UPSC

Previous Year Questions

Which of the statements given above are d) Identifying the entrepreneurs in villages correct? and providing them technology and a) 1 and 2 only finance to set up companies

b) 2 and 3 only 16. National Agricultural Insurance Scheme c) 1 and 3 only replacing Comprehensive Crop d) 1,2 and 3 Insurance Scheme was introduced in

8.5 Schemes/Programmes Related to the year Agriculture a) 1997

b) 1998 14. What is/are the advantage/advantages c) 1999 of implementing the 'National d) 2000 Agriculture Market' scheme? 17. With reference to 'Pradhan Mantri Fasal 1) It is a pan-India electronic trading portal Bima Yojana', consider the following for agricultural commodities. statements: 2) It provides the farmers access to 1) Under this scheme, farmers will have to pay nationwide market, with prices a uniform premium of two percent for any commensurate with the quality of their crop they cultivate in any season of the year. produce. 2) This scheme covers post-harvest losses Select the correct answer using the code arising out of cyclones and unseasonal rains. given below: Which of the statements given above is / are a) 1 only correct? b) 2 only a) 1 only c) Both 1 and 2 b) 2 only d) Neither 1 nor 2 c) Both 1 and 2

15. Which one of the following best d) Neither 1 nor 2

describes the main objective of ‘Seed 18. Consider the following statements: Village’ Concept? 1) The Accelerated Irrigation Benefits a) Encouraging the farmers to use their own Programme was launched during 1996-97 farm seeds and discouraging them to buy to provide loan assistance to poor farmers. the seeds from others 2) The Command Area Development b) Involving the farmers for training in quality Programme was launched in 1974-75 for seed production and thereby to make the development of water- use efficiency. available quality seeds to others at Which of the statements given above is / are appropriate time and affordable cost correct? c) Earmarking some villages exclusively for a) 1 only the production of certified seeds b) 2 only c) Both 1 and 2 d) Neither 1 nor 2 www.laex.in Page No. 89 https://elearn.laex.in

www.laex.in

ECONOMY UPSC

Previous Year Questions

19. An objective of the National Food 22. Consider the following statements: Security Mission is to increase the 1) The loans disbursed to farmers under production of certain crops through Kisan Credit Card Scheme are covered area expansion and productivity under Rashtriya Krishi BeemaYojna of Life enhancement in sustainable manner in Insurance Corporation of India the identified districts of the country. 2) The Kisan Credit Card holders are What are those crops? provided personal accident insurance of Rs a) Rice and wheat only 50,000 for accidental death and Rs. b) Rice, wheat and pulses only 25,000 for permanent disability c) Rice, wheat, pulses and oil seeds only Which of the statements given above is/are d) Rice, wheat, pulses, oil seeds and correct? vegetables a) 1 only

8.6 Agricultural Credit b) 2 only 20. In India, which of the following have c) Both 1 and 2 the highest share in the disbursement d) Neither 1 nor 2

of credit to agriculture and allied 8.7 Agriculture Inputs

activities? 23. The substitution of steel for wooden a) Commercial Banks ploughs in agricultural production is an b) Cooperative Banks example of c) Regional Rural Banks a) labour-augmenting technological progress d) Microfinance Institutions

b) capital-augmenting technological progress 21. The farmers are provided credit from a c) capital-reducing technological progress number of sources for their short and d) None of the above long-term needs. The main sources of 24. In the context of food and nutritional credit to the farmers include security of India, enhancing the 'Seed a) The Primary Agricultural Cooperative Replacement Rates' of various crops Societies, commercial banks, RRBs and helps in achieving the food production private money lenders targets of the future. But what is/are b) The NABARD, RBI, commercial banks and the constraint/ constraints in its private money lenders wider/greater implementation? c) The District Central Cooperative Banks 1) There is no National Seeds Policy in place. (DCCBs), the lead banks, IRDP and JRY 2) There is no participation of private sector d) The Large Scale Multi-purpose Adivasis seed companies in the supply of quality Programme, DCCB, IFFCO and seeds of vegetables and planting materials commercial banks. of horticultural crops.

www.laex.in Page No. 90 https://elearn.laex.in

www.laex.in

ECONOMY UPSC

Previous Year Questions

3) There is a demand-supply gap regarding 8. Key and Explanations

quality seeds in case of low value and high volume crops. 8.1 Procurement Price Or Support Price Select the correct answer using the code 1. Answer: C given below. Explanation: The economic cost of food grains a) 1 and 2 is the total expenditure incurred by the b) 3 only government to procure the food grains and c) 2 and 3 ensure that these food grains reach the d) None beneficiaries. ECFG= MSP + WAREHOUSING/ STORAGE 8.8 Agriculture Marketing COST +TRANSPORTATION COST + 25. In India, markets in agricultural DISTRIBUTION COST + BONUS (If any, products are regulated under the announced by State Government). a) Essential Commodities Act, 1955 Hence, ECFG is MSP and Bonus (given in the b) Agricultural Produce Market Committee question) + procurement incidentals i.e., storage Act enacted by States and transportation cost + distribution cost. c) Agricultural Produce (Grading and Option B is not correct as interest in not a part Marking) Act, 1937. of ECFG d) Food Products Order, 1956 and Meat and Option D is not correct as procurement Food Products Order, 1973. incidentals include storage cost

26. Consider the following statements: Educational Objective: to know about the 1) The Standard Mark of Bureau of Indian working of Public Distribution System in India. Standards (BIS) is mandatory for Also about procurement and distribution automotive tyres and tubes. channels in PDS. 2) AGMARK is a quality Certification Mark 2. Answer : B issued by the Food and Agriculture Explanation: The CACP (Commission on Organisation (FAO). Agriculture Cost and Prices) under Ministry of Which of the statements given above is/are Agriculture and Farmer’s welfare, recommends correct? MSP on 23 crops and this is approved by CCEA a) 1 only (cabinet committee on economic affairs) for a b) 2 only particular year. These 23 crops include seven c) Both 1 and 2 cereal crops (paddy, wheat, jowar, bajra, maize, d) Neither1 nor 2 ragi and barley), five pulse crops (gram, tur, moong, urad and lentil), seven oilseeds (groundnut, sunflower seed, soyabean, rapeseed, mustard, safflower, nigerseed and seasmum), copra (dried coconut), cotton, raw

www.laex.in Page No. 91 https://elearn.laex.in

www.laex.in

ECONOMY UPSC

Previous Year Questions

jute and sugarcane. Out of these 23 crops, all pulses and 3 million tonnes of coarse cereals by crops are procured at MSP from FCI and only the end of XII Plan sugarcane is procured at FRP (Fair Educational Objective: to know about how Remunerative Price) from the sugar mill owners. MSPs are calculated and procedure how it is So, this question could’ve been solved by approved by the cabinet. Also, the crops that are elimination method, where Coffee is not in the covered under MSP.

list, #3 is wrong, by elimination we get correct 4. Answer: B answer B. Explanation: Option A. Minimum Support Price Educational Objective: To know about how is the price at which government purchases MSPs are calculated and procedure how it is crops for the farmers, to safeguard the interests approved by the cabinet. Also the crops that are of the farmers and MSPs are announced at the covered under MSP. beginning of the sowing season for certain crops

3. Answer: D based on the recommendations of the Explanation: Option 1 is incorrect Commission for Agricultural Costs and Prices The CACP (Commission On Agriculture Cost (CACP). And Prices) under Ministry of Agriculture and Option B; It is the price at which govt Farmer’s welfare, recommends MSP on 23 crops purchases the crop after harvesting, the main and this is approved by CCEA (cabinet difference between Procurement Price and MSP committee on economic affairs) for a particular is that MSP is declared before sowing while PP is year. declared after harvesting. Procurement prices Option 2 is incorrect are higher than MSP; hence option B is the The NFSM was launched under ministry of correct answer. Agriculture and Farmer’s welfare and not by Option C ; The price at which the procured and ministry of consumer affairs food and public buffer stock food grains are provided through distribution. National Food Security Mission the PDS is called an issue price, in other words, was launched in 2007-08 to increase the the Price at which the FCI sells the food grain production of rice, wheat and pulses by 10, 8 for PDS consumption is known as Issue Price. and 2 million tonnes, respectively by the end of Educational Objective: to know about the XI Plan through area expansion and procurement channels for Public Distribution productivity enhancement; restoring soil fertility System and the price spent to procure the food.

and productivity; creating employment 5. Answer: A opportunities; and enhancing farm level Explanation: FRP is the minimum price at economy. The Mission is being continued during which sugarcane is to be purchased by sugar 12th Five Year Plan with a new target of mills from the farmers. FRP is approved by additional production of 25 million tonnes of CCEA (cabinet committee on economic Affairs) food grains comprising of 10 million tonnes rice, on the basis of recommendations made by CACP 8 million tonnes of wheat, 4 million tonnes of (commission on agricultural costs and prices) an

www.laex.in Page No. 92 https://elearn.laex.in

www.laex.in

ECONOMY UPSC

Previous Year Questions

attached office of union ministry of agriculture (TPDS) is operated under the joint responsibility and farmers welfare. Sugarcane’s FRP is of the Central and the State/Union Territory determined under sugarcane (control) order, (UT) Governments. Central Government is 1966 which is uniformly applicable all over the responsible for procurement, allocation and country. transportation of foodgrains upto the designated Option C: The Directorate of Marketing and depots of the Food Corporation of India where in Inspection (DMI), is an attached Office under these are issued by the Government of India at Ministry of Agriculture &Farmers Welfare , uniform Central issue prices to the which implements agricultural marketing States/Union Territories. The operational policies and programmes for the integrated responsibilities for allocation and distribution of development of marketing of agricultural and foodgrains within the States/UTs, identification other allied produce in the country with a view of eligible beneficiaries, issuance of ration cards to safeguard the interests of farmers as well as to them and supervision over and monitoring of the consumers. It maintains a close liaison functioning of Fair Price Shops (FPSs) rest with between the Central and the State the concerned State/UT Governments. Governments. Educational Objective: to know about the Option D: An Agricultural Produce Market working of Public Distribution System in India. Committee (APMC) is a marketing Also, about procurement and distribution board established by a state channels in PDS.

government in India to ensure farmers are 7. Answer: A safeguarded from exploitation by large retailers, Explanation: Option A: The government fixes as well as ensuring the farm to retail price the Statutory Minimum Price (fixed spread does not reach excessively high levels remunerative price) of sugarcane for each sugar Educational Objective: To know why and how season, FRP is the minimum price at which the Fair and Remunerative Price is given for sugarcane is to be purchased by sugar mills Sugarcane and issues associated with it. from the farmers. FRP is approved by CCEA

6. Answer : D (cabinet committee on economic Affairs) on the Explanation: Option B: the Government of basis of recommendations made by CACP India announces MSP for 23 crops which (commission on agricultural costs and prices) an include seven cereal crops (paddy, wheat, jowar, attached office of union ministry of agriculture bajra, maize, ragi and barley), five pulse crops and farmers welfare (gram, tur, moong, urad and lentil), seven Option B: The Essential Commodities Act (ECA) oilseeds (groundnut, sunflower seed, soyabean, was enacted by the Central Government in 1955 rapeseed, mustard, safflower, nigerseed and to control and regulate trade and prices of seasmum), copra (dried coconut), cotton, raw commodities declared essential under the Act. jute and sugarcane. The Act empowers the Central and state Hence option B is wrong. governments concurrently to control production, Option C: Targeted Public Distribution System www.laex.in Page No. 93 https://elearn.laex.in

www.laex.in

ECONOMY UPSC

Previous Year Questions

supply and distribution of certain commodities Statement 2: the area under jowar cultivation in view of rising prices. is 2.23 mn hectares which is much less than The essential commodities in India are as that of oil seeds which is around 11.25 mn follows: hectares. Hence statement 2 is incorrect  Cattle fodder, including oilcakes and other Statement 3: statement is correct as the area concentrates; under cotton cultivation is 11.97 mn hectares  Coal including coke and other derivatives; and that of sugarcane is 4.88 mn hectares.  Component parts and accessories of Statement 4: statement 4 is incorrect as the automobiles; area under sugarcane cultivation has not  Cotton and woollen textiles;'. steadily decreased.  Drugs. Educational Objective: To know about the  Foodstuffs, including edible oilseeds and trends in crop production and crop patterns oils; being followed in India

 Iron and steel, including manufactured 9. Answer: D products of iron and steel; Explanation: Statement 1: The per capita GDP  Paper, including newsprint, paperboard (current prices) of india is Rs. 1,26,406, hence and straw board; statement one is wrong  Petroleum and petroleum products; Statement 2: The rice production in 2017-18 is 112.91 mntonnes, hence statement 2 is  Raw cotton, whether ginned or unginned, incorrect and cotton seed; Statement 3: The production of wheat in 2017-  Raw jute; 18 is 99.7 mntonnes, hence statement 3 is The list of commodities declared “essential” incorrect under ECA 1955, is reviewed from time to time NOTE: All the above figures as per current in the light of changed in economic situation estimates and hence do not match with the and in regard to their production and supply figures of the year in which the above question Educational Objective: to know about appeared in CSE Prelims legislations relating to food security in India like Educational Objective: the relevance of such food security act, Essential Commodities Act questions involving the numbers is on a decline (ECA) etc. trend. However, one need to have a rough idea 8.2 Trends in Indian Agriculture about such data.

Production

10. Answer: D 8. Answer: A Explanation: Statement 1: India’s fruit export Explanation: Statement 1: The area under rice for the year 2018-19 stood at 4,817 crores. cultivation in kharif season is around 39.54 mn India is the second largest producer of fruits hectares which is the highest, hence statement and vegetables in the world after China, one is correct contributing 12.6% and 14.0% of the total world

www.laex.in Page No. 94 https://elearn.laex.in

www.laex.in

ECONOMY UPSC

Previous Year Questions

production of fruits and vegetables respectively. Educational Objective: To know about the China tops the list of fruit production followed regular production trends of edible oil in India. by India, Brazil, USA, Spain, Mexico, etc. Hence Also, about the exports and imports of edible oil.

statement 1 is incorrect 12. Answer : B Statement 2: The top exporters of Tobacco Explanation: In India, about 60% of total net are Brazil ($1.99B), the United tates sown area comes under rainfed lands. Rainfed ($997M), Zimbabwe ($977M), India ($657M) crops account for 48 percent area under food and China ($654M). The top importers crops and 68 percent under non-food crops. are Belgium-Luxembourg ($1.36B), China India ranks first among the rainfed agricultural ($1.21B), Germany ($792M), the United States countries of the world in terms of both extent ($653M) and Russia ($600M). Hence, statement and value of produce. The reduction in rainfall 2 is also incorrect. and lack of seed preservation technology are the Educational Objective: to know about the main causes for India to be dependent on basic facts relating to the agricultural imports to meet the requirement of oilseeds in production in India. the country. 8.3 Edible Oil Educational Objective: To know about the

11. Answer : A challenges being faced by India in the Explanation: Statement 1: Edible oil imports production and processing of edible oils

and domestic production (in lakh tonnes) 8.4 Inflation 20 13. Answer : B 12015 2016- 2017- Explanation: Statement 1 is incorrect as the 4 - Year 2013-24 1 1 - 1 average area of foodgrains in last 5 years is 7 8 1 6 124.52 million hectares and the average area of 5 commercial crops in last 5 years is 17.76 million 13 hectares. Statement 2 is correct, A factor, which 8 has often been argued as driving food inflation 148. 153.1 119.4 Imports 109.76 . 5 7 4 in India, relates to rising income and 5 diversification of diets raising the demand for 3 high-value food products, and thereby adding to Domesti 107.4 110.1 inflationary pressures productio 107.62 98 91.8 9 0 n Hence, statement 1 is correct Statement 2: the Government imposes customs duty ranging between 35% to 54% on imports of edible oils and hence, Statement 2 is incorrect

www.laex.in Page No. 95 https://elearn.laex.in

www.laex.in

ECONOMY UPSC

Previous Year Questions

The states can administer agriculture marketing as per their agri-marketing regulations, under which, the State is divided into various market areas and each market area is administered by a separate APMC which will impose its own marketing regulation that include fees. NAM addresses these challenges by-  Creating a unified market through online

Statement 3 is correct : The food supply chain trading platform across the country (at has various structural constraints like state and national level). inefficient marketing systems, weak storage  Promoting uniformity. infrastructure, stagnant productivity,  Rationalizing procedures across the unavailability of credit, middle men etc unified markets. Educational Objective: to have knowledge  Eliminating information irregularity about the fundamentals of Inflation and various between the buyers and sellers and factors affecting the Inflation in the country. promoting real time price finding on the

8.5 Schemes/Programmes Related to basis of actual demand and supply.  Promotes transparency in auction process. Agriculture

 Providing an accessible nationwide market 14. Answer: C for the farmer with equal prices for the Explanation: The National Agriculture Market quality of his produce. (NAM) is a pan-India electronic trading portal,  Availability better quality produces at more which links the existing Agricultural Produce reasonable prices and online payment to Market Committee (APMC) mandis across the the consumer. country to form a unified national market for Educational Objective: To know about the agricultural commodities. various initiatives of the government to improve The NAM portal is a single window service for the agricultural marketing in India.

any information and services related to APMC 15. Answer : B that includes: Explanation: A village, wherein trained group of  Commodity arrivals and prices farmers are involved in production of seeds of  Buy and sell trade offers various crops and cater to the needs of  Provision to respond to trade offers, among themselves, fellow farmers of the village and other services farmers of neighbouring villages in appropriate The NAM reduces the transaction costs and time and at affordable cost is called "a seed information irregularity even when the village" agriculture produce continues to flow through Objectives: the mandis.  Increasing the seed production

www.laex.in Page No. 96 https://elearn.laex.in

www.laex.in

ECONOMY UPSC

Previous Year Questions

 Increasing the seed replacement rate present, 10% subsidy on premium is available  Organizing seed production in cluster (or) to small & marginal farmer compact area replacing existing local Educational Objective: The relevance of such varieties with new high yielding varieties questions being asked is reducing. However, one  Self sufficiency and self reliance of the should have rough idea about the schemes and village. To meet the local demand, timely their implementation year.

supply. 17. Answer: B The present programme of seed village scheme Explanation: The government of India in April is having two phases: 2016, launched the Pradhan Mantri Fasal Bima 1. Seed production of different crops: The area Yojana (PMFBY) after scraping down the earlier which is suitable for raising a particular insurance schemes viz. Modified National crop will be selected, and raised with single Agricultural Insurance Scheme (MNAIS), variety of a kind. Weather-based Crop Insurance scheme and the 2. Establishing seed processing unit: If the National Agriculture Insurance Scheme (NAIS). seeds are not processed and handled Premium: 2% to be paid by farmers for Kharif properly, all the past efforts in production crops, and 1.5% for Rabi crops. The premium may be lost. Thus seed processing and for annual commercial and horticultural crops packaging is very important aspect in seed will be 5% hence statement 1 is incorrect production. Risks covered under the scheme: Educational Objective: to have knowledge Yield Losses (standing crops, on notified area about various initiatives to improve the seed basis). Comprehensive risk insurance is quality and supply to the farmers to improve the provided to cover yield losses due to non- agricultural produce. preventable risks, such as Natural Fire and

16. Answer: C Lightning, Storm, Hailstorm, Cyclone, Typhoon, Explanation: National Agricultural Insurance Tempest, Hurricane, Tornado. Risks due to Scheme was introduced in the rabi season of Flood, Inundation and Landslide, Drought, Dry 1999-00 replacing Comprehensive Crop spells, Pests/ Diseases also will be covered. Insurance Scheme, to protect the farmers In cases where majority of the insured farmers against losses suffered by them due to crop of a notified area, having intent to sow/plant failure on account of natural calamities. The and incurred expenditure for the purpose, are scheme is currently implemented by Agriculture prevented from sowing/planting the insured Insurance Company of India (AICIL). crop due to adverse weather conditions, shall be The scheme is available to all the farmers, eligible for indemnity claims upto a maximum of loanee and non-loanee, irrespective of size of 25 per cent of the sum-insured. their holding. The scheme covers all food crops In post-harvest losses, coverage will be available (cereals, millets and pulses) and oil seeds and up to a maximum period of 14 days from Annual commercial/ horticultural crops. At harvesting for those crops which are kept in “cut & spread” condition to dry in the field. www.laex.in Page No. 97 https://elearn.laex.in

www.laex.in

ECONOMY UPSC

Previous Year Questions

For certain localized problems, Loss/damage and pulses by 10 million tonnes, 8 million resulting from occurrence of identified localized tonnes, and 2 million tonnes respectively by the risks like hailstorm, landslide, and Inundation end of the Eleventh Plan (viz. March 2012). The affecting isolated farms in the notified area Mission aims to increase production through would also be covered. area expansion and productivity; create Educational Objective: to know about the employment opportunities; and enhance the insurance schemes of the government to assist farm-level economy (i.e. farm profits) to restore the farmers in reducing the burden of losses confidence of farmers. The approach is to bridge because of calamities. the yield gap in respect of these three crops

18. Answer: B through dissemination of improved technologies Explanation: Statement 1: The central and farm management practices while focusing government launched the AIBP from 1996-97 for on districts which have high potential but extending loan assistance to states for relatively low level of productivity at present. completion of near complete irrigation projects The NFSM has three components (i) National and not to extend loans to poor farmers. Hence, Food Security Mission - Rice (NFSM-Rice); (ii) statement 1 is incorrect. National Food Security Mission - Wheat (NFSM- Statement 2: Command Area refers to the area Wheat); and National Food Security Mission - whose water needs are fulfilled by a water Pulses (NFSM Pulses).Hence option 2 is correct. body/basin project e.g.: River Ganga’s command Educational Objective: to have knowledge area spans across 5 states of Uttarakhand, UP, about the various initiatives taken to ensure the Bihar, & West Bengal. It includes food security in India.

the area which is otherwise uncultivable. 8.6 Agricultural Credit Command area development plan was launched 20. Answer: A in 1974-75 to narrow the gap between irrigation Explanation: In India, commercial banks have created and actually utilized in major and the highest share in the disbursement of credit medium irrigation schemes and hence to agriculture and allied activities. The increasing water use efficiency. Therefore, commercial banks disburse around statement 2 is correct. 60% credit followed by cooperative banks Educational Objective: To know about the around 30% and RRB and others. Hence, option various schemes to improve the irrigational area 1 is correct in the country to improve the agricultural Educational Objective: to know about the produce. The above two schemes are now availability of agricultural credit to the farmers merged with Pradhan Mantri Krishi Sinchai and various financial institutions working Yojana. towards this goal.

19. Answer: B Explanation: The National Food Security Mission (NFSM) was launched in 2007-08 with a view to enhancing the production of rice, wheat, www.laex.in Page No. 98 https://elearn.laex.in

www.laex.in

ECONOMY UPSC

Previous Year Questions

21. Answer : A The benefits under the scheme are as under: Explanation: Sources of agricultural credit can Death due to accident be broadly classified into institutional and non- (within 12 months institutional sources. of the accident) Non-Institutional sources include moneylenders, a) Rs.50,000/- caused by traders and commission agents, relatives and outward, violent landlords, but institutional sources include co- and visible means. operatives, commercial banks including the SBI Group, RBI and NABARD. The majority of Permanent total b) Rs.50,000/- agricultural credit to farmers come from disability

institutional sources hence option B is correct Loss of two limbs or two Educational Objective: to know about the c) eyes or one limb Rs.50,000/- availability of agricultural credit to the farmers and one eye and various financial institutions working Loss of one limb or one towards this goal. d) Rs.25,000/- eye 22. Answer: B Educational Objective: to know Explanation: The Kisan Credit Card (KCC) comprehensively about the KISAN CREDIT scheme was announced in the Budget speech of CARD scheme and the benefits and challenges 1998-99 to fulfil the financial requirements of of the scheme. the farmers at various stages of farming through 8.7 Agriculture Inputs institutional credit.

The model scheme was prepared by the National 23. Answer : B Bank for Agriculture and Rural Development Explanation: Capital” in an economic context (NABARD) on the recommendation of V Gupta means machinery or capital goods which can be committee. The KCC scheme is being employed to produce other goods. Substitution implemented by the all Co-operative banks, of steel can be considered as the substitution of Regional Rural Banks and Public Sector Banks a lesser machine by a better machine. This throughout the country. Scheme covers risk of encourages steel production. Hence it is a KCC holders against death or permanent capital-augmenting technological progress. disability resulting from accidents. The loans Hence, option B is correct disbursed to farmers under Kisan. Educational Objective: To have knowledge Credit Card Scheme are not covered under about the technological developments in the Rashtriya Krishi Beema Yojna of Life Insurance field of agriculture and also various methods of Corporation of India. Hence, statement 1 is modernization of agriculture.

incorrect 24. Answer : B Statement 2 is correct: Explanation: Statement 1: Thus, the National Risk Coverage Seed Policy 2002 was launched to – provide

www.laex.in Page No. 99 https://elearn.laex.in

www.laex.in

ECONOMY UPSC

Previous Year Questions

intellectual property protection to new varieties; Option C: Agricultural Produce (Grading and usher this sector into planned development; Marking) Act, 1937 is An Act to provide for the protect the interest of farmers and encourage grading and marking of agricultural and other conservation of agro-biodiversity. Import produce. of seeds and planting material. Export of seeds. Option D: Fruit Products Order -1955, Promotion of Domestic Seed,Hence statement 1 promulgated under Section 3 of the Essential is incorrect as there is already a national seed Commodities Act - 1955, aims at regulating policy in place. sanitary and hygienic conditions in manufacture Statement 2: There are many private seeds of fruit, vegetable products companies like Monsanto, Syngenta etc who are Meat Food Products Order, 1973 (MFPO) in the market for providing quality seeds and promulgated under the provisions of Essential planting material in horticulture sector. Hence, Commodities Act, 1955 provides for sanitary statement 2 is incorrect and other requirements, limits of heavy metals, Statement 3 is correct as in case of low value preservatives, insecticides, residue, etc., for and high volume crops such as wheat and rice, meat food products. farmers tend to use their own preserved seeds Educational Objective: To know about the as there is a demand-supply gap regarding various legislations brought to make the quality seeds agricultural marketing more efficient.

Educational Objective: to know about the 26. Answer : A challenges associated with quality seeds in India Explanation : The Bureau of Indian and steps taken by the government of India in Standards (BIS) is the national Standards Body this regard. of India working under the aegis of Ministry of 8.8 Agriculture Marketing Consumer Affairs, Food & Public

25. Answer: B Distribution, Government of India. Explanation: Option A: The Essential It is established by the Bureau of Indian Commodities Act (ECA) was enacted by the Standards Act, 1986 which came into effect on Central Government in 1955 to control and 23 December 1986. regulate trade and prices of commodities BIS is responsible for the harmonious declared essential under the Act. development of the activities of standardization, Option B is correct: The APMC Act empowers marking and quality certification of goods and state governments to notify the commodities, for matters connected therewith or incidental and designate markets and market areas where thereto. the regulated trade takes place. The Act also AGMARK was established under the provides for the formation of agricultural Agriculture Produce (Grading and marking) Act produce market committees (APMC) that are of India, 1937 ( amended in 1986), while FSSAI responsible for the operation of the markets. was established under the Food Safety and Standard Act, 2006.

www.laex.in Page No. 100 https://elearn.laex.in

www.laex.in

ECONOMY UPSC

Previous Year Questions

AGMARK is a certification mark employed on 3. Which one of the following committees agricultural products in India, assuring that recommended the abolition of they conform to a set of standards approved by reservation of items for the small scale the Directorate of Marketing and Inspection, an sector in industry? agency of the Government of India. Hence a) AbidHussain Committee statement 2 is incorrect. b) Narasimhan Committee Educational objective: To learn about Bureau c) Nayak Committee of Indian Standards (BIS) and AGMARK. d) Rakesh Mohan Committee

4. Consider the following statements: 9. Industry Small-scale industries are in most

9.1 Core Industries and IIP cases, not as efficient and competitive as the large-scale ones. 1. In the 'Index of Eight Core Industries', Yet the Government provides which one of the following is given the preferential treatment and reservations highest weight? in a range of products to the small firms a) Coal production because small-scale industries b) Electricity generation 1) Provide higher employment on a per unit c) Fertilizer production capital development basis d) Steel production

2) Promote a regional dispersion of industries 2. In India, in the overall Index of and economic activities Industrial Production, the Indices of 3) Have performed better in export of Eight Core Industries have a combined manufactured products than the large weight of 37.90%. Which of the scale ones following are among those Eight Core 4) Provide jobs to low-skill workers, who Industries? otherwise may not find employment 1) Cement avenues elsewhere 2) Fertilizers Which of the above statements are correct? 3) Natural Gas a) 1 and 4 4) Refinery products b) 1 and 2 5) Textiles c) 2 and 3 Select the correct answer using the codes d) 3 and 4

given below: 9.3 PSU’s a) 1 and 5 only

b) 2,3 and 4 only 5. Consider the following statements: c) 1,2,3 and 4 only 1) MMTC Limited is India's largest international trading organization. d) 1,2,3,4 and 5

9.2 Small scale Industries (MSME’s) www.laex.in Page No. 101 https://elearn.laex.in

www.laex.in

ECONOMY UPSC

Previous Year Questions

2) NeelachalIspat Nigam Limited has been set 2) Share of India's exports in world trade up by MMTC jointly with the Government of increased. Orissa. 3) FDI inflows increased. Which of the statements given above is/are 4) India's foreign exchange reserves increased correct? enormously. a) 1 only Select the correct answer using the codes b) 2 only given below: c) Both 1 and 2 a) 1 and 4 only d) Neither 1 nor 2 b) 2, 3 and 4 only

6. Which of the following are public sector c) 2 and 3 only undertakings of the Government of d) 1, 2, 3 and

India? 9. Economic liberalization in India started 1) BalmerLawrie and Company Ltd. with 2) Dredging Corporation of India a) Substantial changes in industrial licensing 3) Educational Consultants of India Ltd. policy Select the correct answer using the code given b) The convertibility of Indian rupee below: c) Doing away with procedural formalities for Code: foreign direct investment a) 1 and 2 only d) Significant reduction in tax rates.

b) 2 and 3 only c) 1, 2 and 3 9.5 Investment and Disinvestment

d) 1,2 and 3 10. Why is the Government of India 9.4 Historical Development of Industries disinvesting its equity in the Central & Industrial Policy Public Sector Enterprises (CPSES)?

7. There was no independent development of 1) The Government intends to use the industries in India during British rule revenue earned from the disinvestment because of the mainly to pay back the external debt. a) absence of heavy industries 2) The Government no longer intends to b) scarcity of foreign capital retain the management control of the c) scarcity of natural resources CPSEs. d) preference of the rich to invest in land Which of the statements given above is/ are

8. Which of the following has/have correct? occurred in India after its liberalization a) 1 only of economic policies in 1991? b) 2 only 1) Share of agriculture in GDP increased c) Both 1 and 2 enormously. d) Neither 1 nor 2

www.laex.in Page No. 102 https://elearn.laex.in

www.laex.in

ECONOMY UPSC

Previous Year Questions

11. The Government holding in BPCL is, Select the correct answer using the codes a) more than 70% given below: b) between 60% and 70% a) 1 only c) between 50% and 60% b) 2 and 3 d) less than 50% c) 1 and 3 only d) 1,2 and 3 12. With reference to the National

Investment Fund to which the 14. In the year 2001, the Prime Minister disinvestment proceeds are routed, announced a five-year excise duty consider the following statements: holiday for industries in 1) The assets in the National Investment a) Cyclone prone coastal Andhra Pradesh Fund are managed by the Union Ministry b) Border states of North-East of Finance c) Earthquake revaged Kutch district 2) The National Investment Fund is to be d) Recently formed states of Chhattisgarh maintained within the Consolidated Fund and Jharkhand

of India 15. During the year 2000-01, which one of 3) Certain Asset Management Companies are the following industries recorded appointed as the fund managers highest growth rate in India? 4) A certain proportion of annual income is a) Cement used for financing select social sectors b) Coal Which of the statements given above is/are c) Electricity correct? d) Steel a) 1 and 2 9.7 Different types of industries

b) 2 only 16. Tourism industry in India is quite small c) 3 and 4 compared to many other countries in d) 3 only

terms of India's potential and size. 9.6 Current trends in Industrial Sector Which one of the following statements

13. What is/are the recent policy initiative is correct in this regard? (s) of Government of India to promote a) Distances in India are too far apart and its the growth of manufacturing sector? luxury hotels are too expensive for 1) Setting up of National Investment and Western tourists Manufacturing zones b) For most of the months India is too hot for 2) Providing the benefit of 'single window Western tourists to feel comfortable clearance' c) Most of the picturesque resorts in India 3) Establishing the Technology Acquisition such as in the North-East and Kashmir and Development Fund are, for all practical purposes, out of bounds

www.laex.in Page No. 103 https://elearn.laex.in

www.laex.in

ECONOMY UPSC

Previous Year Questions

d) In India, the infrastructure required for a) India is the second largest producer of attracting tourists is inadequate nitrogenous fertilizers in the world

17. Some time back, the Government of b) India is the ninth largest steel producing India, decided to delicense 'white goods' country in the world industry. 'white goods' include, c) India is the second largest producer of silk a) Stainless steel and aluminium utensils in the world b) Milk and milk products d) India ranks third in the world in coal c) Items purchased for conspicuous production

consumption d) Soaps, detergents and other mass 9. Key and Explanations consumption goods 9.8 Miscellaneous 9.1 Core Industries and IIP

18. Which one of the following is not a 1. Answer : B feature of Limited Liability Partnership Explanation: The Eight Core Industries firm? comprise nearly 40.27% of the weight of items a) Partners should be less than 20 included in the Index of Industrial Production b) Partnership and management need not be (IIP). These are Electricity, steel, refinery separate products, crude oil, coal, cement, natural gas c) Internal governance may be decided by and fertilizers. mutual agreement among partners Index of Industrial Production d) It is corporate body with perpetual The Index of Industrial Production (IIP) is an succession index which details out the growth of various sectors in an economy such as mineral mining, 19. Consider the following pairs: electricity, manufacturing, etc. Automobile Manufacturer & Country It is compiled and published monthly by 1) BMW AG : USA the Central Statistical Organisation 2) Daimler AG : Sweden (CSO), Ministry of Statistics and Programme 3) Renault S.A. : France Implementation six weeks after the reference 4) Volkswagen AG : Germany month ends, i.e a lag of six weeks. Which of the pairs given above is/are The Base Year of the Index of Eight Core correctly matched? Industries has been revised from the year 2004- a) 1, 2 and 3 05 to 2011-12 from April, 2017. b) 3 and 4 The eight Core Industries in decreasing order of c) 4 only their weightage: Refinery Products>Electricity> d) 1,2 and 4

Steel> Coal> Crude Oil> Natural Gas> 20. Which one of the following statements is Cement> Fertilizers. NOT correct?

www.laex.in Page No. 104 https://elearn.laex.in

www.laex.in

ECONOMY UPSC

Previous Year Questions

Earlier in July, 2018 a report by V.K Saraswat Earlier in July, 2018 a report by V.K Saraswat (NITI Aayog member) had recommended that the (NITI Aayog member) had recommended that the government should consider classifying government should consider classifying the aluminium sector as India’s ninth core the aluminium sector as India’s ninth core industry. industry. Weight (In Educational Objective: To know about Index of Industry Percent industrial production.

age) 9.2 Small Scale Industries (MSME’s) Petroleum & Refinery 28.04 3. Answer : A Production Explanation: A bid Hussain Committee Electricity generation 19.85 recommended the abolition of reservation of Steel production 17.92 items for the small scale sector in industry. Coal production 10.33 Educational Objective: To know about Crude Oil production 8.98 important committees recommendations. Natural Gas production 6.88 4. Answer : B Cement production 5.37 Explanation: Small scale industries (SSI) are Fertilizers production 2.63 those industries in which manufacturing, Educational Objective: To know about core providing services, productions are done on a industries. small scale or micro scale. Small scale

2. Answer : C industries play an important role in social and Explanation: Index of Industrial Production economic development of India. The Index of Industrial Production (IIP) is an Importance of MSMEs for Indian Economy index which details out the growth of various Employment: It is the second largest sectors in an economy such as mineral mining, employment generating sector after agriculture. electricity, manufacturing, etc. It provides employment to around 120 million It is compiled and published monthly by persons in India. the Central Statistical Organisation Contribution to GDP: With around 36.1 million (CSO), Ministry of Statistics and Programme units throughout the geographical expanse of Implementation six weeks after the reference the country, MSMEs contribute around 6.11% month ends, i.e a lag of six weeks. of the manufacturing GDP and 24.63% of the The Base Year of the Index of Eight Core GDP from service activities. Industries has been revised from the year 2004- MSME ministry has set a target to up its 05 to 2011-12 from April, 2017. contribution to GDP to 50% by 2025 as India The eight Core Industries in decreasing order becomes a $5 trillion economy. of their weightage: Refinery Products> Exports: It contributes around 45% of the Electricity> Steel> Coal> Crude Oil> Natural overall exports from India. Gas> Cement> Fertilizers. www.laex.in Page No. 105 https://elearn.laex.in

www.laex.in

ECONOMY UPSC

Previous Year Questions

Inclusive growth: MSMEs promote inclusive 6. Answer: D growth by providing employment opportunities Explanation: Public sector undertaking (PSU) in rural areas especially to people belonging to or a public sector enterprise those companies weaker sections of the society. are owned by the union government of India or For example: Khadi and Village industries one of the many state or territorial require low per capita investment and employs a governments or both. The company stock needs large number of women in rural areas. to be majority-owned by the government to be a Financial inclusion: Small industries and retail PSU. businesses in tier-II and tier-III cities create PSUs strictly may be classified as central public opportunities for people to use banking services sector enterprises (CPSEs) or state level public and products. enterprises (SLPEs). In 1951 there were just five Promote innovation: It provides opportunity for enterprises in the public sector in India, but in budding entrepreneurs to build creative March 1991 this had increased to 246. products boosting business competition and CPSEs are companies in which the direct fuels growth. holding of the Central Government or other Thus, Indian MSME sector is the backbone of CPSEs is 51% or more. They are administered the national economic structure and acts as a by the Ministry of Heavy Industries and Public bulwark for Indian economy, providing Enterprises. resilience to ward off global economic shocks Balmer Lawrie & Co. Ltd. (BL) is a partnership and adversities. firm founded on 1 February 1867 Educational Objective: To know about in Kolkata, India by two Scotsmen: George importance of MSMEs Stephen Balmer and Alexander Lawrie. Today

9.3 PSU’s BalmerLawrie is a Mini-Ratna I Public Sector 5. Answer : C Enterprise under the Ministry of Petroleum and Explanation: MMTC Limited is the largest Natural Gas Government of India international trading company in India and has DCI was incorporated as a public sector been in existence for almost five decades. It organisation in March 1976. handles the export of primary products such as Educational Consultants India coal, iron ore, and manufactured agro and Limited EdCIL is a Public Sector Undertaking industrial products. MMTC also imports in India under administrative control of important commodities such as ferrous and the Ministry of Human Resource nonferrous metals for industry, and agricultural Development. It is a Miniratna-I category fertilizers. NeelachalIspat Nigam Ltd located at company. Hence all are correct. Kalinga Nagar Industrial Complex, Duburi is a Educational Objective: To know about PSUs

joint venture of MMTC and Orissa Government. Educational Objective: To know about

important PSUs

www.laex.in Page No. 106 https://elearn.laex.in

www.laex.in

ECONOMY UPSC

Previous Year Questions

www.laex.in Page No. 107 https://elearn.laex.in

www.laex.in

ECONOMY UPSC

Previous Year Questions

9.4 Historical Development of Industries Impact on Agriculture: In the area of & Industrial Policy agriculture, the cropping patterns has 7. Answer : A undergone a huge modification, but the impact Explanation: Indian Industries During British of liberalisation cannot be properly measured. It Rule was a well-known producer of handicraft is observed that there are still all-pervasive and textile business. India was also famous for government controls and interventions starting its industries in silk and cotton textiles. In from production to distribution for the produce. addition, Indians were skilled in metal and high- Free flow of capital: Liberalisation has priced stonework too. When the Britishers improved flow of capital into the country which came, they were followers of de-industrialization makes it inexpensive for the companies to in India. access capital from investors. Lower cost of Major reasons which were responsible for capital enables to undertake lucrative projects bleak growth of industrial sector: which they may not have been possible with a 1. Systematic De-Industrialisation: The higher cost of capital pre-liberalisation, leading Primary motive of Britishers behind de- to higher growth rates. industrialization was two-fold(i) To make India a Stock Market Performance: Generally, when a net exporter of Raw material to the British country relaxes its laws, taxes, the stock market Industries.(ii) To sell British products at a values also rise. Stock Markets are platforms on higher rate in the Indian market. which Corporate Securities can be traded in real Lopsided modern industrial structure: British time. rulers never permitted modernization nor did Political Risks Reduced: Liberalisation policies they encourage the growth of Industries. in the country lessens political risks to Unbalanced and lopsided growth structure was investors. The government can attract more a legacy of British rule in India. foreign investment through liberalisation of 3. Lacking capital goods industry: Capital economic policies. These are the areas that goods industry refers to the industry which support and foster a readiness to do business in produces goods such as machines, tools, the country such as a strong legal foundation to etc. which are further capable of producing settle disputes, fair and enforceable laws. consumer goods. Educational Objective: To know about impact That is why Jawahar Lal Nehru emphasized of LPG reforms in india

on setting up of heavy industries in the 9. Answer : A second five year plan. Explanation: Economic liberalization in India Educational Objective: To know about started with industrial licensing policy .Sincethe industrial sector growth trend adoption of the New economic strategy in 1991,

8. Answer: B there has been a drastic change in the Indian Explanation: Some of the Impacts of economy. With the arrival of liberalisation, the liberalisation in India government has regulated the private sector

www.laex.in Page No. 108 https://elearn.laex.in

www.laex.in

ECONOMY UPSC

Previous Year Questions

organisations to conduct business transactions of it is used for educational, health, and with fewer restrictions. barriers included tax employment sectors. laws, foreign investment restrictions, accounting Educational Objective: To know about National regulations, and legal issues. The economic Investment Fund.

liberalisation reduced all these obstacles and 9.6 Current trends in Industrial Sector

waived few restrictions over the control of the 13. Answer : D economy to the private sector. Explanation: NIMZs have been conceived as Educational Objective: To know about LPG large integrated industrial townships with state reforms

of-the-art infrastructure; land use on the basis 9.5 Investment and Disinvestment of zoning; clean and energy efficient technology;

10. Answer : D necessary social infrastructure; skill Explanation: The Government use the revenue development facilities, etc., to provide a earned from the disinvestment for various conducive environment for manufacturing activities not particularly to pay external debt. industries. Due to management issues, fiscal constrains it Government of India has notified the Scheme for may go for disinvestment. Technology Acquisition and Development Fund Educational Objective: To know about (TADF) to provide funding support to MSMEs for disinvestment. the acquisition and development of clean and

11. Answer : C green technology. The Scheme is applicable to Explanation : As of September 2018, 54% of all existing and new Micro, Small and Medium the shares of BPCL were owned by Enterprises (MSMEs) including those in the the Government of India (through the President National Investment and Manufacturing Zones of India), with the rest owned by Foreign (NIMZs). Portfolio Investors (17%), BPCL trust for Educational Objective: To know about various investing in shares (9%), Mutual funds and UTI government initiative to develop growth in (7.5 %), Insurance companies (6%) and the manufacturing sector.

balance held by individual share. 14. Answer : C Educational Objective: To know about PPP in Explanation: In the year 2001, the Prime India. Minister Atal Bihari Vajapayee announced a five

12. Answer : C year excise duty holiday for industries in Explanation: On 27 January 2005, the earthquake ravaged kuch district in Gujarath. government had decided to constitute a National Educational Objective: To know about tax Investment Fund into which realization from related announcement towards industries sale of minority shareholding of the sector. Governments in CPSEs would be channelized. 15. Answer : D This fund is managed by professionals and part Explanation: The steel industry recorded highest growth rate of 7%, electricity generation www.laex.in Page No. 109 https://elearn.laex.in

www.laex.in

ECONOMY UPSC

Previous Year Questions

growth rate was 4%, coal industry growth rate 17. Answer : C was 3.3%. Explanation: The India consumer durable Educational Objective: To know about segments are: industrial growth.  White goods: air conditioners,

9.7 Different Types of Industries refrigerators, whashing machines, sewing machines, 16. Answer : D  Brown goods: microwave ovens, cooking Explanation: In India due to its culture and range, chimneys etc monuments it is preferred for the tourist destination by lack infrastructure is an  Consumer electronics: TVs, electronic hindrance to this industry. accessories. PCs etc Challenges to the Growth of White goods: Heavy consumer durables such as air conditioners, refrigerators, stoves, etc., Tourists in India still face many infrastructure which used to be painted only in white enamel related problems like inadequate roads, water, finish. Despite their availability in varied colors sewer, hotels and telecommunications etc. now, they are still called white goods.  Safety and security of tourists, especially of Educational Objective: To know about various the foreign tourists, is a major hurdle to the types of goods. tourism development. Attacks on foreign

nationals raise questions about India’s 9.8 Miscellaneous

ability to welcome tourists from far away 18. Answer : A countries. Among the 130 countries Explanation: A limited liability surveyed, India was placed at the 114th partnership (LLP) is a partnership in which position in terms of safety and security some or all partners (depending on the aspect in the WEF Index 2017. jurisdiction) have limited liabilities. It therefore  Lack of skilled manpower is another can exhibit elements challenge to Tourism Industry in India. of partnerships and corporations. In an LLP,  Absence of basic amenities like drinking each partner is not responsible or liable for water, well maintained toilets, first aid, another partner's misconduct or negligence. cafeteria etc. at tourist places. LLP is a corporate body and a legal entity  Seasonality in Tourism, with the busy separate from its partners. It will have perpetual season being limited to six months from succession. October to March and heavy rush in Indian Partnerships Act, 1932 is not applicable November and December. to LLPs and there shall not be any upper limit  Non-acceptance of International Cards at on numbers of partners in an LLP unlike an small outlets. ordinary partnership firm where the maximum Educational Objective: To know about tourism numbers of partners cannot exceed 20. Hence industry. statement 1 is wrong.

www.laex.in Page No. 110 https://elearn.laex.in

www.laex.in

ECONOMY UPSC

Previous Year Questions

Educational objective: To learn about limited acid, ammonium nitrate, and urea. Synthetic liability partnerships and its features. ammonia and nitric acid are used primarily as

19. Answer : B intermediates in the production of ammonium Explanation: Bayerische Motoren Werke AG, nitrate and urea fertilizers. commonly referred to as BMW is a German Educational objective: To learn about the multinational company which produces luxury India’s ranking in the global production of vehicles and motorcycles. The company was nitrogen fertilizers, steel, silk and gold.

founded in 1916 as a manufacturer of aircraft 10. Infrastructure

engine. Hence pair 1 is wrong. Daimler AG previously named Daimler- 10.1 Energy Sector

Benz and DaimlerChrysler, commonly known 1. Consider the following statements: and referred to as Mercedes, is a Petroleum and Natural Gas Regulatory German multinational automotive corporation, Board (PNGRB) is the first regulatory body headquartered in Stuttgart, Baden- set up by the Government of India. Württemberg. Hence pair 2 is wrong. 1) One of the tasks of PNGRB is to ensure Groupe Renault legally Renault S.A. is a competitive markets for gas. French multinational automobile 2) Appeals against the decisions of PNGRB go manufacturer established in 1899. The before the Appellate Tribunals for company produces a range of cars and vans, Electricity. and in the past has manufactured trucks, Which of the statements given above are tractors, tanks, buses/coaches, aircraft and correct? aircraft engines, and autorail vehicles. Hence a) 1 and 2 only pair 3 is correct. b) 2 and 3 only Volkswagen AG known internationally as c) 1 and 3 only the Volkswagen Group, is a d) 1, 2 and 3

German multinational automotive manufacturin 2. Consider the following statements: g company headquartered in Wolfsburg, Lower 1) National Thermal Power Corporation has Saxony, Germany. Hence pair 4 is correct. diversified into hydropower sector Educational objective: To learn about global 2) Power Grid Corporation of India has automobile manufacturers and their location diversified into telecom sector origin Which of the statements given above is/are

20. Answer : A correct? Explanation:India is the third largest producer a) 1 only of nitrogenous fertilizers in the world after b) 2 only China and USA. Hence option A is wrong. c) Both 1 and 2 The nitrogenous fertilizer industry includes the d) Neither 1 nor 2 production of synthetic ammonia, nitric

www.laex.in Page No. 111 https://elearn.laex.in

www.laex.in

ECONOMY UPSC

Previous Year Questions

3. Consider the following statements: c) Replacing the coal-based power plants 1) Damodar Valley Corporation is the first with natural gas, nuclear, solar, wind and multipurpose river valley project of tidal power plants over a period of time independent India d) Providing for financial turnaround and 2) Damodar Valley Corporation includes revival of power distribution companies thermal and gas power stations 6. Which one of the following brings out Which of the statements given above is/are the publication called "Energy correct? Statistics" from time to time?

a) 1 only a) Central Power Research Institute

b) 2 only b) Planning Commission

c) Both 1 and 2 c) Power Finance Corporation Ltd.

d) Neither 1 nor 2 d) Central Statistical Organization

4. Consider the following statements: 7. With which one of the following has the 1) The Oil Pool Account of Government of B.K. Chaturvedi Committee dealt? India was dismantled with effect from 1 a) Review of Centre-State relations April 2002 b) Review of Delimitation Act 2) Subsidies on PDS kerosene and domestic c) Tax reforms and measures to increase LPG are borne by Consolidated Fund of revenues India d) Price reforms in the oil sector

3) An expert committee headed by Dr. RA Mashelkar to formulate a national auto 10.2 Government Measures

fuel policy recommended that Bharat 8. The SEZ Act, 2005 which came into State-II Emission Norms should be applied effect in February 2006 has certain throughout the country by 1 April, 2004 objectives. In this context, consider the Which of the statements given above are following: correct? 1) Development of infrastructure facilities a) 1 and 2 2) Promotion of investment from foreign b) 2 and 3 sources c) 1 and 3 3) Promotion of exports of services only d) 1, 2 and 3 Which of the above are the objectives of this

5. Which one of the following is a purpose Act? of 'UDAY', a scheme of the Government? a) 1 and 2 only a) Providing technical and financial b) 3 only assistance to start- up entre-preneurs in c) 2 and 3 only the field of renewable sources of energy d) 1, 2 and 3

b) Providing electricity to every household in 9. Among other things, which one of the the country by 2018 following was the purpose for which the

www.laex.in Page No. 112 https://elearn.laex.in

www.laex.in

ECONOMY UPSC

Previous Year Questions

Deepak Parekh Committee was Select the correct answer using the code constituted? given below. a) To study the current socio-economic a) 1 and 2 conditions of certain minority communities b) 3, 4 and 5 b) To suggest measures for financing the c) 1,3 and 4 development of infrastructure d) 2 and 5

c) To frame a policy on the production of 10.3 Public Sector Undertakings

genetically modified organisms 12. With reference to the Public Sector d) To suggest measures to reduce the fiscal Undertakings in India, consider the deficit in the Union Budget.

following statement 10. With what purpose is the Government 1) Minerals and Metals Trading Limited is the of India promoting the concept of "Mega largest non-oil importer of the country. Food Parks"? 2) Project and Equipment Corporation of India 1) To provide good infrastructure facilities for Limited is under the Ministry of Industry. the food processing industry. 3) One of the objectives of Export Credit 2) To increase the processing of perishable Guarantee Corporation of India Limited is to items and reduce wastage. enforce quality control and compulsory pre- 3) To provide emerging and eco-friendly food shipment inspection of various exportable processing technologies to entrepreneurs. commodities. Select the correct answer using the codes Which of these statements is/are correct? given below:

a) 1 only a) 1 only b) 1 and 2 b) 1 and 2 only c) 2 and 3 c) 2 and 3 only d) 3 only d) 1, 2 and 3

11. In India, which of the following review 10.4 Miscellaneous

the independent regulators in sectors 13. In the context of any country, which like telecommunications, insurance, one of the following would be considered as part of its social capital? electricity, etc.? a) The proportion of literates in the 1) Ad Hoc Committees set up by the population Parliament b) The stock of its buildings, other 2) Parliamentary Department Related infrastructure and machines Standing Committees c) The size of population in the working age 3) Finance Commission group 4) Financial Sector Legislative Reforms d) The level of mutual trust and harmony in Commission the society

5) NITI Aayog

www.laex.in Page No. 113 https://elearn.laex.in

www.laex.in

ECONOMY UPSC

Previous Year Questions

10. Key and Explanations operation four coal based thermal power station,

three hydel power stations and one gas turbine 10.1 Energy Sector station.

1. Answer : B Educational Objective: To know about the Explanation: PGNRB was constituted in 2006, multipurpose river valley projects and the A number of regulatory bodies have existed corporations implementing those projects

before. 4. Answer: A The board has several functions such as to Explanation: The Oil Pool Account of provide, by regulations, and enforce, retail Government of India was dismantled with effect service obligations for retail outlets and from 1 April 2002. Subsidies on PDS kerosene marketing service obligations for entities; and domestic LPG are borne by Consolidated monitor transportation rates and take corrective Fund of India action to prevent restrictive trade practice by the An expert committee headed by Dr. R. A. entities etc. This clearly indicates that is Mashelkar to formulate a national auto fuel ensures a competitive market for gas. policy recommended that Bharat Stage – II The Appellate Tribunal established under Emission Norms should be applied throughout section 110 of the Electricity Act, 2003 (36of the country by 1 April, 2005. Hence 1 and2 2003) shall be the Appellate Tribunal for statements are correct. PGNRB. Educational Objective: To know about the Educational Objective: to about the regulatory contemporary developments in the field of bodies and boards regulating the oil industry in Infrastructure like various committees set up, India. expenditure, new projects etc.

2. Answer : C 5. Answer: D Explanation: National thermal power Explanation: Financial problems with power cooperation has diversified into the hydro distribution companies is being overcome by projects. It had undertaken Koldem project in this scheme. As on March 2015, the Himachal Pradesh. accumulated losses and the outstanding debt of Educational Objective: to know about the discoms are approximately Rs. 3.8 lakh crore steps taken by government of India in power and Rs. 4.3 lakh crore, respectively. Became sector and also the projects undertaken major news and Economic Survey dedicated a

full chapter to this scheme. Then Budget also 3. Answer : C took major action. Explanation: Damodar Valley corporation is the Ujjwal DISCOM Assurance Yojana (UDAY) is first multipurpose river valley project of the financial turnaround and revival package for independent India was in the year 1954. The electricity distribution companies of India joint venture project are Maithon power limited (DISCOMs) initiated by the Government of Panchat and tilayiya are hydropower station. India with the intent to find a permanent The Damodar valley corporation has under its

www.laex.in Page No. 114 https://elearn.laex.in

www.laex.in

ECONOMY UPSC

Previous Year Questions

solution to the financial mess that the power Special Economic Zones Act, 2005, was passed distribution is in. It allows state governments, by Parliament in May, 2005 and received which own the DISCOMs, to take over 75 Presidential assent on the 23rd of June, 2005. percent of their debt as of September 30, 2015, The act envisages that the SEZs would attract a and pay back lenders by selling bonds. large ow of foreign and domestic investment in DISCOMs are expected to issue bonds for the infrastructure and productive capacity leading remaining 25 percent of their debt. to generation of additional economic activity and Educational Objective: to know about various creation of employment opportunities. schemes of the central government in order to Features boost the power sector. A SEZ is a designated duty free enclave to be

6. Answer: D treated as foreign territory for the purpose of Explanation: It is published by Ministry of trade operations and duties and tariffs. A SEZ statistics and programmed implementation, does not require a license for imports. Other Central Statistical Organisation. notable features are as follows: Educational Objective: To know about the  The units must become net foreign various publications of various ministries and exchange earners within 3 years SEZ are departments in the field of infrastructure. allowed manufacturing, trading and 7. Answer: D service activities. Explanation: B. K Chaturvedi is former cabinet  Full freedom for subcontracting. secretary and member planning commission he  The domestic sales from the SEZ are also headed committee to restructure centrally subject to full custom duties and import sponsored schemes. policy is in force, when they sell their Educational Objective: government of India produce to domestic markets. There was wants to consolidate various centrally sponsored no routine examination by the custom schemes. Hence the initiatives towards this authorities. becomes important for the examination.  The corporation in SEZs will not have to

10.2 Government Measures pay any income tax on their profits for the first five years and only 50% of the 8. Answer: A tax for 2 more years thereafter. Explanation: Main objectives of SEZs special If half of the profit is reinvested in the export sones: corporation, the concession of 50% tax is Making available goods and services free of extendable for next 3 years taxes and duties supported by and integrated Educational Objective: to know about the infrastructure for export production and single various initiatives of government in order to window approval mechanism and a package of boost the exports in the manufacturing sector in incentives to attract foreign and domestic India. investments for promoting export led growth.

www.laex.in Page No. 115 https://elearn.laex.in

www.laex.in

ECONOMY UPSC

Previous Year Questions

9. Answer : B various initiatives in the sector like mega food Explanation: The High-level government park scheme.

committee on infrastructure headed by Deepak 11. Answer: A Parekh submits the last part of its report in Explanation: Finance commission and NITI October. The Committee consisted of Aayog are of advisory nature, they do not review UdayKotak, G.M.R Rao, Sanjay Reddy and top the functioning of any regulator. So, FSLRC was officials from LIC, SBI, ICICI and IDFC. setup once to review financial legislations, and Some of their proposals are as follows:- not regulators in the country.  Establish a PPP model for power The Parliamentary Departmental related distribution, starting rst with the cities standing Committees may do such work, and  Modernise public sector distribution the ad hoc committees sometimes setup to companies. The viability gap for such review the working of regulators specific to their companies should be funded by the respective departments. Central government Educational Objective: To know about the  Privatise coal mining wherein Coal India or various regulating bodies in the various sectors its arms/subsidiaries can retain and their functions. ownership of mines 10.3 Public Sector Undertakings

 Constitute a High level task force headed by a deputy governor of RBI to decide on 12. Answer : B measures to restore the health of power Explanation: MMTC Ltd., Metals and Minerals projects facing nancial trouble due to Trading Corporation of India, is one of the two scarcity of fuel Fixed charges of a highest earners of foreign reasonable amount should be exchange for India and India's largest public apportioned to idle power plants. sector trading body. Not only handling the Educational Objective: to know about the export of primary products such as coal, iron committees set up by the government of India in ore, and manufactured agro and industrial various sectors like infrastructure. products, MMTC also imports important commodities such as ferrous and nonferrous 10. Answer: D metals for industry, and agricultural fertilizers. Explanation: Ministry of food processing Hence statement 1 is correct. industry was set up during Eleventh Five Year plan. This ministry will provide good PEC Ltd -formerly known as The Project & infrastructural facilities for food processing, Equipment Corporation of India Ltd – is an increase the processing of perishable items and international trading company under the aegis reduce wastage and provide emerging of Department of Commerce, Ministry of technologies to entrepreneurs Commerce & Industry, Government of India, Educational Objective: To know about the was incorporated in 1971. scope of food processing industries in India and

www.laex.in Page No. 116 https://elearn.laex.in

www.laex.in

ECONOMY UPSC

Previous Year Questions

ECGC Ltd. (Formerly known as Export Credit Common Wealth Games in India Guarantee Corporation of India Ltd.) wholly amounted to owned by Government of India, was set up in a) Export 1957 with the objective of promoting exports b) Import from the country by providing credit risk c) Production insurance and related services for exports. d) Consumption

Hence statement 3 is incorrect. 2. Consider the following statements: Educational objective: To learn about various 1) In India, during the financial year 2004- Public Sector Undertakings in India. 2005 an increase of below 10% over the value of exports (in rupee terms) in the 10.4 Miscellaneous

financial year 2003-2004 was reported. 13. Answer : D 2) According to the WTO, India's share in the Explanation: Options a and d are referred to world merchandise exports crossed 2% in human capital. the year 2005. Social capital is the effective functioning of Which of the statements given above is/are social groups through interpersonal correct? relationships, a shared sense of identity, a a) 1 only shared understanding, shared sense of identity, b) 2 only a shared understanding, shared norms, c) Both 1 and 2 shared values, trust, cooperation, d) Neither 1 nor 2 and reciprocity. 3. A: For the first time, India had no trade Social capital is a measure of the value of deficit in the year 2002-03. resources, both tangible (public spaces, private R: For the first time, India's exports property) and intangible ("actors", "human crossed worth $50 billion in the year 2002 capital", people), and the impact that these -03. relationships have on the resources involved in a) Both A and R are individually true and R each relationship, and on larger groups. It is is the correct explanation of A generally seen as a form of capital that b) Both A and R are individually true but R is produces public goods for a common good. not the correct explanation of A Educational objective: To learn about social c) A is true but R is false capital and its importance.

d) A is false but R is true

11.External Sector in India 4. Consider the following statements: 11.1 Trade 1) In the last 5 years, Indian software exports 11.1.1 Export Related Trade have increased at a compound annual

growth rate of about 60%. 1. In terms of economy, the visit by foreign nationals to witness the XIX

www.laex.in Page No. 117 https://elearn.laex.in

www.laex.in

ECONOMY UPSC

Previous Year Questions

2) The software and service industry in India 7. Which one of the following was the registered an overall growth of about 28% largest IT software and services in rupee terms during the year 2001-2002. exporter in India during the year 2002- Which of these statements is/are correct? 03? a) Only 1 a) Birlasoft b) Only 2 b) Infosys Technologies c) Both 1 and 2 c) Tata Consultancy Service d) Neither 1 nor 2 d) Wipro Technologies 8. Which one of the following was the 5. In terms of value, which one of the largest IT software and services following commodities accounted for the exporter in India during the year 2002- largest agricultural exports by India 03? during the three year period from 1997- a) Birlasoft 1998 to 1999-2000? b) Infosys Technologies CAGR Year/ 201 201 201 201 % c) Tata Consultancy Service Descri 3- 4- 5- 7-18 (2013- ption 14 15 16 (E) d) Wipro Technologies

18) Export 107. 126. 11.1.2 Import Related Trade

87.3 97.8 10.49 s 8 0 9. Among the agricultural commodities Domes 19.0 21.0 21.7 25.0 5.42 tic imported by India, which one of the 106. 118. 129. 151. following accounts for the highest Total 9.55 3 8 5 0 imports in terms of value in the last a) Cereals five years? b) Marine Products a) Spices c) Spices b) Fresh fruits d) Tea c) Pulses

6. A: The rate of growth of India's exports d) Vegetable oils

has shown an appreciable increase after 10. Which of the following best describes 1991. the term 'import cover', sometimes seen R: The Government of India has in the news? resorted to devaluation. a) It is the ratio of value of imports to the a) Both A and R are individually true and R Gross Domestic Product of a country is the correct explanation of A b) It is the total value of imports of a country b) Both A and R are individually true but R is in a year not the correct explanation of A c) It is the ratio between the value of exports c) A is true but R is false and that of imports between two countries d) A is false but R is true

www.laex.in Page No. 118 https://elearn.laex.in

www.laex.in

ECONOMY UPSC

Previous Year Questions

d) It is the number of months of imports that imported commodities (in terms of could be paid for by a country's rupee value)? international reserves a) Africa b) America 11. Despite having large reserves of coal, c) Asia and Oceania why does India import millions of d) Europe tonnes of coal?

1) It is the policy of India to save its own coal 11.1.3 Miscellaneous

reserves for future, and import it from 14. A: The new EXIM policy is liberal, market- other countries for the present use. oriented and favours global trade. 2) Most of the power plants in India are coal- R: GATT has played a significant role in based and they are not able to get the liberalization of the economy. sufficient supplies of coal from within the a) Both A and R are individually true and country. R is the correct explanation of A 3) Steel companies need large quantity of b) Both A and R are individually true but R coking coal which has to be imported. is not the correct explanation of A Which of the statements given above is/are c) A is true but R is false correct? d) A is false but R is true a) 1 only 15. India has the maximum volume of b) 2 and 3 only foreign trade with c) 1 and 3 only a) USA d) 1, 2 and 3

b) Japan 12. Consider the following statements: c) Germany 1) India's import of crude and petroleum d) UAE

products during the year 2001-02 accounted for about 27% of India's total 16. Which one of the following is not imports. correct? Dinar/New Dinar is the 2) During the year 2001-02, India's exports currency of had increased by 10% as compared to the a) Sudan previous year. b) Yugoslavia Which of these statements is/are correct? c) UAE a) Only 1 d) Tunisia

b) Only 2 17. The currency of the proposed European c) Both 1 and 2 Monetary Union will be d) Neither 1 nor 2 a) Dollar

13. Which one of the following regions of b) Euro the world supplies the maximum of our c) Guilder d) Mark www.laex.in Page No. 119 https://elearn.laex.in

www.laex.in

ECONOMY UPSC

Previous Year Questions

11.2 Balance of Payment (BOP) b) 2 and 3 c) 1 and 3 18. Consider the following actions which d) 1, 2 and 4

the Government can take: 1) Devaluing the domestic currency. 21. The balance of payments of a country is a systematic record of 2) Reduction in the export subsidy. a) All import and export transactions of a 3) Adopting suitable policies which attract country during a given period of time, greater FDI and more funds from FIIs. normally a year Which, of the above action/ actions can help b) Goods exported from a country during a in reducing the current account deficit? year a) 1 and 2 c) Economic transaction between the b) 2 and 3 government of one country to another c) 3 only d) Capital movements from one country to d) 1 and 3 another

19. Assertion (A): 'Balance of Payments' 22. Which of the following constitute represents a better picture of a country's Capital Account? economic transactions with the rest of the 1) Foreign Loans world than the 'Balance of Trade'. 2) Foreign Direct Investment Reason (R): 'Balance of Payments' takes 3) Private Remittances into account the exchange of both visible

and invisible items whereas 'Balance of 4) Portfolio Investment Trade' does not. Select the correct answer using the codes a) Both A and R are individually true and R given below. is the correct explanation of A a) 1, 2 and 3 b) Both A and R are individually true but R is b) 1, 2 and 4 not the correct explanation of A c) 2, 3 and 4 c) A is true but R is false d) 1, 3 and 4

d) A is false but R is true 11.3 Foreign Investment

20. With reference to Balance of Payments, 11.3.1 FDI

which of the following constitutes / 23. A great deal of Foreign Direct constitute the Current Account? Investment (FDI) to India comes from 1) Balance of trade Mauritius than from many major and 2) Foreign assets mature economies like UK and France. 3) Special Drawing Rights Why? 4) Balance of invisibles a) India has preference for certain countries Select the correct answer using the code as regards receiving FDI given below. a) 1 only www.laex.in Page No. 120 https://elearn.laex.in

www.laex.in

ECONOMY UPSC

Previous Year Questions

b) India has double taxation avoidance b) Services sector agreement with Mauritius c) Food processing c) Most citizens of Mauritius have ethnic d) Telecommunication

identity with India and so they feel secure 27. Consider the following statements: to invest in India 1) The maximum limit of shareholding of Indian d) Impending dangers of global climatic promoters in private sector banks in India is change prompt Mauritius to make huge 49% of paid up capital. investments in India 2) Foreign Direct Investment up to 49 per cent from all sources is permitted in private sector 24. Global capital-flows to developing banks in India under the automatic route. countries increased significantly during Which of these statements is/are correct? the nineties. In view of the East Asian a) Only 1 Financial crisis and Latin American b) Only 2 experience, which type of inflow is good c) Both 1 and 2 for the host country? d) Neither 1 nor 2 a) Commercial loans 28. With reference to Government of India's b) Foreign Direct Investment decisions regarding Foreign Direct c) Foreign Portfolio Investment Investment (FDI) during the year 2001-02, d) External Commercial Borrowings consider the following statements:

25. Which of the following would include 1) Out of the 100% FDI allowed by India in tea Foreign Direct Investment in India? sector, the foreign firm would have to 1) Subsidiaries of foreign companies in India disinvest 33% of the equity in favour of an 2) Majority foreign equity holding in Indian Indian partner within four years. companies 2) Regarding the FDI in print media in India, 3) Companies exclusively financed by foreign the single largest Indian shareholder should companies have a holding higher than 26%. 4) Portfolio investment Which of these statements is/are correct? Select the correct answer using the codes given a) Only 1 below: b) Only 2 a) 1, 2, 3 and 4 c) Both 1 and 2 b) 2 and 4 only d) Neither 1 nor 2 c) 1 and 3 only 29. The largest share of Foreign Direct d) 1, 2 and 3 only Investment (1997- 2000) went to

26. In the last one decade, which one among a) Food and food-product sector the following sectors has attracted the b) Engineering sector highest Foreign Direct Investment inflows c) Electronics and electric equipment sector into India? d) Services sector a) Chemicals other than fertilizers

www.laex.in Page No. 121 https://elearn.laex.in

www.laex.in

ECONOMY UPSC

Previous Year Questions

11.3.2 FII b) Fuller capital account convertibility c) Foreign exchange reserves 30. Both Foreign Direct Investment (FDI)

and Foreign Institutional Investor (FII) d) Effect of oil-prices on the Indian economy

are related to investment in a country. 34. Consider the following statements: Full Which one of the following statements convertibility of the rupee may mean best represents an important difference 1) Its free float with other international between the two? currencies. a) FII helps bring better management skills 2) Its direct exchange with any other and technology, while FDI only brings in international currency at any prescribed capital place inside and outside the country. b) FII helps in increasing capital availability 3) It acts just like any other international in general, while FDI only targets specific currency. sectors Which of these statements are correct? c) FDI flows only into the secondary market, a) 1 and 2 while FII targets primary market b) 1 and 3 d) FII is considered to be more stable than c) 2 and 3 FDI d) 1, 2 and 3

31. Participatory Notes (PNs) are associated 35. Capital Account Convertibility of the with which one of the following? Indian Rupee implies a) Consolidated Fund of India a) That the Indian Rupee can be exchanged b) Foreign Institutional Investors by the authorized dealers for travel c) United Nations Development Programme b) That the Indian Rupee can be exchanged d) Kyoto Protocol for any major currency for the purpose of 32. Which of the following is issued by trade in goods and services registered foreign portfolio investors to c) That the Indian Rupee can be exchanged overseas investors who want to be part for any major currency for the purpose of of the Indian stock market without trading financial assets registering themselves directly? d) None of the above

a) Certificate of Deposit 36. Consider the following statements: The b) Commercial Paper Indian Rupee is fully convertible c) Promissory Note 1) In respect of Current Account of Balance d) Participatory Note of Payment

11.4 Currency Convertibility 2) In respect of Capital Account of Balance of Payment 33. Tarapore Committee was associated 3) Into gold with which one of the following? Which of these statements is/are correct? a) Special Economic Zones a. 1 alone www.laex.in Page No. 122 https://elearn.laex.in

www.laex.in

ECONOMY UPSC

Previous Year Questions

b. 3 alone 40. Which one of the following is not the most c. 1 and 2 likely measure the Government/RBI takes d. 1, 2 and 3 to stop the slide of Indian rupee? a) Curbing imports of non-essential goods and 37. Convertibility of rupee implies promoting exports a) Being able to convert rupee notes into gold b) Encouraging Indian borrowers to issue rupee b) Allowing the value of rupee to be fixed by denominated Masala Bonds market forces. c) Easing conditions relating to external c) Freely permitting the conversion rupee to commercial borrowing other currencies and vice-versa. d) Following an expansionary monetary policy d) Developing in international market for 41. Consider the following statements: currencies in India.

The price of any currency in international 11.5 Exchange Rate market is decided by the

38. Consider the following statements: a) World Bank 1) Purchasing Power Parity (PPP) exchange b) Demand for goods/services provided by rates are calculated by comparing the prices the country of the same basket of goods and services in c) Stability of the government of the different countries. concerned country 2) In terms of PPP dollars, India is the sixth d) Economic potential of the country in largest economy in the world. question Which of the statements given above is / are Which of the statements given above are correct? correct? a) 1 only a. 1, 2, 3 and 4 b) 2 only b. 2 and 3 only c) Both 1 and 2 c. 3 and 4 only d) Neither 1 nor 2

d. 1 and 4 only 39. Assertion: Devaluation of a currency may 42. In the context of India, which of the promote export. following factors is/ are Reason: Price of the country's products in the international Market may fall due to contributor/contributors to reducing devaluation. the risk of a currency crisis? a) Both A and R are individually true and R is 1) The foreign currency earnings of India's IT the correct explanation of A sector b) Both A and R are individually true but R is 2) Increasing the government expenditure not the correct explanation of A 3) Remittances from Indians abroad c) A is true but R is false Select the correct answer using the code d) A is false but R is true given below.

a) 1 only

www.laex.in Page No. 123 https://elearn.laex.in

www.laex.in

ECONOMY UPSC

Previous Year Questions

c) 2 only 46. Which one of the following statements b) 1 and 3 only. is correct with reference to FEMA in d) 1, 2 and 3 India?

43. The problem of international liquidity is a) The Foreign Exchange Regulation Act related to the non-availability of (FERA) was replaced by Foreign Exchange a) Goods and services Management Act (FEMA) in the year 2001 b) Gold and silver b) FERA was given a sunset clause of one year c) Dollars and other hard currencies till 31st May, 2002 to enable Enforcement d) Exportable surplus Directorate to complete the investigation of pending issues 44. Consider the following statements: The c) Under FEMA, violation of foreign exchange price of any currency in international rules has ceased to be a criminal offence market is decided by the d) As per the new dispensation, Enforcement 1) World Bank Directorate can arrest and prosecute the 2) Demand for goods/services provided by people for the violation of foreign exchange the country concerned rules

3) Stability of the government of the concerned country 11.6 Forex Reserves

4) Economic potential of the country in 47. Which one of the following groups of question. items is included in India's foreign- Choose the correct of these statements exchange reserves? a) 1, 2, 3 and 4 are correct a) Foreign-currency assets, Special Drawing b) 2 and 3 are correct Rights (SDRS) and loans from foreign c) 3 and 4 are correct countries d) 1 and 4 are correct b) Foreign-currency assets, gold holdings of 45. Which one of the following is the the RBI and SDRS correct sequence of decreasing order of c) Foreign-currency assets, loans from the the given currencies in terms of their World Bank and SDRS value in Indian Rupees? d) Foreign-currency assets, gold holdings of a) US dollar, Canadian dollar, New Zealand the RBI and loans from the World Bank

dollar, Hong Kong dollar 48. Consider the following statements: b) US dollar, New Zealand dollar, Canadian 1) During the year 2004, India's foreign dollar, Hong Kong dollar exchange reserves did not exceed the 125 c) US dollar, Hong Kong dollar, New Zealand billion US Dollar mark. dollar, Canadian dollar 2) The series of index numbers of wholesale d) US dollar, Hong Kong dollar, Canadian prices introduced from April, 2000 has the dollar, New Zealand dollar year 1993-94 as base year.

www.laex.in Page No. 124 https://elearn.laex.in

www.laex.in

ECONOMY UPSC

Previous Year Questions

Which of the statements given above is/are Educational Objective: To know about the correct? general trends of exports and imports in India. a) 1 only And also the share of exports in the GDP of the b) 2 only country.

c) Both 1 and 2 3. Answer: D d) Neither 1 nor 2 Explanation: A trade deficit occurs when a

11.Key and Explanations country's imports exceeds its exports. In this

situation, there is an outflow of domestic 11.1 Trade currency to foreign markets. Here, 11.1.1 Export Related Trade the balance of trade is said to be negative or

1. Answer: A unfavourable. Informally, there is said to be Explanation: The visit through foreign nationals a trade gap. in India toward witnessing of xix common India’s trade export in 2002 was 50098 million wealth games amounted to export. Since the and imports were 57453 million which shows a commonwealth games were held in India, and trade deficit of approx. 7453 million and hence various commodity or goods and services made statement 1 is incorrect. in the country were purchased by the foreign Educational Objective: To have knowledge nationals witnessing the CWG India which about current trends of trade deficit of India resulted in flow of foreign currency in India. with other countries and also about the value of Therefore, it may be termed as export. exports of India.

Educational Objective: To have knowledge 4. Answer: C about the currency transaction with foreign Explanation: As per current figures countries. Also to know which transaction statement 1 is incorrect as the CAGR between amounts to exports and imports. 2013-2018 is 10.49% Statement 2 is correct as 2. Answer: D the software and service industry in India Explanation: The value of exports from India in registered an overall growth of about 28% in 2003 was 59,361 million and in 2004 was rupee terms during the year 2001-2002 75,904 million, Hence statement 1 is in correct. Educational Objective: To have knowledge Statement 2 is incorrect as the share of relating to exports of India in various sectors in merchandise export of India did not cross 2% the past decade

NOTE: present data India's share in global 5. Answer: B trade (merchandise and services) was 2.1% Explanation: The total marine exports from (481.74 USD billion out of total 23,044 USD 1997-2000 was the highest, Hence option B is billion) for exports and 2.6% (600.62 USD billion correct out of total 23,112 USD billion) for imports in Ite 1997- 199 199 2000-

2017. m 98 8-99 9-00 01 Total Quanti 385818. 3029 3430 44047

www.laex.in Page No. 125 https://elearn.laex.in

www.laex.in

ECONOMY UPSC

Previous Year Questions

ty in 00 34.0 31.0 3.00 11.1.2 Import Related Trade ton 0 0 9. Answer : B Value 4626 5116 6443. in 4697.48 Explanation: Import of Fresh fruits is among .86 .67 89 Crore the highest imports in terms of value in the last US$ 1105 1187 1416. five years. Hence option b is correct 1295.86 Million .49 .87 32 Educational Objective: To know about the Educational Objective: To have knowledge general trends in imports of the country in the about the share of various sectors and goods in past decade and also the share of various the exports of the country commodities in the imports.

6. Answer: A 10. Answer: D Explanation: The rate of growth of India’s Explanation: Import Cover measures the export has enormous growth since 1991 as number of months of imports that can be India brought New Economic Policy of LPG and covered with foreign exchange reserves available because of Devaluation. Hence statement 1 is with the central bank of the country. Eight to correct and statement 2 is incorrect ten months of import cover is essential for Educational Objective: to know about the the stability of a currency. Hence Option D is contribution of exports in the growth of a correct country and also about the initiatives of the Educational Objective: To have knowledge government in order to boost the exports about the various terminologies used in the

7. Answer : C foreign trade of a country

Tata Consultancy Service was the largest IT 11. Answer : B software and services exporter in India Explanation: Statement 1 is incorrect as India during the year 2002-03. Infosys Technologies doesn’t have any such policy to have its reserves stands second. for future. Educational objective: To learn about largest Statement 2 and 3 are correct as majority of IT software and services exporters in India. electricity produced in India from Thermal

8. Answer : C power plants which require huge amount of coal inputs which cannot be supplied from Indian Explanation: Tata Consultancy Service was the resources and also steel plants require huge largest IT software and services exporter in amount of coking coal which we import from India during the year 2002-03. Infosys Australia. Technologies stands second. Educational objective: To learn about largest IT software and services exporters in India.

www.laex.in Page No. 126 https://elearn.laex.in

www.laex.in

ECONOMY UPSC

Previous Year Questions

Educational Objective: to have knowledge General of Foreign Trade) is the main governing about coal industry in India. Also, the body in matters related to Exim Policy. geographical distribution of . The General Agreement on Tariffs and Trade (GATT) is a legal agreement between many 12. Answer : D countries, whose overall purpose was to Explanation: The question is related to the promote international trade by reducing or current affairs of 2000-01. During the given eliminating trade barriers such as tariffs or period pearls, precious and semi precious quotas. stones had the highest share in the imports of Educational Objective: To have knowledge on India. how India became an open economy and the Explanation: Statement 1 is correct background of the situation in the country. Also Statement 2 is incorrect as India’s export in year about the role of WTO in the LPG reforms. 2000 was 42538 million and in year 2001 was

43878 million 15. Answer: B Educational Objective: To have knowledge Explanation: India is currently USA’s 9th about the export and import trends in the oil largest goods trading partner with $87.9 billion industry of India in total (two way) goods trade during 2018. 13. Answer: D Goods exports totaled $33.5 billion; goods Explanation: Imports from Europe in India imports totaled $54.3 billion. averaged 195.74 INR Billion from 1991 until The U.S. goods trade deficit with India was 2019, reaching an all-time high of 605.30 INR $20.8 billion in 2018. Hence option A is correct Billion in May of 2019. Hence option D is Educational Objective: To be updated with the correct. As far as a country is concerned, India trends of India’s trade with the foreign countries

imports highest from China. 16. Answer : A Educational Objective: To have knowledge Currency of U.A.E is Dinar about the share of exports and imports of Sudan’s currency is Sudanese Pound various commodities of India with the trading Yugoslavia’s currency is Yugoslav Dinar countries. Currency of Tunisia is Tunisian Dinar. 11.1.3 Miscellaneous Educational objective: To learn about currency names.

14. Answer: B Explanation: Option B is correct as both the 17. Answer : B above statements are individually true but not The Economic and Monetary Union (EMU) the correct explanation of statement A. represents a major step in the integration of EU India New Foreign Trade Procedure 2015- economies launched in 1992. EMU involves the 2020. Exim Policy or Foreign Trade Policy is a coordination of economic and fiscal policies, a set of guidelines and instructions established by common monetary policy, and a common the DGFT in matters related to the import and currency, the euro. export of goods in India. ... DGFT (Directorate www.laex.in Page No. 127 https://elearn.laex.in

www.laex.in

ECONOMY UPSC

Previous Year Questions

The Economic and Monetary Union (EMU) is Educational Objective: to have knowledge of an umbrella term for the group of policies aimed balance of payment (BOP) and balance of trade at converging the economies of member states of and the differences between them.

the European Union. 20. Answer : C Educational objective: To learn about Explanation: The balance of payments (BoP) Economic and Monetary Union (EMU) and its record the transactions in goods, services, and common currency. assets between residents of a country with the

11.2 Balance of Payment (BOP) rest of the world for a specified time period

typically a year. 18. Answer: D Educational Objective: to have knowledge Explanation: Current account deficit is excess about the components of balance of payment of imports over exports, after devaluation of

domestic currency, domestic goods would 21. Answer: A become cheaper which will create additional Explanation: There are two main accounts in demand for countries products in the world the BoP – the current account and the capital market which may increase the exports and account. Current Account: The current decrease the current account deficit. account records exports and imports in goods, Rising capital inflow through FDI and FII may trade in services and transfer payments. appreciate domestic currency and worsen a Capital Account: The capital account records country’s current account by increasing more all international purchases and sales of assets imports and reducing exports. FDI and FII such as money, stocks, bonds, etc. It includes represent non debt liabilities foreign investments and loans. Hence statement 1 and 3 are correct Educational Objective: To know about the basics of Current Account Deficit and the measures that the government takes to reduce the CAD

19. Answer: A Educational Objective: to know Explanation: BOP of nation consists of visible comprehensively about the balance of payment account, invisible account and capital account. 22. Answer : B Whereas BOT =export of goods – import of Explanation: The balance of payments of a goods. if the country’s imports are more than country is a systematic record of the balance of exports than the deficit can be made up by payments (BOP) of a country is the record of all invisibles like remittance by NRI and FDI economic transactions between the residents of investment . Hence A and R are individually true a country and the rest of the world in a and R is the correct explanation of A particular period (over a quarter of a year or more commonly over a year). Hence option A is

www.laex.in Page No. 128 https://elearn.laex.in

www.laex.in

ECONOMY UPSC

Previous Year Questions

the correct answer. Option B is correct as notice. It is not debt creating. Hence option is capital account consists of Foreign Loans, correct Foreign Direct Investment, Portfolio Investment. Educational Objective: To have knowledge about the foreign direct investment and its benefits in the growth of a country

25. Answer : D Explanation: It is an investment from a party in one country into a business or corporation in Educational Objective: to know about another country with the intention of components of BOP i.e. current account and establishing a lasting interest. capital account. And also, about what Lasting interest differentiates FDI from foreign constitutes these components portfolio investments, where investors passively

11.3 Foreign Investment hold securities from a foreign country. Foreign direct investment can be made by 11.3.1 FDI expanding one’s business into a foreign country 23. Answer: A or by becoming the owner of a company in Explanation: The DTAA, or Double Taxation another country. Avoidance Agreement is a tax treaty signed Broadly, foreign direct investment includes between India and another country (or any "mergers and acquisitions, building new two/multiple countries) so that taxpayers can facilities, reinvesting profits earned from avoid paying double taxes on their income overseas operations, and intra company loans". earned from the source country as well as the FDI is the sum of equity capital, long-term residence country. India has a DTAA with capital, and short-term capital as shown in the Mauritius hence Foreign Direct Investment (FDI) balance of payments. to India comes from Mauritius than from many Hence option D is correct major and mature economies like UK and Educational Objective: to know what France. Therefore option B is correct constitutes FDI and how it is different from Educational Objective: to know about the foreign portfolio investment trends of investment that India receives from the

foreign countries 26. Answer : D Explanation : “FDI in the telecom sector has 24. Answer : B jumped nearly five times in the last 3 years – Explanation: FDI is defined as investment by from $1.3 billion in 2015-16 to $6.2 billion in foreign multinational or its subsidiary or a 2017-18,” .The government has already foreign company in terms of setting up a project increased FDI in the telecom space to 100% and in another country either by way of 100% the sector is witnessing steadily growth since subsidiary or by way of joint venture. Thus FDI then. Hence option D is correct as 100 % FDI is investment is long term in nature. Thus, it allowed in telecom sector in India cannot be pulled out of the country at short www.laex.in Page No. 129 https://elearn.laex.in

www.laex.in

ECONOMY UPSC

Previous Year Questions

Educational Objective: To know about the enterprise. It is termed as a direct investment trends in FDI inflow into the country and share because the investor company looks for a of the various sectors of economy substantial amount of management control or

27. Answer: B influence over the foreign company. Explanation: The maximum limit of FDI is the considered as one of the primary shareholding of Indian promoters in private means of acquiring external assistance. The sector banks in India is 49% of paid up capital. countries where the availability of finance is However earlier it was 40% which was increased quite low can get finance from developed to 49% in 2002 to provide level playing field. So countries having the good financial condition. in present context answer is C. There are a number of ways through which a Educational Objective: to get updated with the foreign investor can get controlling ownership cap fixed by government of India on FDI into like by way of merger or acquisition, by various sectors purchasing shares, by participating in a joint venture or by incorporating a wholly owned 28. Answer : B subsidiary. Explanation: Out of the 100% FDI allowed by Definition of FII India in tea sector, the foreign firm would have FII is an abbreviation used for Foreign to disinvest 26% of the equity in favour of an Institutional Investor, are the investors that pool Indian partner within five years. their money to invest in the assets of the Educational Objective: To get updated with the country situated abroad. It is a tool for making cap fixed by government of India on FDI into quick money for the investors. Institutional various sectors

investors are companies that invest money in 29. Answer : B the financial markets in the country based Explanation: During the given period i.e. 1997- outside the investor country. It needs to get 2000 highest share of FDI went into Engineering itself registered with the securities exchange Sector board of the respective country for making the Educational Objective: to know about the investment. It includes banks, mutual funds, trends in FDI inflow into the country and share insurance companies, hedge funds, etc. of the various sectors of economy FII plays a very crucial role in any country’s

11.3.2 FII economy. Market trend moves upward when any 30. Answer : B foreign company invests or buys securities, and Explanation: Definition of FDI similarly, it goes down if it withdraws the Foreign Direct Investment shortly known as FDI investment made by it. refers to the investment in which foreign funds Key Differences Between FDI and FII are brought into a company based in a The significant differences between FDI and FII different country from the investor company’s are explained below: country. In general, the investment is made to  Foreign Direct Investment or FDI is defined gain a long lasting interest in the investee as the investment made by a company in www.laex.in Page No. 130 https://elearn.laex.in

www.laex.in

ECONOMY UPSC

Previous Year Questions

the company situated outside the the overseas investors who want to invest in the country. Foreign Institutional Investor or stock markets in India, without being registered FII is when investors, most commonly in under SEBI. In the year 2007, the percentage of the form of institutions that invest in the participatory notes in FIIs were almost 50%. country’s financial market. The participatory notes are also regarded as  FII is a way to to make quick money, the offshore derivative instruments as they are used entry and exit to the stock market are by the investors abroad but not within India. very easy. On the other hand, the entry They are used by the clients of and exit are not easy in FDI. Foreign Institutional Investors (FIIs) who do not  FDI brings long-term capital in the wish to participate directly in the Indian stock investee company whereas FII may bring market. long or short term capital in the country. Educational Objective: To know about the  In the case of FDI, there is the transfer of channels through which an investor can invest funds, resources, technology, strategies, as FII know-how. Conversely, FII involves the 32. Answer: D transfer of funds only. Explanation: Foreign institutional investors  FDI increases job opportunities, (FIIs), issue the financial instruments to infrastructural development in the investors in other countries who want to invest investee country and thus leads to in Indian securities. economic growth, which is not in the An FII is an investor or investment case of FII. fund registered in a country outside of the one  FDI results in the increase in the country’s in which it is investing. productivity. As opposed to FII that This system lets unregistered overseas investors results in the increase in the country’s buy Indian shares without the need to register capital. with the Indian regulatory body. These  FDI targets a particular company, but FII investments are also beneficial to India. They does not target a particular company. provide access to quick money to the Indian  FDI obtains management control in the capital market. Because of the short-term company. However, FII does not enable nature of investing, regulators have fewer such control. guidelines for foreign institutional investors. To Hence option B is the correct answer invest in the Indian stock markets and to avoid Educational Objective: to know the difference the cumbersome regulatory approval process, these investors trade participatory notes. Hence, between FDI and FII option D is correct 31. Answer: B Educational Objective: To have knowledge Explanation: Participatory Notes which are also about how foreign portfolio investors can invest regarded as P-Notes (PNs) are the instruments into an economy.

issued by registered FIIs. The PNs are issued for www.laex.in Page No. 131 https://elearn.laex.in

www.laex.in

ECONOMY UPSC

Previous Year Questions

11.4 Currency Convertibility 35. Answer : C Explanation: Capital account convertibility 33. Answer: B refers to a liberalization of a country's capital Explanation: The Committee on Capital transactions such as loans and investment, Account Convertibility (CAC) or Tarapore both short term and long term as well as Committee was constituted by the Reserve Bank speculative capital flows. In a way, capital of India for suggesting a roadmap on full account convertibility removes all the restrains convertibility of Rupee on Capital Account. The on international flows on India's capital committee submitted its report in May account. It means that the Indian Rupee can be 1997. The committee observed that there is no exchanged for any major currency for the clear definition of CAC. The CAC as per the purpose of trading financial assets. Hence standards, refers to the freedom to convert the option C is correct. local financial assets into foreign financial Educational Objective: To know the concept of assets or vice versa at the market determined capital account and current account rates of exchange. convertibility of Indian Rupees The Tarapore committee observed that the

Capital controls can be useful in insulating the 36. Answer: A economy of the country from the volatile capital Explanation: The Indian Rupee is fully flows during the transitional periods and also in convertible in respect of Current Account of providing time to the authorities, so that they Balance of Payment and not in respect of can pursue discretionary domestic policies to Capital Account of Balance of Payment and not strengthen the initial conditions. in gold. Hence only statement 1 is correct and Hence option B is correct statement 2 and 3 are incorrect Educational Objective: To know about various Educational Objective: to know what is full committees’ setup on the full capital account convertibility in current account and capital convertibility of Indian economy. account.

34. Answer: A 37. Answer : C Explanation: The full convertibility means no Explanation: Convertibility of rupee means that RBI dictated rates and there is a unified market those who have foreign exchange (e.g. US determined exchange rate regime. Encouraged dollars, Pound Sterlings etc.) can get them with the success of the LERMS, the government converted into rupees and vice-versa at the introduced the full convertibility of Rupee in market determined rate of exchange. Rupee is Trade account (means only merchandise trade both convertible on capital account and current no service trade) from March 1993 onwards. account. Hence Option C is correct answer Hence statement 1 and 2 are correct. Educational Objective: To know the concept of Educational Objective: To know what is full convertibility of Indian rupee convertibility in current account and capital

account

www.laex.in Page No. 132 https://elearn.laex.in

www.laex.in

ECONOMY UPSC

Previous Year Questions

11.5 Exchange Rate (Mainly Currency Educational Objective: to know about the Devaluation) appreciation and depreciation of the currency. The steps that central bank or government takes 38. Answer: A to stop the excessive depreciation of rupee. Explanation: PPP is an economic theory that

compares different countries' currencies 41. Answer: B through a “basket of goods” approach. Explanation: The price of any currency According to this concept, two currencies are in is international market is determined by the equilibrium—known as the currencies being at forces of demand and supply. Demand for a par—when a basket of goods is priced the same currency increases if the demand for the goods in both countries, taking into account the and services provided by that country increases. exchange rates. Hence, statement 1 is correct Moreover, a stable Govt gives confidence to Educational Objective: To know the concept of investors and raises the demand for currency of purchasing power parity and method of its that country. Hence Statement 2 and 3 are calculation. And also the status of India with correct. respect to PPP. Educational Objective: to know how a currency appreciates or depreciates based on 39. Answer: A the market forces andalso the role of various Explanation: Devaluation is a deliberate organisations.

downward adjustment to the value of country’s currency relative to another currency. It is 42. Answer : B monetary policy tool used by countries that Explanation: A currency crisis results in a have a fixed exchange rate or semi fixed sharp depreciation of domestic currency and exchange rate. After devaluation of the currency may force the authorities to sell foreign the country’s goods become cheaper for exchange reserves and take measures (for e.g. foreigners and become more competitive in a raise domestic interest rates) to defend the global market. Thus leads to increase in exports. downfall in currency. Hence A is correct and R is the correct S1: If a country has a strong export explanation of it. performance, a currency crisis is difficulty to Educational Objective: To know the concept of occur. Since S1 leads to greater exports, it devaluation and the effects of devaluation on will reduce the risk. exports and imports of the country S2: Increase in government expenditure may actually increase imports and adversely affect 40. Answer: D the currency crisis. Usually, austerity in public Explanation: Following expansionary monetary spending is one of the measures prescribed to policy by RBI will increase the flow of money in contain a foreign exchange or currency crisis the market thereby increasing the inflation S3: Greater flow of remittances would help avert which will again result in deprecation of rupee. the currency crisis since greater inflow of foreign Hence option D is correct currency (which is converted into rupees by the

www.laex.in Page No. 133 https://elearn.laex.in

www.laex.in

ECONOMY UPSC

Previous Year Questions

recipient of the remittance in India) raises the 45. Answer : A demand for domestic currency. Explanation: USD = Rs. 91.55, Canadian dollar Decline in the value of a country’s currency is = Rs. 53.97, Newzealand dollar = Rs.46.04, known as Currency Crisis that negatively affects Hongkong Dollar= Rs. 9.21 the economy by creating instabilities in currency Educational Objective: To be updated with the exchange rates. The foreign currency earnings of international market of currencies and how India’s Information Technology (IT) sector and rupee values with other major currencies.

remittances from Indians abroad together can 46. Answer : C reduce the risk of a currency crisis in India. Explanation: FERA was enacted in September Educational Objective: To know the steps that 1973 and it came in force from January 1, can be taken during the currency crisis either 1974. It was amended by the Foreign Exchange by the central bank or government. Regulation (Amendment) Act 1993 and later in

43. Answer: C 2000, was replaced by FEMA. Explanation: The problem of international Under FEMA, violation of foreign exchange rules liquidity is concerned with the imbalances in the has ceased to be a criminal offence and would now demand for and supply of international liquidity. be treated as a civil offence and the Enforcement As dollar is the dominating component in the Directorate would no longer have the power to forex, the problem of international liquidity is arrest persons for such offences. related to the non-availability of dollar and other Educational objective: To learn on Foreign hard currencies. Hence option C is correct Exchange Management Act (FEMA).

Educational Objective: to have knowledge about the various terminologies used in the 11.6 Forex Reserves

currency exchange market. 47. Answer: B

44. Answer: B Explanation: The Forex Reserves (‘foreign Explanation: The price of any currency is exchange reserves’) of an economy is its ‘foreign international market is determined by the forces currency assets’ added with its gold reserves, of demand and supply. Demand for a currency SDRs (Special Drawing Rights) and Reserve increases if the demand for the goods and Tranche in the IMF. services provided by that country increases. Reserve tranche is a portion of the required Moreover, a stable Govt gives confidence to quota of currency each member country must investors and raises the demand for currency of provide to the International Monetary Fund that country. Hence option B is correct. (IMF) that can be utilized for its own purposes. Educational Objective: to have knowledge Educational Objective: To know how the forex about how currency appreciates or depreciates reserves are maintained and what constitutes it.

depending on the situation. Whether central 48. Answer : C banks involve in deciding the value of the Explanation: Both statements are correct: currency. During the year 2004, India's foreign exchange

www.laex.in Page No. 134 https://elearn.laex.in

www.laex.in

ECONOMY UPSC

Previous Year Questions

reserves did not exceed the 125 billion US Dollar b) World Trade Organization mark. The series of index numbers of wholesale c) ASEAN- India Free Trade Agreement prices introduced from April, 2000 has the year d) G-20 Summits

1993-94 as base year. 4. NAMA-11 (Nama-11) group of countries Educational Objective: To know about the frequently appears in the news in the current forex reserves with India and the context of the affairs of which one of benefits of having high forex reserves. the following? 12. International Organisation a) Nuclear Suppliers Group

12.1 WORLD TRADE ORGANIZATION b) World Bank (WTO) c) World Economic forum

1. In the context of which of the following d) World Trade Organization

do you sometimes find the terms 'amber 5. Consider the following statements: box, blue box and green box' in the 1) The World Intellectual Property news? Organisation (WIPO) is a specialized agency a) WTO affairs of United Nations System of Organisations. b) SAARC affairs 2) WIPO has its headquarters at Rome. c) UNFCCC affairs 3) The Trade Related Aspects of Intellectual d) India-EU negotiations on FTA Property Rights (TRIPS) Agreement is binding on all WTO members. 2. The terms 'Agreement on Agriculture', 4) Least developed country members of WTO 'Agreement on the Application of are not required to apply the provisions of Sanitary and Phytosanitary Measures' TRIPS Agreement for a period of 20 years and 'Peace Clause' appear in the news from the general date of application of the frequently in the context of the affairs Agreement. of the Which of these statements are correct? a) Food and Agriculture Organization a) 1,2,3 and 4 b) United Nations Framework Conference on Climate Change b) 2,3 and 4 c) World Trade Organization c) 1,2 and 4 d) United Nations Environment Programme d) 1 and 3

3. In the context of the affairs of which of 6. The earlier name of WTO was the following is the phrase "Special a) UNCTAD Safeguard Mechanisms" mentioned in b) GATT the news frequently? c) UNIDO a) United Nations Environment Programme d) OECD www.laex.in Page No. 135 https://elearn.laex.in

www.laex.in

ECONOMY UPSC

Previous Year Questions

b) International Federation of Standards Users 7. The economist who was associated with the WTO draft document is: c) International Organisation for Standardization a) AK Sen d) World Standards Cooperation b) TN Srinivasan

c) JN Bhagwati 10. Consider the following statements: d) Avinash Dixit The Ministerial Meeting of WTO held in

December 1999 was unsuccessful because it 8. In order to comply with TRIPS attempted to link trade with Agreement, India enacted the 1) labour related issues Geographical Indications of Goods 2) environment related issues (Registration & Protection) Act, 1999. The difference /differences between a 3) terrorism related issues "Trade Mark" and a Geographical 4) debt related issues Indication is/are Which of these statements are correct? 1) A Trade Mark is an individual or a a) 1, 3 and 4 company's right whereas a Geographical b) 1 and 2 Indication is a community's right c) 2 and 3 2) A Trade Mark can be licensed whereas a d) 2 and 4

Geographical Indication cannot be licensed 3) A Trade Mark is assigned to the 11. India enacted The Geographical manufactured goods whereas the Indications of Goods (Registration and Geographical Indication is assigned to the Protection) Act, 1999 in order to agricultural goods/products and comply with the obligations to (WTO) handicrafts only a) ILO b) IMF Which of the statements given above is/are c) UNCTAD correct? d) WTO

a) 1 only b) 1 and 2 only 12. Consider the following statements: (WTO) c) 2 and 3 only 1) India has ratified the Trade Facilitation d) 1,2 and 3 Agreement (TFA) of WTO.

9. As regards the use of international food 2) TFA is a part of WTO's Bali Ministerial safety standards as reference point for Package of 2013. the dispute settlements, which one of 3) TFA came into force in January 2016. the following does WTO collaborate Which of the statements given above is/are with? correct? a) Codex Alimentarius Commission a) 1 and 2 only www.laex.in Page No. 136 https://elearn.laex.in

www.laex.in

ECONOMY UPSC

Previous Year Questions

b) 1 and 3 only 2) The World Bank participates as observer in c) 2 and 3 only IMFC's meetings, d) 1, 2 and 3 Which of the statements given above is/are

13. With reference to the 'National correct? Intellectual Property Rights Policy', a) 1 only consider the following statements: b) 2 only (2017) (WTO) c) Both 1 and 2 1) It reiterates India's commitment to the Doha d) Neither 1 nor 2 Development Agenda and the TRIPS

Agreement. 16. 'Global Financial Stability Report' is 2) Department of Industrial Policy and prepared by the Promotion is the nodal agency for regulating a) European Central Bank intellectual property rights in India. b) International Monetary Fund Which of the above statements is/are c) International Bank for Reconstruction and correct? Development a) 1 only d) Organization for Economic Cooperation b) 2 only and Development

c) Both 1 and 2 17. Which of the following organizations

d) Neither 1 nor 2 brings out the publication known as

12.2 INTERNATIONAL MONETARY FUND 'World Economic Outlook’? (IMF) a) The International Monetary Fund 14. Recently, which one of the following b) The United Nations Development currencies has been proposed to be Programme added to the basket of IMF's SDR? c) The World Economic Forum a) Rouble d) The World Bank b) Rand 18. Regarding the International Monetary c) Indian Rupee Fund, which one of the following d) Renminbi statements is correct?

15. With reference to the International a) It can grant loans to any country Monetary and Financial Committee b) It can grant loans to only developed (IMFC), consider the following countries statements: c) It grants loans to only member countries 1) IMFC discusses matters of concern affecting d) It can grant loans to the central bank of a the global economy, and advises the country

International Monetary Fund (IMF) on the direction of its work. www.laex.in Page No. 137 https://elearn.laex.in

www.laex.in

ECONOMY UPSC

Previous Year Questions

19. Which of the following is/are treated as 2) Singapore Regional Training Institutes is artificial currency? one of the institutes that provides training a) ADR in macroeconomic analysis and policy, and b) GDR related subjects as a part of programme of the IMF Institute. c) SDR Which of the statements given above is/are d) Both ADR and SDR correct?

20. Stiglitz Commission established by the a) 1 only President of the United Nations General b) 2 only Assembly was in the international news. c) Both 1 and 2 The commission was supposed to deal d) Neither 1 nor 2 with (2010)

a) The challenges posed by the impending 12.3 WORLD BANK GROUP global climate change and prepare a road 23. Which one of the following is not a sub- map index of the World Bank's 'Ease of Doing b) The workings of the global financial Business Index'? systems and to explore ways and means to a) Maintenance of law and order secure a more sustainable global order b) Paying taxes c) Global terrorism and prepare a global action plan for the mitigation of terrorism c) Registering property d) Expansion of the United Nations Security d) Dealing with construction permits Council in the present global scenario

24. With reference to 'IFC Masala Bonds', 21. Who among the following served as the sometimes seen in the news, which of Chief Economist of the International the statements given below is/are Monetary Fund? correct? a) Ashok Lahiri 1) The International Finance Corporation, b) Sumaritra Ghoshal which offers these bonds, is an arm of the c) Saumitra Chaudhuri World Bank. d) Raghuram Rajan 2) They are the rupee-denominated bonds and are a source of debt financing for the public 22. Consider the following statements: and private sector. 1) Poverty Reduction and Growth Facility Select the correct answer using the code (PRGF) has been established by the given below. International Development Association (IDA) a) 1 only to provide further assistance to low-income countries facing high level of indebtedness. b) 2 only c) Both 1 and 2

www.laex.in Page No. 138 https://elearn.laex.in

www.laex.in

ECONOMY UPSC

Previous Year Questions

d) Neither 1 nor 2 d) International Monetary Fund

25. India's ranking in the 'Ease of Doing 29. Consider the following organisation: Business Index' is sometimes seen in 1) International Bank for Reconstruction and the news. Which of the following has Development declared that ranking? 2) International Finance Corporation a) Organization for Economic Cooperation and 3) International Fund for Agricultural Development (OECD) Development b) World Economic Forum 4) International Monetary Fund c) World Bank Which of these are agencies of the United d) World Trade Organization (WTO) Nations?

a) 1 and 2 26. Which one of the following issues the b) 2 and 3 'Global Economic Prospects' report c) 3 and 4 periodically? d) 1,2,3 and 4 a) The Asian Development Bank

b) The European Bank for Reconstruction and 30. The theme of the World Development Development Report 2001, is c) The US Federal Reserve Bank a) From Plan to Market d) The World Bank b) Knowledge for Development

c) Attacking Poverty 27. The International Development Association, a lending agency, is d) The State in the Changing World

administrated by the 31. Match List-I with List-II and select the a) International Bank for Reconstruction & correct answer using the codes given Development below the lists: b) International Fund for Agricultural List-I List-II Development A. WTO 1. Provides loans to c) United Nations Development Programme address short-term d) United Nations Industrial Development balance of payment Organization problems

28. “World Development Report” is an annual B. IDA 2. Multi-lateral trade publication of negotiation body a) United Nations Development Programme C. IMF 3. Sanction of soft loans b) International Bank of Reconstruction and D. IBRD 4. Facilitating lending and Development borrowing for c) World Trade Organisation reconstruction and www.laex.in Page No. 139 https://elearn.laex.in

www.laex.in

ECONOMY UPSC

Previous Year Questions

development 2) China's population is more than the Codes: combined population of any two other A B C D countries a) 2 3 4 1 Which of the statements given above is/are correct? b) 2 3 1 4 a) 1 only c) 3 2 4 1 b) 2 only d) 3 2 1 4

c) Both 1 and 2 32. According to the World Development d) Neither 1 nor 2

Report, low income economies are those for which the per capita GNP in 12.5 WORLD ECONOMIC FORUM 1994 was 35. Who among the following is the founder a) US $ 925 or less of World Economic Forum? b) US $ 825 or less a) Klaus Schwab c) US $ 725 or less b) John Kenneth Galbraith d) US $ 525 or less c) Robert Zoellick 12.4 BRICS d) Paul Krugman

33. Consider the following statements: 36. Which one of the following countries 1) New Development Bank has been set up by occupies the first place in the "Global APEC. Competitive Report" of World Economic 2) The headquarters of New Development Forum? Bank is in Shanghai. a) USA Which of the statements given above is / are b) Singapore correct? c) Hong Kong a) 1 only d) France

b) 2 only 12.6 EUROPEAN UNION c) Both 1 and 2 37. In the context of bilateral trade d) Neither 1 nor 2

negotiations between India and 34. With reference to BRIC countries, European Union, what is the difference consider the following statements: between European Commission and 1) At present, China's GDP is more than the European Council? combined GDP of all the three other 1) European Commission represents the EU in countries trade negotiations whereas European Council participates in the legislation of

www.laex.in Page No. 140 https://elearn.laex.in

www.laex.in

ECONOMY UPSC

Previous Year Questions

matters pertaining to economic policies of 2) India is the largest shareholder in AIIB. the European Union 3) AIIB does not have any members from 2) European Commission comprises the Heads outside Asia. of State or government of member countries Which of the statements given above is/are whereas the European Council comprises of correct? the persons nominated by European a) 1 only Parliament. b) 2 and 3 only Which of the statements given above is/are c) 1 and 3 only correct? d) 1,2 and 3 a) 1 only

b) 2 only 40. Consider the following statements: c) Both 1 and 2 1) The agreement on South Asian Free Trade Area (SAFTA) came into effect from 1st d) Neither 1 nor 2

December, 2005. 38. Consider the following statements about 2) As per SAFTA agreement terms, India, the European Union: Pakistan and Sri Lanka have to decrease 1) The European Union was known earlier as their custom duties to the level of 0 to 5 the European Community percent by the year 2013. 2) The Single European Act (1986) and the Which of the statements given above is/are Maastricht Treaty were milestones in its correct? formation a) 1 only 3) Citizens of European Union countries enjoy b) 2 only dual citizenship c) Both 1 and 2 4) Switzerland is a member of the European Union d) Neither 1 nor 2

Which of the above statements are correct? 41. Result of their annual survey, the a) 2 and 4 National Geographic Society and an b) 1 and 3 international polling firm GlobeScan gave India top rank in Greendex 2009 c) 3 and 4 score. What is this score? d) 1,2 and 3

a) It is a measure of efforts made by different 12.7 OTHER MULTILATERAL countries in adopting technologies for ORGANIZATIONS reducing carbon footprint 39. With reference to Asian Infrastructure b) It is a measure of environmentally Investment Bank (AIIB), consider the sustainable consumer behaviour in different following statements : (2019) countries 1) AIIB has more than 80 member nations. www.laex.in Page No. 141 https://elearn.laex.in

www.laex.in

ECONOMY UPSC

Previous Year Questions

c) It is an assessment of 2) ISO 9000 relates to the quality management programmes/schemes undertaken by system and standards. different countries for improving the 3) ISO 14000 relates to environmental conservation of natural resources management system standards. d) It is an index showing the volume of carbon Which of the statements given above is/are credits sold by different countries correct?

a) 1, 2 and 3 42. Which one of the following is not related to United Nations? b) 3 only a) Multilateral Investment Guarantee Agency c) 2 and 3 b) International Finance Corporation d) None

c) International Centre for Settlement of 46. Consider the following countries Investment Disputes 1) Brazil d) Bank for International Settlements 2) Mexico

3) South Africa 43. Consider the following countries: According to UNCTAD, which of the above a) Brunei Darussalam is/are categorized as "Emerging Economies"? b) East Timor a) 1 only c) Laos b) 1 and 3 only Which of the above is/are member/members c) 2 and 3 only of ASEAN? d) 1,2 and 3 a) 1 only

b) 2 and 3 only 47. Consider the following pairs: c) 1 and 3 only Large Bank Country of Origin d) 1, 2 and 3 1. ABN Amro Bank : USA

2. Barclays Bank : UK 44. Other than Venezuela, which one among 3. Kookmin Bank : Japan the following from South America is a member of OPEC? Which of the above pairs is/are correctly matched? a) Argentina a) 1 only b) Brazil b) 2 only c) Ecuador c) 1 and 2 d) Bolivia d) 2 and 3

45. Consider the following statements: 48. The Asia-Pacific Economic Co-operation 1) The Headquarters of the International Forum was held in the year 2001 in Organization for Standardization are located a) Bangkok in Rome. b) Hong Kong www.laex.in Page No. 142 https://elearn.laex.in

www.laex.in

ECONOMY UPSC

Previous Year Questions

c) Jakarta “Green box” subsidies are therefore allowed d) Shanghai without limits, provided they comply with the policy-specific criteria. 49. Consider the following statements: 1) The Commonwealth has no charter, treaty Amber Box: All domestic support measures or constitution considered to distort production and trade 2) All the territories/countries once under the (with some exceptions) fall into the amber box British empire (jurisdiction/rule/mandate) as all domestic supports except those in the automatically joined the Commonwealth as blue and green boxes. its members These include measures to support prices, or Which of the statements given above is/are subsidies directly related to production correct? quantities. a) 1 only Blue Box: This is the “amber box with b) 2 only conditions”. Such conditions are designed to c) Both 1 and 2 reduce distortion. d) Neither l nor 2 Any support that would normally be in the

amber box is placed in the blue box if the 12. Key and Explanations support also requires farmers to limit production. 12.1 WORLD TRADE ORGANIZATION At present, there are no limits on spending on (WTO) blue box subsidies. 1. Answer : A Hence these terminologies are related to WTO Explanation: Agreement On Agriculture (AoA) Affairs. Hence option A is correct – WTO Educational objective – to have knowledge AoA is aimed to remove trade barriers and to about various terminologies related to World promote transparent market access and Trade Organization integration of global markets 2. Answer : C In WTO terminology, the subsides provided are Explanation: The Agreement on the Application identified with different colour boxes viz. Green of Sanitary and Phytosanitary Measures, also Box Subsidies, Amber box Subsidies and Blue known as the SPS Agreement or just SPS, is an box subsidies. international treaty of the World Trade Green Box: subsidies that do not distort Organization (WTO). trade, or at most cause minimal distortion. Broadly, the sanitary and phytosanitary They are government-funded and must not ("SPS") measures covered by the agreement are involve price support. those aimed at the protection of human, animal They also include environmental protection and or plant life or health from certain risks. regional development programmes.

www.laex.in Page No. 143 https://elearn.laex.in

www.laex.in

ECONOMY UPSC

Previous Year Questions

Trade negotiators generally refer to Article 13 of NAMA negotiations formally began in January the World Trade Organization's Agreement on 2002 after the Negotiating Group on Market Agriculture as the Peace Clause. Access (NGMA) was created. More about Agreement on Agriculture is given It is one of the most active group in the WTO’s in Question Number 1 non-agricultural market access negotiations. Hence these terminologies are related to WTO. Hence option D is correct. Educational objective – to have knowledge Educational objective – to know about the about various terminologies related to World lobbying groups that are active in negotiating Trade Organization with World Trade Organization’s for market

access.

3. Answer : B 5. Answer : D Explanation: WTO’s Special Safeguard Mechanism (SSM) is a protection measure Explanation: The World Intellectual Property allowed for developing countries to take Organization (WIPO) is one of the 17 specialized contingency restrictions against agricultural agencies of the United Nations. imports that are causing injuries to domestic It was created in 1967 “to encourage creative farmers. activity, to promote the protection of intellectual The contingency measure is imposition of tariff property throughout the world.” if the import surge causes welfare loss to the WIPO is headquartered in Geneva, Switzerland. domestic poor farmers. The design and use of Hence 2 is wrong. By elimination option D is the SSM is an area of conflict under the WTO. correct The Special Safeguard Mechanism (SSM) The Trade Related Aspects of Intellectual allowed developing countries to raise import Property Rights (TRIPS) Agreement is binding on duties on agricultural products in response all WTO members. to import surges. Educational objective : to have knowledge Educational objective – to have knowledge about International Organisations that are about various terminologies related to World related to trade and intellectual property.

Trade Organization 6. Answer : B

4. Answer : D Explanation: The World Trade Organization Explanation: The NAMA-11 – is the group of (WTO) is an intergovernmental organization that developing countries with common interests in is concerned with the regulation of international the NAMA trade negotiations. trade between nations. Members are Argentina, Bolivarian Republic of The WTO officially commenced on 1 January Venezuela, Brazil, Egypt, India, Indonesia, 1995 under the Marrakesh Agreement, signed Namibia, Philippines, South Africa and Tunisia. by 123 nations on 15 April 1994, replacing the General Agreement on Tariffs and Trade

www.laex.in Page No. 144 https://elearn.laex.in

www.laex.in

ECONOMY UPSC

Previous Year Questions

(GATT), which commenced in 1948. Hence Educational Objective- to know the option b is correct. fundamental differences between trademark, It is the largest international economic copyright, patency, GI tag etc.

organization in the world. 9. Answer : A Educational objective- to know about the Explanation: Codex Alimentarius A collection origin and organizational structure of World of internationally recognized standards, codes of Trade Organization. practice, guidelines, and other

7. Answer : C recommendations relating to foods, food production, and food safety. Explanation:J.N. Bhagawati is the economist who is associated with the drafting of the WTO. Its texts are developed and maintained by the Codex Alimentarius Commission, a body that He is an American economist who had an was established in early November 1961 by the immense role to play in the reshaping of the Food and Agriculture Organization of the United world economy. Nations(FAO) He did multiple researches on free trade and in The Codex Alimentarius is recognized by the establishing an economy for his advocacy of World Trade Organization as an international international trade. reference point for the resolution of disputes Educational Objective : to have knowledge concerning food safety and consumer about important Indian personalities and their protection. Hence option A is correct. contributions in the International Forum.

Educational objective – to know about dispute 8. Answer : B settlement mechanism in WTO and also various Explanation : Trademark belongs to an standards and procedures followed to settle the individual or company… Whereas, the disputes. Geogrpahical Indication belongs to 10. Answer : B region/location/area, represented by group of Explanation : Ministerial Meeting of WTO companies on behalf of an association/group. 1999, seattle There were two major issues, Hence Option 1 is correct 1. first, whether to start a new comprehensive Trademark can be assigned or licensed to round of negotiations such as the Uruguay anyone in the world while GI cannot be assigned Round or confine negotiations to the so-called or licensed to someone outside that place or "built in agenda" of agriculture and services don’t belong to the group of authorized mandated at the last Ministerial. It also producers; hence 2 is correct. included environment related issues GI tag is also given for manufactured goods. 2.Secondly, what should the negotiations Hence 3 is wrong. encompass, more specifically what should be included in the agenda of the meeting.

www.laex.in Page No. 145 https://elearn.laex.in

www.laex.in

ECONOMY UPSC

Previous Year Questions

The meeting was unable to resolve both The Trade Facilitation Agreement came into issues, and ended in stalemate. force in 2017, when the number of member The deliberations were suspended without countries which ratified the agreement reached agreement on a new round of negotiations and a count of 112. Hence statement 3 is incorrect. without agreement on a ministerial declaration. Educational objective: To learn about the Educational objective - to know about dispute Trade Facilitation Agreement (TFA).

settlement mechanism in WTO and also various 13. Answer : C standards and procedures followed to settle the Explanation : The Union Cabinet approved the disputes. National Intellectual Property Rights (IPR) Policy

11. Answer : D in 2016 that will lay the future roadmap for Explanation : The Geographical Indications of intellectual property in India. Goods (Registration and Protection) Act, The Policy recognizes that India has a well- 1999 (GI Act) is a sui generis Act of established TRIPS compliant legislative, the Parliament of India for protection administrative and judicial framework to of geographical indications in India. India, as a safeguard IPRs, which meets its international member of the World Trade obligations while utilizing the flexibilities Organization (WTO), enacted the Act to provided in the international regime to address comply with the Agreement on Trade-Related its developmental concerns. It reiterates India’s Aspects of Intellectual Property Rights. commitment to the Doha Development Agenda The GI tag ensures that none other than those and the TRIPS agreement. Hence statement 1 is registered as authorised users (or at least those correct. residing inside the geographic territory) are These objectives are sought to be achieved allowed to use the popular product name. through detailed action points. The action by Educational objective: To learn about the different Ministries/ Departments shall be Geographical Indications of Goods (Registration monitored by DIPP which shall be the nodal and Protection) Act, 1999 department to coordinate, guide and oversee

implementation and future development of IPRs 12. Answer : A in India. Hence statement 2 is correct. Explanation : India has formally ratified the Educational objective: To learn about the WTO’s (World Trade Organisation) Trade National Intellectual Property Rights Policy Facilitation Agreement, which aims at easing and its importance.

customs procedures to boost commerce. The main aim of the agreement was to boost global 12.2 INTERNATIONAL MONETARY FUND growth by reducing the cost of the transaction (IMF)

in terms of export and import. 14. Answer : D It was outcome of WTO’s 9th Bali (Indonesia) Explanation : In the review conducted in ministerial package of 2013. November 2015, the IMF decided to add the www.laex.in Page No. 146 https://elearn.laex.in

www.laex.in

ECONOMY UPSC

Previous Year Questions

Renminbi (Chinese yuan) to the basket, across financial and non-financial sectors in effective 1 October 2016. Since that date, the advanced and emerging market economies. XDR basket has consisted of the following five It is released twice per year, in April and currencies: U.S. dollar 41.73%, euro 30.93%, October. Released by International Monetary renminbi (Chinese yuan) 10.92%, Japanese yen Fund. Hence option B is correct 8.33%, British pound 8.09%. hence Option D It draws out the financial ramifications of is correct. economic imbalances highlighted by the Educational objective – to have knowledge IMF’s/World Economic Outlook. about IMF and its objectives. Also about the The GFSR issues recommendations for central IMF’s Special Drawee Rights and its purpose. banks, policymakers and others who supervise

15. Answer : C global financial markets. Explanation : International Monetary and Educational objective – to have knowledge Financial Committee is the Ministerial-level about prominent Reports and Indices released committee of the International Monetary Fund by the international organizations and their (IMF). objectives

It meets twice a year, once during the Fund- 17. Answer : A Bank Annual Meetings in October and once Explanation : World Economic Outlook (WEO) during the Spring Meetings in April. is a survey by the IMF that is usually Functions: published twice a year in the months of April It discusses the management of the and October. Hence option A is correct. international monetary and financial system. It It analyzes and predicts global economic advices the IMF. Hence 1 is correct. developments during the near and medium It advises the IMF on any other matters of term. common concern affecting the global economy. In response to the growing demand for more IMFC has 24 members, drawn from the pool of frequent forecast updates, the WEO Update is 189 governors, and represents all member published in January and July between the two countries. India is one of the current main WEO publications released usually in April members. World bank participates as and October. observer. Hence 2 is correct. Educational objective – to have knowledge Educational Objective - to have knowledge about prominent Reports and Indices released about organization structure, functions, by the international organizations and their objectives of important international objectives

Organizations like IMF, WB etc. 18. Answer : C

16. Answer : B Explanation : IMF came into existence after the Explanation : The GFSR provides an UN conference in Bretton Woods in 1944. assessment of balance sheet vulnerabilities www.laex.in Page No. 147 https://elearn.laex.in

www.laex.in

ECONOMY UPSC

Previous Year Questions

It currently has 189 member countries. A global depositary receipt (GDR) is very similar The most powerful countries in the global to an American depositary receipt (ADR). It is economy have the most voting power. a type of bank certificate that represents shares Its main objectives are foster global monetary in a foreign company, such that a foreign cooperation, secure financial stability, facilitate branch of an international bank then holds the international trade, promote high employment shares. and sustainable economic growth and reduce Educational Objective – to have knowledge poverty around the world. about the international currency and also Its main function is to give financial assistance artificial currency as issued by IMF and their to member countries with balance of advantages in global economy.

payments problems. Countries must embark 20. Answer : B on structural adjustment policies monitored by Explanation : Stiglitz Commission is a short the IMF. It lends only to member countries. name given for two commissions led by the US Hence option C is correct. economist Joseph E. Stiglitz: Educational Objective – to have knowledge Commission of Experts on Reforms of the about organization structure, functions, International Monetary and Financial objectives of important international System, convened by the President of the Organizations like IMF, WB etc. United Nations General Assembly Miguel

19. Answer : C d'Escoto Brockmann Explanation : An American depositary receipt The commission dealt with the workings of the (ADR) is a certificate issued by a U.S. bank that global financial systems and to explore ways represents shares in foreign stock. ADRs trade and means to secure a more sustainable global on American stock exchanges. ADRs and their order. Hence option B is correct dividends are priced in U.S. dollars. Hence Educational Objective – to have knowledge not artificial currency. about the current issues in the United Nations The SDR (Special Drawing Right) is an artificial and various Commissions set up by UNO to "basket" currency used by the IMF resolve the issues.

(International Monetary Fund) for internal 21. Answer : D accounting purposes. The SDR is also used by Explanation : Raghuram Rajan is the Katherine some countries as a peg for their own currency, Dusak Miller Distinguished Service Professor of and is used as an international reserve asset. Finance at University of Chicago Booth School Hence option C is correct of Business. He was the 23rd Governor of the A global depositary receipt (GDR) is a bank Reserve Bank of India (RBI) between September certificate issued in more than one country 2013 and September 2016. Between 2003 and for shares in a foreign company. 2006, Dr. Rajan was the Chief Economist and

www.laex.in Page No. 148 https://elearn.laex.in

www.laex.in

ECONOMY UPSC

Previous Year Questions

Director of Research at the International position. India has improved it’s ranking in 4 Monetary Fund. Hence option D is correct. parameters; starting a business, dealing with Educational Objective – to have knowledge construction permits, trading across borders about the current developments in significant and resolving insolvency. Other parameters Organizations like WTO, IMF, World Bank, UNO are; Electricity availability, Property registration, etc. also to have knowledge about the Credit availability, Protecting minority Investors, achievements of Indians in the international Paying Taxes, Contracts enforcement. forum. Educational objective: To have knowledge about the important reports by world 22. Answer : B organizations. Also know about the India’s rank Explanation :The Poverty Reduction and and performance in it.

Growth Facility (PRGF) is an arm of the International Monetary Fund which lends to 24. Answer : A the world's poorest countries. It was created in Explanation: overseas rupee bonds are known September 1999, replacing the Enhanced as Masala bonds. Any corporate, body corporate Structural Adjustment Facility. Hence option 1 and Indian bank is eligible to issue Rupee is wrong. denominated bonds overseas. These bonds are The IMF – Singapore Regional Training Institute issued for raising funds from overseas (STI), located in Singapore, serves as the market. Hence, statement 2 is wrong. International Monetary Fund’s (IMF) regional First time it was issued by International training center for the Asia-Pacific region. Finance Corporation, an arm of World Bank. It provides training on macroeconomic and Hence, statement 1 is correct. financial management, and related legal and Recently, Kerala issued this. statistical issues, to government officials Educational objective: To know about the from 37 countries. Most training takes place in different types of bonds and their issuing two week seminars, with shorter events agencies.

organized for senior officials. Hence option 2 is 25. Answer : C wrong. Explanation: World bank releases the Ease of Educational objective – to have knowledge Doing Business annually. In 2019 edition, India about the organisations that are working in improved in its rank by 14 places bagging 63rd close co-operation with WTO, IMF, World Bank position. India has improved it’s ranking in 4 Group etc.

parameters; starting a business, dealing with 12.3 WORLD BANK GROUP construction permits, trading across borders 23. Answer : A and resolving insolvency. Hence, option C is correct. Explanation: World bank releases the Ease of Doing Business annually. In 2019 edition, India Educational objective: To have knowledge improved in its rank by 14 places bagging 63rd about the important reports by world www.laex.in Page No. 149 https://elearn.laex.in

www.laex.in

ECONOMY UPSC

Previous Year Questions

organizations. Also know about the India’s rank 29. Answer : D and performance in it. Explanation: The United Nations (UN) is

26. Answer : D an intergovernmental organization that aims to maintain international peace and security, Explanation: World Bank has released the develop friendly relations among nations, report “Global Economic Prospects: Heightened achieve international cooperation, and be a Tensions, Subdued Investment”. Global center for harmonizing the actions of Economic Prospects is a biannual report, last nations. The UN one was published in January 2019. is headquartered on international Educational objective: To have knowledge territory in New York City; about the important reports by world International labour Organization (ILO), Food organizations. Also know about the India’s rank and Agriculture Organization of the United and performance in it.

Nations (FAO), United Nations Educational, 27. Answer : A Scientific and Cultural Organization (UNESCO), Explanation: The International Development World Health Organization (WHO), World Bank Association (IDA) is an international financial Group International Monetary Fund (IMF), institution which offers International Fund for Agricultural Development concessional loans and grants to the world's (IFAD) etc., are the agencies of UN poorest developing countries. It was established Educational objective: To learn about the in 1960 to complement the agencies of UN along with their headquarters.

existing International Bank for 30. Answer : C Reconstruction and Development by lending to developing countries. Hence, option A is Explanation: The World Development correct Report (WDR) is an annual report published since 1978 by the International Bank for Educational objective: To have knowledge Reconstruction and Development (IBRD) about the world bank and it’s organizations

or World Bank. 28. Answer : B The theme was attacking poverty. Explanation: The World Development The recent edition studies how the nature of Report (WDR) is an annual report published work is changing as a result of advances in since 1978 by the International Bank for technology today. Reconstruction and Development (IBRD) Educational objective: To have knowledge or World Bank. Hence, option B is correct. about the important reports by world Educational objective: To have knowledge organizations.

about the important reports by world organizations.

www.laex.in Page No. 150 https://elearn.laex.in

www.laex.in

ECONOMY UPSC

Previous Year Questions

31. Answer : B The Bank shall support public or private Explanation: projects through loans, guarantees, equity participation and other financial instruments. Institution Function Educational objective: To have knowledge Multilateral trade negotiation WTO about different multilateral banks and their body objectives along with the Headquarters. IDA Sanction of soft loans 34. Answer : A Provides loans to address Explanation: Option 2 is wrong. As the IMF short term balance of population of India and other country like payment problems Brazil and Russia exceeds the population of Facilitating lending and China. borrowing for Educational objective- to have idea about the IBRD reconstruction and trends in GDP growth of various countries. And development also about the population trends.

Educational objective: to have knowledge about the functions of various organizations of 12.5 WORLD ECONOMIC FORUM

the world bank group. 35. Answer : A

32. Answer : C Explanation: Professor Klaus Schwab was born Explanation: The correct option is C when in Ravensburg, Germany in 1938. He is we consider the world development report of Founder and Executive Chairman of the 1994. World Economic Forum, the International Educational objective: to have an idea of the Organization for Public-Private Cooperation. per capita GNP value. Hence, option A is correct.

12.4 BRICS Educational objective: to have an idea of the founders of important organizations.

33. Answer : B 36. Answer : B Explanation: New development bank was set up by the BRICS ( Brazil, Russia, India, China Explanation: World Economic Forum releases and South Africa) Formerly referred to as the global competitiveness report annually. the BRICS Development Bank, is a multilateral India has moved down 10 places to rank 68th development bank. Hence, statement 1 is on an annual global competitiveness index, incorrect. largely due to improvements witnessed by several other economies, while Singapore has The Bank has its headquarters in Shanghai, replaced the US as the world's most China. Hence, statement 2 is correct competitive economy.

www.laex.in Page No. 151 https://elearn.laex.in

www.laex.in

ECONOMY UPSC

Previous Year Questions

Educational objective: To have knowledge respective State citizenship. Hence, about the top countries in the important statement 3 is correct. reports. Also know about India’s rank. The EU traces its origins to the European Coal

and Steel Community (ECSC) and 12.6 EUROPEAN UNION the European Economic Community (EEC), es

37. Answer: A tablished, respectively, by the 1951 Treaty of Paris and 1957 Treaty of Rome. Hence, Explanation: The European Council statement 1 is correct. is the EU's supreme political body. The European Commission is part political Switzerland is not a member of European body and part administration, which leads to union. Hence, statement 4 is incorrect. confusion about its exact nature. Also, it Educational objective: To have knowledge represents EU in trade negotiations. Hence, about European Union as it is one of important statement 1 is correct. economic union of world.

The European Council brings together the 12.7 OTHER MULTILATERAL political leaders – the Presidents and Prime ORGANIZATIONS Ministers – of the member states. Hence, 39. Answer : A statement 2 is incorrect. Explanation :AIIB is an international financial Educational objective: To have knowledge institution. It aims to support the building of about European Union as it is one of important infrastructure in the Asia Pacific region. It economic union of world.

Started operations in 2016 38. Answer : D  HQ : Beijing, China Explanation: The European Union is a group of  86 members (it keeps changing as new 28 countries that operate as a cohesive members join) economic and political block. The Maastricht  The capital of the bank is $100 billion, Treaty-1992 (also called the Treaty on equivalent to 2/3 of the capital of the European Union) was signed on 7 February Asian Development Bank and about half 1992 by the members of the European that of the World Bank Community in Maastricht, Netherlands to  China is the largest shareholder with further European integration. Hence, 26.06% voting shares. India is the statement 2 is correct. second largest shareholder with 7.5% European Communities (ECSC, EAEC, and voting shares followed by Russia (5.93%) EEC) incorporated as European Union. and Germany (4.5%). Hence 2 is wrong European citizenship was created, allowing  Germany, France are some countries citizens to reside in and move freely between outside Asia. Hence 3 is wrong Member States. Also, citizens have their

www.laex.in Page No. 152 https://elearn.laex.in

www.laex.in

ECONOMY UPSC

Previous Year Questions

Educational objective – to know about various received high marks for its housing, infrastructure and investment banks in India transportation and food choices. and the world. Educational objective – to have idea about

important reports and Indices and the 40. Answer : B parameters that they use to rank the countries. Explanation : SAFTA refers to the “Agreement

on South Asian Free Trade Area (SAFTA)”. This 42. Answer : D agreement was reached in 2004 in the 12th Explanation : SAARC summit at Islamabad. The objective was to create a free trade area comprising Bangladesh, Bhutan, India, Maldives, Nepal, Pakistan and Sri Lanka. This agreement came into force in 2006 and is currently NOT fully operational. Hence 1 is wrong. As per SAFTA agreement terms, India, Pakistan and Sri Lanka have to decrease their custom First three options are part of World bank duties to the level of 0 to 5 percent by the year group as shown in above table. Hence they 2013. Hence 2 is correct. are part of United Nations. Educational objective – to have knowledge The Bank for International Settlements (BIS) about the regional groupings in which India is is an international financial institution that part of. And also about the Free Trade aims to promote global monetary and financial Agreements that India has signed and its stability through the coordination of global commitments. central banks and their monetary policy efforts.

Hence it is not part of United Organisations 41. Answer : B Educational Objective – to know about the Explanation : Greendex is compiled annually United Nations and Special agencies established by National Geographic and GlobeScan, under UNO.

measures the way consumer patterns are responding to environmental concerns. 43. Answer : C The countries researchers examined included Explanation : Australia, India, China, the United States, the United Kingdom, Brazil, South Korea, Argentina, Hungary, Russia, Mexico, Sweden, Spain, South Africa, Germany, France, Japan and Canada. India came in first, pulling in a Greendex score of 61.4. In particular, the country

www.laex.in Page No. 153 https://elearn.laex.in

www.laex.in

ECONOMY UPSC

Previous Year Questions

Educational objective – to have knowledge 45. Answer : C about important international groupings like Explanation : The International Organization OPEC, OECD, SAARC, APEC, ASEAN etc. for Standardization (ISO) is an international 44. Answer : C standard-setting body composed of Explanation : representatives from various national standards  OPEC was founded in Baghdad, Iraq, with the organizations. signing of an agreement in 1960 Founded on 23 February 1947, the organization  Founding Members- Islamic Republic of Iran, promotes worldwide proprietary, industrial, and Iraq, Kuwait, Saudi Arabia and Venezuela. commercial standards.  13 Member Countries. Congo (2018) was the It is headquartered in Geneva, Switzerland, recent one to join. and works in 164 countries. Hence 1 is wrong.  Qatar terminated its membership on 1 The ISO 9000 family of quality management January 2019. systems is a set of standards that helps  OPEC membership is open to any country that organizations ensure they meet customers and is a substantial exporter of oil and which other stakeholder needs within statutory and shares the ideals of the organization. regulatory requirements related to a product or  Headquarters - Vienna, Austria. service. Hence 2 is correct.  Objective - to co-ordinate and unify ISO 14000 is a family of standards related to petroleum policies among Member Countries, environmental management that exists to help in order to secure fair and stable prices for organizations minimize how their operations petroleum producers negatively affect the environment; hence 3 is correct. Educational objective – to have knowledge about the organizations related to product quality enhancement and standards

46. Key : D Explanation : UNCTAD is a permanent intergovernmental body established by the United Nations General Assembly in 1964. Our

headquarters are located in Geneva, Educational objective – to have knowledge Switzerland. about important international groupings like It supports developing countries to access OPEC, OECD, SAARC, APEC etc. the benefits of a globalized economy more fairly and effectively. Along with other UN departments and agencies, it also measures

the progress made in the Sustainable

www.laex.in Page No. 154 https://elearn.laex.in

www.laex.in

ECONOMY UPSC

Previous Year Questions

Development Goals, as set out in Agenda economies and the advent of regional trade 2030. Given countries are all emerging blocs in other parts of the world; it aimed to economies. Hence option d is correct. establish new markets for agricultural products Educational objective: To learn about UNCTAD and raw materials beyond Europe. and emerging economies in the world. Headquartered in Singapore, APEC is recognized as one of the highest-level multilateral blocs and 47. Answer : B oldest forums in the Asia-Pacific region, and Explanation :BN AMRO Bank N.V. is exerts a significant global influence. a Dutch bank with headquarters in Amsterdam. 13th Asia-Pacific Economic Co-operation ABN AMRO Bank is the third-largest bank in Forum was held in the year 2001 in Shangai, the Netherlands. It was re-established in its China. current form in 2009, following the acquisition 32nd Asia-Pacific Economic Co-operation and break-up of the original ABN AMRO. Hence Forum is going to be held in Kaula Lumpur, pair 1 is wrong. Malaysia in 2020. Barclays Bank is a British Educational objective: To learn about Asia- multinational investment bank and financial Pacific Economic Co-operation. services company, headquartered in London,

England. Apart from investment banking, 49. Answer : A Barclays is organised into four core Explanation : The Commonwealth of Nations, businesses: personal banking, corporate generally known simply as banking, wealth management, and investment the Commonwealth, is a political association management. Hence pair 2 is correct. of 54 member states, nearly all KB Kookmin Bank or KB is among the four former territories of the British Empire. Few largest banks ranked by asset value in South Middle Eastern countries and also Myanmar did Korea at the end of March 2014. It is the largest not join the Commonwealth. Hence statement 2 is among banks in Korea and 60th largest in the incorrect. world by 2017. Hence pair 3 is wrong. The charter of the Commonwealth was adopted Educational objective: To learn about on 19 December 2012 and was officially signed international economy and its banks. by Queen Elizabeth II in London, on the commonwealth day on 11 March 2013. Hence 48. Answer : D statement 1 is incorrect. Explanation : The Asia-Pacific Economic Educational objective: To learn about the Cooperation (APEC) is an inter-governmental Commonwealth of Nations and its charter. forum for 21 member economies in the Pacific Rim that promotes free trade throughout the Asia-Pacific region. APEC started in 1989, in response to the growing interdependence of Asia-Pacific www.laex.in Page No. 155 https://elearn.laex.in

www.laex.in

ECONOMY UPSC

Previous Year Questions

13.Indicies And Report 2) National Council of Applied Economic Research 1. The Multi-dimensional Poverty Index 3) Indira Gandhi Institute of Development developed by Oxford Poverty and Research Human Development Initiative with 4) World Bank UNDP support covers which of the List-II following? 1. UN India Human Development Report

1) Deprivation of education, health, assets 2. India Development Report and services at household level 3. World Development Report 2) Purchasing power parity at national level. 4. Human Development Report 3) Extent of budget deficit and GDP growth Codes: rate at national level a) A-4; B-1; C-2; D-3 Select the correct answer using the codes b) A-4; B-2; C-1; D-3 given below: c) A-2; B-3; C-4; D-1 a) 1 only d) A-2; B-1; C-4; D-3 b) 2 and 3 only 4. The misery index is the sum of a c) 1 and 3 only country's unemployment and inflation d) 1, 2 and 3

rate. The higher the index, the more 2. Which of the following is/are the miserable is the country to live in. The indicator /indicators used by IFPRI to figure given below is the Misery Index compute the Global Hunger Index for various countries in Europe: Report?

1. Undernourishment 2. Child stunting 3. Child mortality Select the correct answer using the code given below. a) 1 only b) 2 and 3 only c) 1,2 and 3 d) 1 and 3 only Which of the following conclusions can be 3. Match List-I with List- II and select the drawn from the misery index given correct answer using the codes given above? below the lists: 1) Britain is the most miserable country to List-I live in 1) United Nations Development Programme 2) The inflation rate in Spain is less than in Belgium and Britain www.laex.in Page No. 156 https://elearn.laex.in

www.laex.in

ECONOMY UPSC

Previous Year Questions

3) Italy and France seem to have almost or more of the weighted indicators are identical unemployment rate considered living in extreme multidimensional 4) The higher the misery index, the higher poverty. the inflation rate. Educational Objectives: To learn about MPI

Select the correct answer using the codes 2. Answer: C given below: Explanation: The report is a peer-reviewed a) 1 alone publication released annually b) 2 and 3 by Welthungerhilfe and Concern Worldwide. c) 1, 2, 3 and 4 The GHI ranks countries on a 100-point d) None scale, with 0 being the best score (no hunger)

13. Key and Explanations and 100 being the worst. Values less than 10

reflect low hunger, values from 20 to 34.9 1. Answer: A indicate serious hunger; values from 35 to 49.9 Explanation: The Multidimensional Poverty are alarming; and values of 50 or more are Index was launched by the UNDP and the extremely alarming. Oxford Poverty & Human Development Initiative The GHI scores are based on a formula that (OPHI) in 2010. captures three dimensions of hunger— MPI is based on the idea that poverty is not insufficient caloric intake, child under nutrition, unidimensional (not just depends on income and child mortality—using four component and one individual may lack several basic needs indicators: like education, health etc.), rather it is  Undernourishment: the share of the multidimensional. population that is under-nourished, The index shows the proportion of poor people reflecting insufficient caloric intake and the average number of deprivations each  Child Wasting: the share of children under poor person experiences at the same time. the age of five who are wasted (low MPI uses three dimensions and ten indicators weight-for-height), reflecting acute under which are: nutrition. Education: Years of schooling and child  Child Stunting: the share of children enrollment (1/6 weightage each, total 2/6); under the age of five who are stunted Health: Child mortality and nutrition (1/6 (low height-for-age), reflecting chronic weightage each, total 2/6); under nutrition. Standard of living: Electricity, flooring,  Child Mortality: the mortality rate of drinking water, sanitation, cooking fuel and children under the age of five. assets (1/18 weightage each, total 2/6) India’s rank has slipped from 95th position (in A person is multi dimensionally poor if she/he 2010) to 102nd (in 2019). Over a longer-term is deprived in one third or more (means 33% or duration, the fall in India’s rank is sharper, i.e, more) of the weighted indicators (out of the ten indicators). Those who are deprived in one half www.laex.in Page No. 157 https://elearn.laex.in

www.laex.in

ECONOMY UPSC

Previous Year Questions

from 83rd out of 113 countries in 2000 to Each WDR provides in-depth analysis of a 102nd out of 117 in 2019. specific aspect of economic development. Past reports have considered such topics as According to the report, India’s child wasting agriculture, youth, equity, public rate was extremely high at 20.8% - services delivery, the role of the state, transition the highest for any country. economies, labour, infrastructure, health, the Child wasting refers to the share of children environment, risk management, and poverty. under the age of five who are wasted, i.e, they The reports are the Bank's best-known have low weight with respect to their height, contribution to thinking about development. reflecting acute under nutrition. Educational Objectives: To learn about The share of wasting among children in India different Reports

marked a steep rise from 16.5% in the 2008- 4. Answer: D 2012 to 20.8% in 2014-2018. Explanation: The first misery index was created According to United Nations Children's Fund by Arthur Okun in 1960’s and was equal to (UNICEF), child wasting is a strong predictor of the sum of inflation and unemployment rate mortality among children (under 5 yrs. of age). figures to provide a snapshot of the US Educational Objectives: To know about focus economy. areas and objectives of GHI report. The higher the index, the more is the misery felt

3. Answer: A by average citizens. Explanation: It has broadened in recent times to include Human Development Report that is published other economic indicators, such as bank lending by the United Nations Development rates. Programme (UNDP). In recent times, variations of the original misery The indices that form the part of the 2019 index have become popular as a means to gauge Report are: the overall health of the global economy. Inequality-adjustedHuman Development Index A variation of the original misery index is (IHDI), the Bloomberg misery index, developed by the Gender Development Index (GDI), online publication. Human development index Educational Objectives: To Know about misery Gender Inequality Index (GII) and index and its applicability.

Multidimensional Poverty Index (MPI). 14. MISCELLANEOUS

B. World Development Report (WDR) 14.1 Casual Factor The World Development Report (WDR) is an

annual report published since 1978 by 1. The economic crisis in the latter half of the International Bank for Reconstruction and 1990s most seriously affected Indonesia, Development (IBRD) or World Bank. Thailand, Malaysia and South Korea. The cause of the crisis was

www.laex.in Page No. 158 https://elearn.laex.in

www.laex.in

ECONOMY UPSC

Previous Year Questions

a) mismanagement of the financial resources 3) to provide for wage loss due to pregnancy and financial sector, in general and confinement b) the prolonged over-valuation of local Which of the statements given above is/are currencies vis-à-vis the western currencies correct? c) the downswing and recession in the western a) 1 and 2 only economies which earlier provided export b) 2 only market to these export oriented countries c) 3 only d) none of the above d) 1, 2 and 3

2. Nobel Prize in Economics for the year 5. Consider the following statements with 1997 was awarded for contribution in the reference to Indira Gandhi National Old area of: Age Pension Scheme (IGNOAPS): a) International Economics 1) All persons of 60 years or above belonging b) Financial, Economics to the households below poverty line in c) Public Economics rural areas are eligible. d) Development Economics 2) The Central Assistance under this Scheme

3. Consider the following statements: is at the rate of Rs. 300 per month per 1) Life Insurance Corporation of India is the beneficiary. Under the Scheme, States have oldest insurance company in India. been urged to give matching amounts. 2) National Insurance Company Limited was Which of the statements given above is/are nationalized in the year 1972 and made a correct? subsidiary of General Insurance a) 1 only Corporation of India. b) 2 only 3) Headquarters of United India Insurance c) Both1 and 2 Company Limited are located at Chennai. d) Neither 1 nor 2

Which of the statements given above are 6. Which one of the following is the correct? objective of National Renewal Fund? a) 1, 2 and 3 a) To safeguard the interests of workers who b) 1 and 2 only may be affected by technological up c) 2 and 3 only gradation of industry or closure of sick d) 1 and 3 only units

14.2 Government Schemes b) To develop the core sector of the economy c) For the development of infrastructure such 4. The endeavour of 'Janani Suraksha as energy, transport, communications and Yojana' Programme is irrigation 1) to promote institutional deliveries d) For human resource development such as 2) to provide monetary assistance to the full literacy, employment, population mother to meet the cost of delivery. control, housing and drinking water

www.laex.in Page No. 159 https://elearn.laex.in

www.laex.in

ECONOMY UPSC

Previous Year Questions

7. 'SWAYAM', an initiative of the 1) There is persistent investment over time Government of India, aims at only in select locals. a) promoting the Self Help Groups in rural 2) Some areas are agro-climatically less areas conducive to development b) providing financial and technical assistance 3) Some areas continue to face little or no to young start-up entrepreneurs agrarian transformation and the c) promoting the education and health of consequent lack of social and economic adolescent girls opportunities. d) providing affordable and quality education 4) Some areas have faced continuous political to the citizens for free instability

14.3 Population And Census Which of the above statements are correct? a) 1, 2 and 3 8. Consider the following specific stages of b) 1, 2 and 4 demographic transition associated with c) 1, 3 and 4 economic development d) 2, 3 and 4

1) Low birthrate with low death rate 2) High birthrate with high death rate 14. Key and Explanations 3) High birthrate with low death rate 14.1 Casual Factor Select the correct order of the above stages using the codes given below: 1. Answer : A a) 1,2, 3 Explanation: The Asian financial crisis was a b) 2, 1,3 period of financial crisis that gripped much c) 2,3, 1 of East Asia and Southeast Asia beginning in d) 3, 2, 1 July 1997 and raised fears of a worldwide economic meltdown due to financial contagion. 9. Consider the following statements: The crisis started in Thailand, with the financial 1) Between Census 1951 and Census 2001, collapse due to lack of foreign currency to the density of the population of India has support its currency peg to the U.S. increased more than three times. dollar. Capital flight ensued almost immediately, 2) Between Census 1951 and Census 2001, beginning an international chain reaction. At the annual growth rate (exponential) of the the time, Thailand had acquired a burden population of India has doubled. of foreign debt that made the country Which of the statements given above is/are effectively bankrupt even before the collapse of correct? its currency. a) 1 only As the crisis spread, most of Southeast Asia and b) 2 only Japan saw slumping currencies, devalued stock c) Both 1 and 2 markets and other asset prices, and a d) Neither 1 nor 2

precipitous rise in private debt. 10. Consider the following statements: www.laex.in Page No. 160 https://elearn.laex.in

www.laex.in

ECONOMY UPSC

Previous Year Questions

Indonesia, South Korea, and Thailand were the 14.2 Government Schemes countries most affected by the crisis. Hong 4. Answer : A Kong, Laos, Malaysia and the Philippines were Explanation: Janani Suraksha Yojana also hurt by the slump. (JSY) is a 100 % centrally sponsored Educational objective: To learn about scheme and it integrates cash assistance with financial crisis in East and South East Asia delivery and post-delivery care. during 1990s. Objectives: It was Launched for

2. Answer : B reducing maternal and neonatal mortality, to Explanation: The Royal Swedish Academy of promote institutional delivery among pregnant Sciences has awarded the 1987 Alfred Nobel women especially with weak socio-economic Memorial Prize in Economic Sciences to status i.e. women from Scheduled Castes, Professor Robert M. Solow, Massachusetts Scheduled Tribes and BPL households. Institute of Technology, Cambridge, USA for his Cash assistance: Under the JSY, eligible contributions to the theory of economic pregnant women are entitled to cash assistance growth. irrespective of the age of mother and number of Educational objective: To learn about areas of children for giving birth in a government or contribution to Economics from Nobel winners. accredited private health facility.

3. Answer : C ‘Pradhan Mantri Matru Vandana Yojana’ Explanation: National Insurance Company (PMMVY), a cash-based maternity benefit Limited (NICL) is the oldest non-life general scheme. The scheme, which aims to provide insurance company of India. It partial compensation for wage loss to women was established on 5th December 1906 at during their pregnancy and offers a cash Kolkata. Hence statement 1 is wrong. incentive of Rs 6,000 to mothers for the birth of The Government of India issued an Ordinance their first child, has so far reached 4.8 million on 19 January 1956 nationalizing the Life beneficiaries. Hence statement 3 is incorrect. Insurance sector and Life Insurance Corporation Educational objective: To learn about Janani came into existence in the same year. Suraksha Yojana (JSY).

United India Insurance Company is an Indian 5. Answer : D leading General Insurance company, fully Explanation: Ministry of Rural Development of owned by Government of India and is India has introduced Indira Gandhi National headquartered in Chennai, India. It was Old Age Pension Scheme (IGNOAPS) under incorporated on 18 February 1938, and was National Social Assistance Programme (NSAP) in nationalized in 1972. the year 2007. IGNOAPS also called as National Educational objective: To learn about life Old Age Pension Scheme (NOAPS). insurance companies in india. The eligible age for IGNOAPS is 60 years. The pension is Rs.200 p.m. for persons between 60

years and 79 years. For persons who are 80

www.laex.in Page No. 161 https://elearn.laex.in

www.laex.in

ECONOMY UPSC

Previous Year Questions

years and above the pension is Rs.500/ - per there are not enough health facilities available, the month. Hence both the given statements are population is unaware of the health and hygiene incorrect. practices to be followed. Educational objective: To learn about Indira As development happens, improvements in Gandhi National Old Age Pension Scheme hygiene, sanitation, food availability reduces the (IGNOAPS). death rate. This is the phase when population

6. Answer : A growth quickens. With further development, Explanation: The main objective of the National education and literacy, the population is apprised Renewal Fund was to provide a social safety net of family planning and contraception which checks to the workers who are likely to be affected by the birth rate as well. technological up-gradation and modernisation Educational objective: To learn about the in the Indian industry. specific stages of demographic transition.

NRF was intended to provide funds for 9. Answer : D employment generation schemes in the Explanation: Density of population refers to organised and unorganised sectors in order to average number of people living per square kilo provide a social safety net for labour. Hence meter. Density of population in a country is option a is correct. measured by dividing its total population by Educational objective: To learn about the total land area. National Renewal Fund and its objectives. Density of population in 1951 was 117 per sq

7. Answer : D km and in 2001 it was 324 per sq km. Explanation: Study Webs of Active Learning Annual growth rate (exponential) of the for Young Aspiring Minds (SWAYAM), was population of India in 1951 was 1.25% and in launched on July 9, 2017 by the Ministry of 2001 was 1.93%. It is clear that Between Human Resource Development to provide one Census 1951 and Census 2001, the annual integrated platform and portal for online growth rate of the population of India has NOT courses. doubled. This covers all higher education subjects and Educational objective: To learn about the skill sector courses. census highlights.

The objective is to ensure that every student 10. Answer : A in the country has access to the best quality Explanation: Regional Development Disparity: higher education at the affordable cost. It refers to difference in economic development Educational objective: To learn about and uneven economic achievement in different SWAYAM SCHEME and its importance. geographical regions.

14.3 Population and Census It is reflected by the indicators like per capita income, the proportion of population living 8. Answer : C below the poverty line, the percentage of urban Explanation: Countries are initially characterized population, percentage of population engaged in with high birth rate and high death rate as the www.laex.in Page No. 162 https://elearn.laex.in

www.laex.in

ECONOMY UPSC

Previous Year Questions

agriculture vis-à-vis engaged in industries, infrastructural development of different states. Educational objective: To learn about Regional Development Disparity.

www.laex.in Page No. 163 https://elearn.laex.in

www.laex.in

ECONOMY UPSC

Previous Year Questions

www.laex.in Page No. 164 https://elearn.laex.in

www.laex.in